You are on page 1of 358

MICROBIOLOGY

MEDICAL COURSE AND STEP 1 REVIEW


FIRST EDITION

Accompanies online videos taught by


Michael Christensen & Rhett Thomson
physeo.com
Copyright © 2018 by Physeo

All rights reserved. No part of this publication may be reproduced,


distributed, or transmitted in any form or by any means, including
photocopying, recording, or other electronic or mechanical
methods, without the prior written permission of Physeo, except in
the case of personal study purposes.
TABLE OF CONTENTS
MICROBIOLOGY FUNDAMENTALS................................................................................6
Section 1 - Bacterial Structures...................................................................................................................................................... 6
Section 2 - Virulence.................................................................................................................................................................... 10
Section 3 - Bacterial Genetics...................................................................................................................................................... 20
Section 4 - Prion Diseases............................................................................................................................................................ 27
Section 5 - Introduction to Fungi................................................................................................................................................. 29
Section 6 - Viral Structures.......................................................................................................................................................... 32
Section 7 - Viral Genetics............................................................................................................................................................ 35
Section 8 - Viral Genomes and Replication................................................................................................................................. 38

BACTERIA.............................................................................................................................41
Section 1 - Clostridium tetani...................................................................................................................................................... 42
Section 2 - Clostridium botulinum............................................................................................................................................... 44
Section 3 - Clostridium difficile .................................................................................................................................................. 46
Section 4 - Clostridium perfringens............................................................................................................................................. 48
Section 5 - Bacillus anthracis...................................................................................................................................................... 50
Section 6 - Bacillus cereus........................................................................................................................................................... 52
Section 7 - Listeria monocytogenes............................................................................................................................................. 54
Section 8 - Corynebacterium diphtheriae ................................................................................................................................... 56
Section 9 - Viridans streptococci................................................................................................................................................. 58
Section 10 - Streptococcus pneumoniae...................................................................................................................................... 60
Section 11 - Streptococcus pyogenes........................................................................................................................................... 62
Section 12 - Streptococcus agalactiae......................................................................................................................................... 66
Section 13 - Streptococcus gallolyticus....................................................................................................................................... 68
Section 14 - Enterococci.............................................................................................................................................................. 70
Section 15 - Staphylococcus aureus............................................................................................................................................. 72
Section 16 - Staphylococcus epidermidis.................................................................................................................................... 74
Section 17 - Staphylococcus saprophyticus................................................................................................................................. 76
Section 18 - Nocardia ................................................................................................................................................................. 78
Section 19 - Actinomyces israelii................................................................................................................................................. 80
Section 20 - Neisseria overview ................................................................................................................................................. 82
Section 21 - Neisseria meningitidis............................................................................................................................................. 84
Section 22 - Neisseria gonorrhoeae ........................................................................................................................................... 86
Section 23 - Moraxella catarrhalis ............................................................................................................................................. 88
Section 24 - Haemophilus influenzae........................................................................................................................................... 90
Section 25 - Bordetella pertussis................................................................................................................................................. 94
Section 26 - Pasteurella multocida, Brucella spp., and Francisella tularensis........................................................................... 96
Section 27 - Salmonella spp......................................................................................................................................................... 98
Section 28 - Salmonella typhi.................................................................................................................................................... 100
Section 29 - Shigella.................................................................................................................................................................. 102
Section 30 - E. coli..................................................................................................................................................................... 104
Section 31 - Klebsiella............................................................................................................................................................... 108
Section 32 - Proteus mirabilis................................................................................................................................................... 110
Section 33 - Pseudomonas aeruginosa...................................................................................................................................... 112
Section 34 - Yersinia pestis and enterocolitica.......................................................................................................................... 116
Section 35 - Enterobacter, Citrobacter, & Serratia marcescens................................................................................................ 118
Section 36 - Bartonella henselae .............................................................................................................................................. 120
Section 37 - Legionella pneumophila........................................................................................................................................ 122
Section 38 - Campylobacter jejuni............................................................................................................................................ 124
Section 39 - Vibrio cholerae & Vibrio vulnificus....................................................................................................................... 126
Section 40 - Helicobacter pylori................................................................................................................................................ 128
Section 41 - Mycobacterium tuberculosis.................................................................................................................................. 130
Section 42 - Mycobacterium leprae .......................................................................................................................................... 134
Section 43 - Mycobacterium avium-intracellulare, Scrofulaceum, and Marinum.................................................................... 138
Section 44 - Borrelia burgdorferi.............................................................................................................................................. 140
Section 45 - Leptospira interrogans........................................................................................................................................... 142
Section 46 - Treponema pallidum.............................................................................................................................................. 144
Section 47 - Chlamydia trachomatis, Chlamydophila pneumoniae, and Chlamydophila psittaci............................................ 148
Section 48 - Coxiella burnetii.................................................................................................................................................... 150
Section 49 - Gardnerella vaginalis............................................................................................................................................ 152
Section 50 - Mycoplasma pneumoniae...................................................................................................................................... 154
Section 51 - Rickettsia prowazekii, Rickettsia rickettsii, and Rickettsia typhi.......................................................................... 156
Section 52 - Ehrlichia & Anaplasma......................................................................................................................................... 158
Section 53 - Miscellaneous Organisms...................................................................................................................................... 160

FUNGI...................................................................................................................................165
Section 1 - Histoplasma capsulatum.......................................................................................................................................... 166
Section 2 - Blastomyces dermatitidis......................................................................................................................................... 168
Section 3 - Coccidioides immitis................................................................................................................................................ 172
Section 4 - Paracoccidioides brasiliensis.................................................................................................................................. 174
Section 5 - Microsporum, Trichophyton, and Epidermophyton................................................................................................. 176
Section 6 - Malassezia furfur..................................................................................................................................................... 180
Section 7 - Sporothrix schenckii................................................................................................................................................ 182
Section 8 - Candida albicans..................................................................................................................................................... 184
Section 9 - Aspergillus fumigatus.............................................................................................................................................. 188
Section 10 - Cryptococcus neoformans..................................................................................................................................... 190
Section 11 - Mucor & Rhizopus spp.......................................................................................................................................... 192
Section 12 - Pneumocystis jirovecii........................................................................................................................................... 194

PARASITES.........................................................................................................................197
Section 1 - Giardia lamblia....................................................................................................................................................... 198
Section 2 - Entamoeba histolytica............................................................................................................................................. 200
Section 3 - Cryptosporidium...................................................................................................................................................... 202
Section 4 - Toxoplasma gondii................................................................................................................................................... 204
Section 5 - Naegleria fowleri..................................................................................................................................................... 206
Section 6 - Trypanosoma brucei, gambiense, and rhodesiense................................................................................................. 208
Section 7 - Malaria.................................................................................................................................................................... 210
Section 8 - Babesia.................................................................................................................................................................... 214
Section 9 - Trypanosoma cruzi.................................................................................................................................................. 216
Section 10 - Leishmania donovani............................................................................................................................................. 218
Section 11 - Trichomonas vaginalis........................................................................................................................................... 220
Section 12 - Enterobius vermicularis (pinworm)...................................................................................................................... 222
Section 13 - Ascaris lumbricoides (giant roundworm).............................................................................................................. 224
Section 14 - Strongyloides stercoralis (threadworm)................................................................................................................ 226
Section 15 - Ancylostoma duodenale and Necator americanus (hookworms).......................................................................... 228
Section 16 - Trichinella spiralis................................................................................................................................................. 230
Section 17 - Trichuris trichiura (whipworm)............................................................................................................................ 232
Section 18 - Toxocara canis....................................................................................................................................................... 234
Section 19 - Onchocerca volvulus............................................................................................................................................. 236
Section 20 - Loa loa and Wuchereria bancrofti......................................................................................................................... 238
Section 21 - Taenia solium......................................................................................................................................................... 240
Section 22 - Diphyllobothrium latum........................................................................................................................................ 243
Section 23 - Echinococcus granulosus...................................................................................................................................... 244
Section 24 - Schistosoma haematobium, mansoni, and japanicom........................................................................................... 246
Section 25 - Clonorchis sinensis................................................................................................................................................ 248
Section 26 - Sarcoptes scabiei................................................................................................................................................... 250
Section 27 - Pediculus corporis, pubis and capitis.................................................................................................................... 252

VIRUSES..............................................................................................................................254
Section 1 - Herpes simplex virus-1 (HSV-1) and herpes simplex virus-2 (HSV-2).................................................................. 255
Section 2 - Varicella-zoster virus (VZV)................................................................................................................................... 260
Section 3 - Epstein-Barr Virus (EBV)....................................................................................................................................... 262
Section 4 - Cytomegalovirus (CMV)......................................................................................................................................... 264
Section 5 - Roseola (HHV-6 and HHV-7).................................................................................................................................. 266
Section 6 - Kaposi sarcoma (HHV-8)........................................................................................................................................ 268
Section 7 - Hepatitis B Virus (HBV)......................................................................................................................................... 270
Section 8 - Smallpox, Cowpox, and Molluscum contagiosum.................................................................................................. 274
Section 9 - Human papillomavirus............................................................................................................................................ 276
Section 10 - Adenovirus............................................................................................................................................................. 280
Section 11 - JC Virus and BK Virus........................................................................................................................................... 282
Section 12 - Parvovirus B19...................................................................................................................................................... 284
Section 13 - Coronavirus and SARS.......................................................................................................................................... 286
Section 14 - Human immunodeficiency virus (HIV)................................................................................................................. 288
Section 15 - Human T-lymphotropic virus (HTLV)................................................................................................................... 292
Section 16 - Hepatitis C Virus (HCV)....................................................................................................................................... 294
Section 17 - Yellow fever .......................................................................................................................................................... 296
Section 18 - Dengue Fever......................................................................................................................................................... 298
Section 19 - West Nile Virus and St. Louis Encephalitis........................................................................................................... 300
Section 20 - Zika Virus.............................................................................................................................................................. 302
Section 21 - Rubella................................................................................................................................................................... 304
Section 22 - Eastern, Western, and Venezuelan equine encephalitis viruses; Chikungunya virus............................................ 306
Section 23 - California Encephalitis.......................................................................................................................................... 308
Section 24 - Rift Valley Fever and Crimean-Congo Hemorrhagic Fever Virus........................................................................ 310
Section 25 - Hantavirus.............................................................................................................................................................. 312
Section 26 - Influenza virus....................................................................................................................................................... 314
Section 27 - Lassa fever encephalitis and Lymphocytic choriomeningitis virus....................................................................... 316
Section 28 - Hepatitis D Virus................................................................................................................................................... 317
Section 29 - Parainfluenza Virus................................................................................................................................................ 320
Section 30 - RSV and hMPV..................................................................................................................................................... 322
Section 31 - Measles.................................................................................................................................................................. 324
Section 32 - Mumps................................................................................................................................................................... 326
Section 33 - Rabies Virus........................................................................................................................................................... 328
Section 34 - Ebola Virus and Marburg Virus............................................................................................................................. 332
Section 35 - Norovirus............................................................................................................................................................... 334
Section 36 - Hepatitis E virus.................................................................................................................................................... 336
Section 37 - Rotavirus and Colorado Tick Fever....................................................................................................................... 338
Section 38 - Poliovirus............................................................................................................................................................... 340
Section 39 - Echovirus............................................................................................................................................................... 342
Section 40 - Rhinovirus............................................................................................................................................................. 344
Section 41 - Coxsackieviruses (A & B)..................................................................................................................................... 346
Section 42 - Hepatitis A Virus.................................................................................................................................................... 348
We would like to extend a special thanks to the following individuasl who have spent many
hours tutoring, guiding and consulting this work, making Physeo Anatomy possible.

Paloma F Cariello, MD, MPH


Assistant Professor
Division of Infectious Diseases
University of Utah School of Medicine
Salt Lake City, UT

Karen Eilbeck, Ph.D.


Professor
Biomedical Informatics
University of Utah

Vicente Planelles, Ph.D.


Professor
Division of Microbiology and Immunology
Department of Pathology
University of Utah School of Medicine
Salt Lake City, UT
6

MICROBIOLOGY FUNDAMENTALS
Section 1 - Bacterial Structures

I. There are several bacterial structures to be familiar with for board exams. These structures can help bacteria
infect host cells and survive. Some structures are unique to certain groups and can be useful in distinguishing
bacterial organisms.

Structure Description
• Phospholipid bilayer
• Contains lipoteichoic acids in gram-positive bacteria → stimulates host immune response
Cytoplasmic membrane
via TNF-ɑ and IL-1 release
• Contains penicillin-binding proteins (PBPs)
• Peptidoglycan that provides rigidity and structural support to cell
• Thick in gram-positive bacteria (gram staining with crystal violet causes blue appearance)
• Thin in gram-negative bacteria
Cell wall
• Absent in Mycoplasma species which have added sterols to maintain stability (must grow
on Eaton’s agar)
• Contains mycolic acid in Mycobacteria species (must use acid-fast stain)
Flagella • Provides bacterial motility
• Allows bacteria to bind to host cells
Pilus (fimbria)
• Allows bacteria-bacteria binding during conjugation
• Network of polysaccharides
• Slime layer: disorganized and loosely attached to cell wall → creates biofilm on prosthetics
Glycocalyx (capsule or (eg, S. epidermidis)
slime layer) • Capsule: organized and firmly attached to cell wall → protects from phagocytosis (targeted
by many vaccines)
• Note: the capsule of Bacillus anthracis is made of a protein network of poly-ᴅ-glutamate
• Only present in gram-negative organisms (gram counter-staining with safranin causes red
appearance)
Outer membrane • Contains porins → transfer of nutrients
• Contains lipopolysaccharides (LPS: lipid A, core polysaccharide, and O antigen)
• Lipid A stimulates host immune response via TNF and IL-1 release → fever, shock, diarrhea
• Located between outer membrane and cytoplasmic membrane in gram-negative bacteria
Periplasm
• Contains hydrolytic enzymes (eg, β-lactamases)
• Layer formed during dormant state → resists heat, chemicals, dehydration and starvation
→ survival of bacteria
• Only present in gram-positive organisms (Clostridium and Bacillus)
Endospore
• Center containing DNA and dipicolinic acid (DPA) → cytoplasmic membrane (spore core)
→ peptidoglycan wall (spore cortex) → cytoplasmic membrane → keratin-like protein wall
(spore coat) → exosporium (present only in B. anthracis and B. cereus)
Table 5.1.1 - Bacterial structures
7

Figure 5.1.1 - Gram positive bacterial envelope

Figure 5.1.2 - Gram negative bacterial envelope


8

Figure 5.1.3 - Envelope of Mycoplasma spp.

Figure 5.1.4 - Envelope of Mycobacteria spp.

Figure 5.1.5 - Acid-fast stain of Mycobacterium Tuberculosis


9

Figure 5.1.6 - Bacterial endospore

A. Polymerases in bacteria
1. DNA polymerase (replication → more DNA)
a) Mistakes can harm progeny
2. RNA polymerase (transcription → mRNA)
a) Mistakes do not harm progeny

REVIEW QUESTIONS ?
1. A researcher is attempting to create a novel • Correct answer: C
staining agent that will only bind to certain • The question stem describes a gram
bacterial species. This species cannot form an positive organism
endospore, does not contain mycolic acids, and • Contains only one phospholipid
contains only one phospholipid bilayer. Which of bilayer, so its not gram negative
the following does the organism described most • Does not contain mycolic acids, so
likely possess? its not mycobacteria
• Gram positive organisms may contain
A. A cytokine inducing structure in outer
flagella
membrane
B. A layer that tightly binds to the safranin • Choice A is incorrect because this describes
counterstain the endotoxin of gram negative bacteria
C. A protein structure allowing for motility • Choice B is incorrect because Safranin binds
D. A relatively thin layer of peptidoglycan to the outer membrane found only in gram
negative bacteria
• Choice D is incorrect because thin
peptidoglycan layers are found in
Mycobacteria and gram negative organisms,
not in gram positive bacteria
10

Section 2 - Virulence

I. Virulence describes the bacteria’s ability to do 3 things:


A. Invade the host
B. Evade immune defenses
C. Inflict disease

Virulence Factor Action Role in Virulence


Pili Adherence Invade the host

IgA protease Adherence Invade the host

Flagella Motility Invade the host

Capsule Prevent phagocytosis Evade immune defenses

M protein Prevent phagocytosis Evade immune defenses

Protein A Prevent phagocytosis Evade immune defenses

Type III secretion system Direct injection of exotoxins into host cell Inflict disease

Endotoxins Triggers host immune response Inflict disease

Exotoxins Secreted from bacteria (activities vary) Inflict disease


Table 5.1.2 - Overview of virulence factors
II. IgA protease IV. Protein A
A. Cleaves IgA antibodies A. Binds to the Fc portion of IgG antibodies → only
the Fab portion of IgG is exposed
B. Allows bacteria to adhere to mucosal surfaces
B. Prevents opsonization and phagocytosis
C. Secreted by multiple bacteria:
C. On the surface of S. aureus
1. S. pneumoniae
2. H. influenzae type b
3. N. meningitidis
4. N. gonorrhoeae

III. M protein
A. Prevents opsonization and phagocytosis
B. Similar to antigens present in the heart →
antibodies that form against the M protein can
attack the heart as well → rheumatic fever
C. Utilized by S. pyogenes (GAS)
11

Figure 5.1.7 - Protein A mechanism


V. Type III secretion system
A. Protein structure that can insert into host cells
→ injects exotoxins directly into host cell
B. Utilized by several gram negative organisms:
1. Salmonella spp
2. Shigella spp
3. E. coli spp
4. P. aeruginosa
12

Photo Credit: (left) Pixie [CC BY-SA 3.0 (https://creativecommons.org/license/by-sa/3.0)]; (right) Schraidt O. Lefebre MD, Brunner MJ, Schmied WH, Schmidt A,
Radics J, Mechtier K, Galán JE, Marlovits TC [CC BY 2.5 (https://creativecommons.org/license/by-sa/2.5)]
Figure 5.1.8 - Visual demonstration of Type III secretion systems
VI. Endotoxins VIII. Exotoxins

A. Lipopolysaccharide (LPS) on the surface of gram A. Secreted proteins that inflict disease
negative bacteria B. Utilized by most gram positive and some gram
B. Lipid A is the toxic component of LPS negative bacteria

C. Bacterial cell death → release of lipid A → host C. Source of exotoxin genes:


release of cytokines (TNF-α, IL-1, and IL-6) → 1. Bacterial chromosome
sepsis → severe sepsis → septic shock
2. Plasmids
D. Source of endotoxin genes:
3. Lysogenic bacteriophages following
1. Bacterial chromosome lysogenic conversion (phage DNA can
produce protein products)
VII. Sepsis
A. Sepsis: “Life-threatening organ dysfunction
caused by a dysregulated host response to
infection” (2016 SCCM/ESICM task force)
B. Septic shock: vasopressor required (2016 SCCM/
ESICM task force)
C. Virulence factor triggers host immune response
→ organ dysfunction (hypotension)
1. Gram negative bacteria (lipid A)
2. Gram positive bacteria
3. Fungi
13

Exotoxin Mechanism Result


Diphtheria toxin (C.
Inhibit elongation factor 2 (EF-2) Blocked protein synthesis
diphtheriae)
Exotoxin A (P. aeruginosa) Inhibit elongation factor 2 (EF-2) Blocked protein synthesis
Remove adenine from rRNA on 60S ribosome (invades
Shiga toxin (Shigella spp) Blocked protein synthesis
GI cells)
Remove adenine from rRNA on 60S ribosome (does not
Shiga-like toxin (EHEC) Blocked protein synthesis
invade GI cells)
Stimulates Gαs subunit → ↑ adenylate cyclase →
Heat-labile toxin (LT) (ETEC) Watery diarrhea
↑cAMP → ↑ Cl- & H2O secreted in gut
Guanylate cyclase overactivation → ↑ cGMP → ↓ Cl-
Heat-stable toxin (ST) (ETEC) Watery diarrhea
and H2O reabsorption in the gut
Stimulates Gαs subunit → ↑ adenylate cyclase →
Cholera toxin (V. cholera) Watery diarrhea
↑cAMP → ↑ Cl- & H2O secreted in gut
Mimics adenylate cyclase → ↑ cAMP → cutaneous
Edema factor (B. anthracis) Edema around black eschar
edema
Blocks Gαi subunit → ↑ adenylate cyclase → ↑ cAMP →
Pertussis toxin (B. pertussis) Persistence of microbe
PMN chemotaxis impaired
Cleaves SNARE proteins → lack of inhibitory
Tetanospasmin (C. tetani) ↑ muscular tone
neurotransmitters (GABA, glycine)
Botulinum toxin (C. Cleaves SNARE proteins → lack of stimulatory
↓ muscular tone
botulinum) neurotransmitter (ACh)
Lecithinase (a phospholipase) that breaks down cell
Alpha toxin (C. perfringens) Hemolysis and myonecrosis
membrane
Streptolysin O (S. pyogenes) A protein that breaks down cell membrane Hemolysis
A superantigen that strongly binds the MHC class II
Toxic shock syndrome toxin Toxic shock syndrome (fever,
antigen-presenting cells to the CD4 on TH1 cells →
(TSST-1) (S. aureus) rash, shock)
perpetual release of IFN-γ, IL-1, IL-2, and TNF-α
A superantigen that strongly binds the MHC class II Toxic shock-like syndrome
Erythrogenic toxin A (S.
antigen-presenting cells to the CD4 on TH1 cells → (fever, rash, shock) & scarlet
pyogenes)
perpetual release of IFN-γ, IL-1, IL-2, and TNF-α fever
Table 5.1.3 - Bacterial exotoxins
14

Figure 5.1.9 - Exotoxins that inhibit protein synthesis

Figure 5.1.10 - Exotoxins that increase fluid secretion using adenylate cyclase
15

Figure 5.1.11 - Exotoxin (heat-stable toxin) that increases fluid secretion through guanylate cyclase

Figure 5.1.12 - Pertussis toxin mechanism


16

Figure 5.1.13 - Mechanism of tetanospasmin

Figure 5.1.14 - Mechanism of botulinum toxin


17

Figure 5.1.15 - Mechanism of exotoxins that disrupt cell membranes

Figure 5.1.16 - Mechanism of superantigens


18

REVIEW QUESTIONS ?
1. A bacterial organism produces a virulence factor 2. An 18-year-old female who has been “feeling
that interacts with host antibodies, allowing it to sick” is brought to the emergency department
adhere to host surfaces. Which of the following by her mother. Upon arrival, her temperature is
statements is consistent with this virulence 39.4°C (103.0°F) and her blood pressure is 78/45
factor? mmHg. Blood cultures confirm bacteremia with
S. pyogenes. Which statement is true regarding
A. It is produced by S. aureus
how the pathogen caused her septic shock?
B. It is secreted from the pathogen
C. It can lead to rheumatic fever A. The organism released a large quantity of
D. It is part of the cell surface cytokines
B. Lipid A dysregulated the immune response
• Correct answer: B C. A virulence factor triggered the release of
• The virulence factor in question is IgA TNF-α
protease, based on the following: D. An exotoxin caused direct cellular damage
• It interacts with host antibodies
• The above interaction allows the • Correct answer: C
bacteria to adhere to mucosal • This presentation is consistent with
surfaces sepsis based on the following:
• IgA protease is secreted • Bacteremia
• Choice A is incorrect because S. aureus • Fever
produces protein A, not IgA protease • Hypotension
• Choice C is incorrect because this describes • S. pyogenes is a gram positive organism
the M protein of S. pyogenes and contains many virulence factors
• Choice D is incorrect because IgA protease is which can trigger host release of
not part of the cell surface cytokines, including TNF-α
• Since the question stem states the
patient has “septic shock”, it should
be assumed the patient requires a
vasopressor
• Choice A is incorrect because this implies
that S. pyogenes directly releases cytokines
• Choice B is incorrect because lipid A is found
only on gram negative bacteria
• Choice D is incorrect because cellular
damage does not cause sepsis (host
response to virulence factors cause sepsis)
19

REVIEW QUESTIONS ?
3. A 30-year-old man with bloody diarrhea is 4. A 15-year-old male is infected with a bacterial
diagnosed with a Shigella infection. Which organism that releases an exotoxin. The role of
statement describes the mechanism through this exotoxin is to prevent the release of glycine
which Shiga toxin alters host cell activities? in the synaptic cleft of neurons. This describes
which exotoxin?
A. Inhibition of an elongation factor
B. Gαs subunit stimulation A. Alpha toxin
C. Ribonucleotide removal on rRNA B. Tetanospasmin
D. Increased levels of cAMP C. Botulinum toxin
D. Streptolysin O
• Correct answer: C E. Erythrogenic exotoxin A
• Shiga toxin and Shiga-like toxins both
inhibit protein synthesis by removing • Correct answer: B
adenine (a ribonucleotide) from the • Glycine and GABA (γ-aminobutyric acid)
rRNA of the 60S ribosomal subunit are inhibitory neurotransmitters
• Choice A is incorrect because this describes • Tetanospasmin prevents the release
the mechanism of diphtheria toxin and of these neurotransmitters, leading to
exotoxin A from Pseudomonas aeruginosa motor disinhibition
• Choices B and D are incorrect because these • Choices A and D are incorrect because these
describe the mechanism by which cholera exotoxins disrupt cell membranes and cause
toxin and the heat-labile toxin (HT) of ETEC myonecrosis and necrotizing fasciitis
cause watery diarrhea • Choice C is incorrect because botulinum
toxin prevents the release of ACh, leading to
paralysis
• Choice E is incorrect because is a
superantigen that causes shock
20

Section 3 - Bacterial Genetics

I. Genetic mutations, alterations, and exchange are significant because these activities can increase the virulence of
bacteria.

Genetic
Direction Speed Description Significance
Change
Random • Increased virulence
Vertical Slow • Genes randomly mutated over time
mutation • Antibiotic resistance
Phase and • Genes are randomly turned on or off within a
antigenic Vertical Fast bacterium → antigenic or structural changes • Increased virulence
variation (e.g. change to capsule, loss of pili)
• Naked bacterial DNA enters cytoplasm of
competent bacteria (S. pneumoniae, N. • Increased virulence
Transformation Horizontal Fast meningitidis, H. influenzae type b) • Utilized often in
• Donor DNA is incorporated into recipient laboratory research
chromosome
• F+ cell transfers plasmid to F─ cell → recipient
cell becomes F+ cell
• Plasmid integrates into chromosome → Hfr
• Increased virulence
Conjugation Horizontal Fast cell → some genes copied and transferred →
• Antibiotic resistance
recipient becomes recombinant F─ cell
• Hfr cell excises plasmid genes → novel plasmid
transferred → recipient becomes F’ cell
• Bacteriophage injects viral DNA into host →
viral DNA replicates and breaks up the host
chromosome (lytic phage)
Generalized • Viral capsids are formed and inadvertently
Horizontal Fast • Increased virulence
transduction include host DNA → bacteria is destroyed and
viral progeny released (lytic cycle)
• Viral progeny find new bacterial host and inject
newly-acquired bacterial DNA
• Bacteriophage injects viral DNA into host
→ viral DNA is incorporated into the host
chromosome and remains dormant (lysogenic • Increased virulence
phage; prophage) (S. pyogenes
• When triggered, viral DNA is excised from erythrogenic toxin,
Specialized the host chromosome, inadvertently excising Botulinum toxin,
Horizontal Fast
transduction bacterial genes → viral DNA replicates and Cholera toxin,
lyses the host cell (lytic phage) Diphtheria toxin,
• Lysogenic conversion: When the prophage Shiga toxin)
DNA produces gene products that enhance • Antibiotic resistance
the virulence of the host bacteria (exotoxins)
before entering the lytic cycle
Table 5.1.4 - Mechanisms of genetic change in bacteria
21

Figure 5.1.19 - Transformation in bacteria

Figure 5.1.17 - Random mutation in bacteria

Figure 5.1.18 - Phase variation in bacteria


22

Figure 5.1.20 - Conjugation in bacteria


23

Figure 5.1.21 - Generalized transduction (lytic cycle)


24

Figure 5.1.22 - Specialized transduction (lysogenic cycle)


II. Restriction modification system
A. Bacteria produce “DNAases” (eg, restriction
endonucleases) to degrade invading viral DNA
→ foreign DNA cannot be incorporated into
host genome
B. Bacteria methylate their own DNA at certain
regions to protect it from enzyme degradation
C. Bacteria can destroy themselves if methylation
fails
25

Figure 5.1.23 - Restriction modification


III. Transposable elements
B. Lack of selective pressures → genes (e.g.
A. DNA segments (transposons) can excise
plasmids) that confer virulence are not
themselves and integrate somewhere else
advantageous → decreased virulence
1. From plasmid integrates into bacterial
chromosome
2. From bacterial chromosome into plasmid
3. From bacterial chromosome to another part
of the chromosome
B. Create plasmids resistant to multiple drugs

I. Loss of virulence
A. Selective pressures → bacteria with
advantageous genes become more abundant →
increased virulence
26

REVIEW QUESTIONS ?
1. A colony of F─ bacteria is introduced to a • Choices A and D are incorrect because there
novel culture medium in the absence of other were no other bacterial species within the
competing bacteria. After a period of time, growth medium from which gain new genes
the bacteria is examined from two different through transformation or conjugation
areas of the culture medium. The bacteria • Choice B is incorrect because there is no
from one area (A) has produced a slime evidence of bacteriophages within the
layer of polysaccharides that was not initially medium
present. Bacteria from the second area (B) • Choice E is incorrect because random
has not produced this slime layer, and are less mutations that would lead to a slime layer
abundant than the bacteria in area B. The would’ve been present in the bacteria from
researchers conclude that the slime layer was the start of its life, not suddenly part way
initially produced by one bacterium in the first through its life
generation. Which of the following processes
describes how bacteria in area A began
producing the slime layer?

A. Transformation
B. Specialized transduction
C. Phase variation
D. Conjugation
E. Random mutation

• Correct answer: C
• The question stem describes phase
variation:
• Within the first generation, a single
bacterium started producing a slime
layer which it had not produced
earlier in its life → the change
was sudden, not gradual over
generations → genes must have
been turned on
27

Section 4 - Prion Diseases

1. Prying: prion diseases 7. Taxi: causes ataxia


2. Sponge character holding a brain: spongiform 8. Demon shouting: causes dementia
encephalopathy 9. Crab man run over and killed: causes death
3. Brain juices (CSF) pooling in the shape of “14-3-3”: 10. Sea-cows hanging outside butcher shop: can be
14-3-3 proteins are found in CSF of infected patients caused by “Mad cow disease” (bovine spongiform
4. Punching the fish with the scissors: prions are encephalopathy)
protease resistant 11. Squid family with insomnia and one dead: can be
5. Helical weapon knocking fish into hanging sheets: caused by fatal familial insomnia
pathologic prions convert normal prion proteins 12. A cross in a field: can be caused by Creutzfeld-Jakob
from an α-helical configuration (PrPc) to a β-pleated disease (CJD)
sheet formation (PrPsc) 13. Snail eating the deceased snail: can be acquired via
6. Fish tripping into sheets and getting other fish Kuru (tribal cannibalism)
tangled in sheets: β-pleated sheet proteins convert 14. Chained up autoclave: prions are autoclave-resistant
other proteins into more β-pleated sheets
28

REVIEW QUESTIONS ?
1. A 65-year-old male with dementia passes away.
It is reported that he had difficulty walking and
experienced frequent falls. His family requests
that an autopsy be performed in order to
provide a definitive diagnosis. A biopsy of his
brain is obtained and shown below.

Photo Credit: Jensflorian [CC BY-SA 3.0 (https://creativecommons.org/


licenses/by-sa/3.0)]
Figure 5.1.24 - Brain biopsy with vacuolations from
spongiform encephalopathy

Photo Credit: CDC (public domain) via Wikimedia Commons


• Which of the following describes the
proteins responsible for neuronal cell death
in this patient’s brain?

A. PrPc; α-helical
B. PrPc; β-sheet
C. PrPsc; α-helical
D. PrPsc; β-sheet

• Correct answer: D
• The patient has spongiform
encephalopathy based on the following:
• Ataxia
• Dementia
• Vacuolations in brain biopsy
• Pathologic prions (PrPsc) are in the
β-sheet configuration
• Choice A is incorrect because pathologic
prions are not in the α-helical configuration
and PrPC describes normal prions
• Choice B is incorrect because PrPC describes
normal prions
• Choice C is incorrect because pathologic
prions are not in the α-helical configuration
29

Section 5 - Introduction to Fungi

I. Fungi structure

Figure 5.1.25 - Fungi structure


II. Types of fungi
A. Two major types:
1. Filamentous (hyphae and mycelia)
2. Yeast

Figure 5.1.26 - Filamentous fungus


30

Photo Credit: (left) Bob Blaylock [CC BY-SA 3.0 (https://creativecommons.org/licenses/by-sa/3.0)]; (right) Microrao [Public domain]
Figure 5.1.27 - Mold (left) and yeast (right)
III. Monomorphic and dimorphic
A. Monomorphic
1. Fungi that only exist as one form (yeast or
mold)
B. Dimorphic
1. Fungi that exist in the form of both yeast
and mold
2. Environmental conditions dictate
morphological features
3. Mnemonic: “mold in the cold and yeast in Photo Credit: CDC [Public domain]

the heat” Figure 5.1.28 - Spores of Aspergillus


C. Fungal spores
1. A dormant cell designed to survive harsh
conditions
2. Structurally distinct from bacterial
endospores
3. Less durable than bacterial endospores
4. Important for fungal asexual reproduction
5. Produced by almost all fungi
31

REVIEW QUESTIONS ?
1. A 2-year-old boy has a diaper rash due to the
fungal organism known as Candida albicans.
Targeting of which of the following structures
would be most helpful in treating this patient’s
condition?
A. Mitochondria
B. Ergosterol
C. Cholesterol
D. Protein synthesis

• Correct answer: B
• This patient has a diaper rash caused by
Candida albicans
• The basis of the question is regarding
which structure is unique to fungi
• Ergosterol is present in the plasma
membrane of fungi and is a unique
molecule from human cells which
makes it an important antifungal drug
target (eg, nystatin)
• A, C, and D are wrong because these are
features of both fungi and human cells
so targeting of these structures would
most likely harm both types of cells and
ultimately be unhelpful or even harmful
32

Section 6 - Viral Structures

I. Viral structures
A. Envelopes
1. Lipid bilayer that surrounds the capsid
2. Lack of envelope = naked virus
B. Capsids
1. Protein structure that surrounds nucleic acids
2. Icosahedral
3. Helical

Figure 5.1.29 - Enveloped virus structures


II. Viral entry
A. Enveloped viruses
1. Adsorption: viral particles on lipid envelope
attach to host cell
2. Penetration: lipid membranes fuse and viral
capsid enters cell
3. Uncoating: capsid releases nucleic acids
B. Naked viruses
1. Adsorption: viral particles on capsid attach
to host cell
2. Penetration: cell brings in viral capsid
through endocytosis
Figure 5.1.30 - Naked virus structures 3. Uncoating: capsid releases nucleic acids
33

Figure 5.1.31 - Adsorption, penetration, and uncoating of naked viruses

Figure 5.1.32 - Adsorption, penetration, and uncoating of enveloped viruses


III. Viral disinfection using alcohol and chlorhexidine
A. Disinfection: inactivation of most pathogens
B. Sterilization: inactivation of all pathogens
C. Alcohol and chlorhexidine destroy lipid
membranes
D. Enveloped viruses: lipid envelope destroyed →
loss of viral particles → no adsorption → not
infectious
E. Naked viruses: viral particles intact on capsid →
adsorption intact → infectious

IV. Bacteriophages
A. A virus that infects bacteria
B. Responsible for the lytic cycle and the lysogenic
cycle
Figure 5.1.33 - Bacteriophage structure
34

REVIEW QUESTIONS ?
1. A research study is evaluating the efficacy of
disinfection using chlorhexidine. Every time
a patient is discharged, pathogens present
on counter surfaces are assessed and listed.
Chlorhexidine is then used to wipe down the
surface. Following disinfection, the researchers
find that all of the bacteria are killed and most
of the viruses are inactivated. However, some of
the viruses remain viable after this procedure.
The surviving viruses must possess which of the
following?
A. A capsid protein for adsorption
B. A bilayer made of phospholipids
C. A double-stranded genome
D. A single-stranded genome

• Correct answer: A
• Chlorhexidine works primarily by
disrupting lipid membranes (i.e.
destruction of bacteria and enveloped
viruses)
• Naked viruses have proteins on their
capsid that allow adsorption and entry
into host cells
• Choice B is incorrect because this describes
enveloped viruses
• Choices C and D are incorrect because
naked viruses may be double-stranded or
single-stranded, so this detail cannot be
determined by the information provided
35

Section 7 - Viral Genetics

I. Viruses can exchange genetic information with other viruses, leading to novel strains. Many genetic activities
alter the genome of the virus, while other activities only lead to temporary phenotypic changes (eg, phenotypic
mixing and complementation).

Genome
Genetic Activity Description
Altered
Recombination Yes • Two chromosomes cross over at homologous regions
Reassortment • Segmented viruses exchange genetic material → genetic (antigenic) shift
Yes
(genetic shift) • Example: pandemics due to influenza-related genetic shift
Random mutation
Yes • Slow, gradual changes to the viral genome
(genetic drift)
• An accidental relationship
• Virus A and virus B infect the same cell → surface proteins from virus B are used to
coat virus A
Phenotypic mixing No • Progeny: virus B surface proteins determine tropism (which cells they can infect)
and surround virus A genome → host cells infected with progeny replicate only
virus A genome → new progeny have original genotype and phenotype of virus A
• Example: HIV capsid within Rhabdovirus envelope
• A dependent relationship
• Virus A and virus B infect the same cell → virus A provides products necessary for
Complementation No virus B to infect or replicate → virus B is now infectious but continually depends on
virus A
• Example: HDV requires HBsAg for adsorption (HBV complements HDV)
Table 5.1.5 - Genetic adjustments and activities in viruses

Figure 5.1.34 - Genetic shift and genetic drift


36

Figure 5.1.35 - Phenotypic mixing

Figure 5.1.36 - HDV superinfection


37

REVIEW QUESTIONS ?
1. 1. Researchers isolate a virus (A) capable • Correct answer: B
of invading and replicating within human • Virus A genome coated with virus B
pharyngeal tissue only. Pharyngeal tissue surface proteins refers to phenotypic
infected with virus A is examined and mixing
researchers confirm the presence of a second • Virus B surface proteins will allow
virus (B). Virus B is capable of infecting the hybrid virus A progeny to enter
pharyngeal and intestinal tissues. Virus A and intestinal mucosal cells → tropism has
B have nonsegmented genomes. Viral progeny altered
from the pharyngeal tissue have the nucleic
acids of virus A with surface proteins from
virus B. Which term best describes the hybrid
progeny?
A. Genetic shift
B. Altered tropism
C. Dependence
D. Genomic change

• Choice A is incorrect because genetic shift


refers to reassortment which only occurs
with segmented viruses
• Choice C is incorrect because dependence
refers to complementation, but virus A is
capable of completing its life cycle without
the surface proteins of virus B
• Choice D is incorrect because phenotypic
mixing is a temporary change, not a
genomic change
38

Section 8 - Viral Genomes and Replication

I. Viral genomes III. Infectivity of purified viral genomes


A. RNA viruses are single-stranded (ssRNA) A. Purified nucleic acids = all but nucleic acids have
B. Reoviruses are double-stranded (dsRNA) been removed

C. DNA viruses are double-stranded (dsDNA) B. Infectious (if injected directly into host cells)
following purification
D. Parvoviruses are single-stranded (ssDNA)
1. Positive-stranded RNA viruses
II. Positive and Negative Sense RNA 2. DNA viruses (except poxviruses and HBV)
A. Positive sense RNA C. Not infectious (even if injected into host cells)
following purification
1. Resembles eukaryotic mRNA
1. Negative-stranded RNA (lacks RdRP)
2. Can be immediately translated upon entry
to the host cytoplasm 2. Poxviruses (requires its own DNA-
dependent RNA polymerase)
B. Negative sense RNA
3. HBV (requires its own DNA polymerase)
1. Must first be transcribed to a positive sense
strand
2. These viruses carry their own RNA-
dependent RNA polymerase to create the
positive sense strand (eukaryotic cells lack
RdRP)

Figure 5.1.37 - Positive-sense and negative-sense viruses


39

Figure 5.1.38 - Poxvirus cytoplasm replication

Figure 5.1.39 - HBV structure


IV. Translation and replication
A. Translation → new proteins
1. Protein products include everything needed
to make new virions (eg, capsids, viral
particles for adsorption, polymerases for
replication) Figure 5.1.40 - RNA virus replication
B. Replication → copy viral genome
1. RNA viruses replicate in the cytoplasm
2. DNA viruses replicate in the nucleus (except
for poxviruses)
40

REVIEW QUESTIONS ?
1. Newly synthesized virions of a DNA virus obtain
a lipid bilayer from the cell membrane of a
eukaryotic host cell. This virus is not known
to infect hepatic cells. Which of the following
occurred at some point during the life cycle of
this virus?
A. DNA formation from an RNA-dependent
polymerase
B. RNA formation from an RNA-dependent
polymerase
C. Lipid membrane fusion with host cells
D. Invagination of host cell membrane

• Correct answer: C
• The DNA virus in question is enveloped
based on the fact that it obtained a lipid
Figure 5.1.41 - DNA virus replication bilayer from the host cell
• Enveloped viruses enter the host cell
V. Polymerases in viruses through fusion of their lipid membrane
with the cytoplasmic membrane of the
A. RNA-dependent RNA polymerase
host cell
1. Found only in negative sense RNA viruses • Choice A is incorrect because this describes
2. Negative strand is transcribed into a positive reverse transcriptase, a polymerase used
strand (mRNA) only by retroviruses (HIV and HTLV) and HBV
• Retroviruses are RNA (the virus in
B. RNA-dependent DNA polymerase (reverse
question is DNA)
transcriptase)
• HBV infects hepatic cells (the virus in
1. Found only in retroviruses (HTLV and HIV) as question does not infect hepatic cells)
well as HBV • Choice B is incorrect because this describes
RNA-dependent RNA polymerase, which is
carried only by negative-sense RNA viruses

• Choice D is incorrect because this describes


VI. Envelope formation endocytosis, which is how naked viruses
A. New virions will acquire their envelope from the enter host cells
plasma membrane of the host cell
B. Herpesviruses acquire their envelope from the
nuclear membrane
41

BACTERIA

Figure 5.2.1 - Bacteria overview figure


42

Section 1 - Clostridium tetani

1. “Tethered to the gates of hell”: Clostridium tetani 11. Horns: tetanus toxin (tetanospasmin) travels to the
2. Purple background: gram positive anterior horn of the spinal cord
3. Arched back: spasms of the spinal extensor muscles 12. Monster snared to the ground: the toxin cleaves
(arched back appearance) SNARE proteins
4. Green gas: produces tetanospasmin (an exotoxin 13. Gabe’s coffin: GABA is inhibited
that causes muscle contractions) 14. Gliding Gabe: glycine is inhibited
5. Dice: diazepam can be used to reduce the muscle 15. Saw: renshaw cells
spasms 16. Rusty nails on robe: rusty nail puncture wounds may
6. Missing jaw near lock of coffin: lockjaw precede clinical symptoms
7. Snails with hard shells: spore-forming 17. Clenched hand: clenched hands
8. Belly ring with dirty rope: unhygienic deliveries and 18. Syringe in the hand: infection can be prevented with
umbilical cord care can be associated with infection the tetanus vaccine
9. Mask: obligate anaerobe 19. Y-shaped immunoglobulins: human tetanus immune
10. Eerie grin: risus sardonicus (eerie grin) globulin is used to treat an acute infection
43

REVIEW QUESTIONS ?
1. A 13-year-old unvaccinated boy presents to
the emergency department due to involuntary
muscle contractions. Physical examination
reveals neck stiffness, clenched hands, and a
decreased ability to open his jaw. The organism
most likely responsible for this patient’s
symptoms produces a toxin that cleaves what
protein?

• Correct answer: SNARE proteins


• The patient is unvaccinated and is having
  involuntary muscle contractions
Photo Credit: Y tambe [GFDL (http://www.gnu.org/copyleft/fdl.html) or CC
BY-SA 3.0 (https://creativecommons.org/licenses/by-sa/3.0)], via Wikimedia
• Decreased ability to open jaw → lockjaw
Commons • Clostridium tetani produces tetanospasmin
Figure 5.2.2 - Gram stain which cleaves SNARE proteins

Photo Credit: Content Provider(s): CDC/Don Stalons [Public domain], via


Wikimedia Commons
Figure 5.2.3 - Gram-positive bacilli

Figure 5.2.4 - Tetanospasmin


44

Section 2 - Clostridium botulinum

1. Underground robot fighting: Clostridium botulinum 9. Ensnared robot in honey: exotoxin cleaves SNARE
2. Purple in the background: gram positive proteins
3. Crime lord’s mask: obligate anaerobe 10. Seed on head: toxin prevents release of
4. Can-shaped robot: improperly canned food can acetylcholine
cause infection in adults due to ingestion of 11. Open mouth: dysarthria
preformed toxin 12. Crime lord’s sunglasses: blurred vision
5. Missing leg: descending paralysis 13. Crime lord’s bowtie: botox injections can be used for
6. Weaponized honey: ingestion of spores in honey therapeutic and cosmetic reasons
can cause infant botulism 14. Fight ring with y-shaped spikes: treat with human
7. Small, floppy robot: floppy baby syndrome botulinum immunoglobulin
8. M&M’s: spore-forming
45

REVIEW QUESTIONS ?
1. A 33-year-old female presents to the physician
due to recurrent and chronic migraines. She
has tried multiple medications but nothing
seems to alleviate her symptoms. The physician
recommends treatment with a medication
known to cause focal paralysis of the injected
region. This drug most likely prevents the
release of what neurotransmitter?

• Correct answer: acetylcholine


• Medication being described is botox
• Botox is used to treat chronic migraines and
Photo Credit: CDC/Dr. George lombard [Public domain]
causes focal paralysis of the injected region
Figure 5.2.5 - Gram stain of Clostridium botulinum • Botox is diluted Clostridium botulinum toxin
so it must work by cleaving SNARE proteins
associated with the release of acetylcholine
into the synaptic cleft
46

Section 3 - Clostridium difficile

1. Difficult army training course: Clostridium difficile 13. Skull on shoulders: toxin A and toxin B disrupt
2. Purple background: gram-positive the integrity of the cytoskeleton within intestinal
3. Flowers: normal intestinal flora mucosal cells
4. Snails in the mud: spore-forming 14. Tire obstacles: toxin A and toxin B cause
5. Mud tester wearing mask: obligate anaerobe pseudomembranous colitis and diarrhea
6. Mud tester wearing mask and gloves: contact 15. Climbing chains: diagnosed using PCR (detects the
precautions including non-sterile gloves and a gown genes that code for toxin B)
are important in preventing infection 16. Mud tester collecting poop: diagnosed with a stool
7. Cleaning guy: prior antibiotic exposure (especially sample
clindamycin) increases risk of infection 17. Big army van: treat with oral vancomycin
8. Poop pumper: patients taking PPIs are at an 18. Metro for recruit transportation: treat with
increased risk of infection metronidazole
9. Drill sergeant “A”: toxin A (enterotoxin) 19. Fit guy holding axe: treat with fidaxomicin
10. Bucket of poop: toxin A causes intestinal 20. Transplanted flower: fecal microbiota transplant
inflammation and fluid secretion may be considered in recurrent cases
11. Drill sergeant “B”: toxin B (cytotoxin)
12. Drill sergeant “B” kills recruit: toxin B can induce
cellular apoptosis
47

REVIEW QUESTIONS ?
1. A 73-year-old female is brought to the
emergency department due to fevers, nausea,
vomiting, and flank pain. Physical examination
reveals costovertebral angle tenderness.
After thorough evaluation she is treated with
levofloxacin for pyelonephritis. Several days
later her symptoms begin to resolve, but she
develops watery diarrhea and abdominal
pain. Nucleic acid amplification testing of the
patient’s stool reveals the presence of the tcdB
gene which encodes for a toxin produced by a
gram-positive organism. Which of the following
is most likely true regarding the toxin?

A. It activates adenylate cyclase


B. It inactivates the 60S ribosomal subunit
Photo Credit: CDC/Dr. Holdeman, Courtesy: Public Health Image Library C. It disrupts the integrity of the cytoskeleton
Figure 5.2.6 - Gram stain of Clostridium difficile D. It cleaves SNARE proteins

• Correct answer: C
• This patient has a C. diff infection
• The patient was treated with antibiotics
(levofloxacin) and then developed
watery diarrhea
• Antibiotic exposure (not just
clindamycin) is a risk factor for C. diff
infections
• Nucleic acid amplification testing
revealed the presence of a gene that
encodes for a toxin produced by a gram-
positive organism
• Toxin A and toxin B produced by C.
diff both disrupt the integrity of the
cytoskeleton
• A is incorrect because this is describing the
Photo Credit: User: Samir [CC BY 3.0 (https://creativecommons.org/licenses/
mechanism of other microorganisms (eg,
by/3.0)] Vibrio cholerae, enterotoxigenic E. coli, and
Figure 5.2.7 - Pseudomembranous colitis Bacillus anthracis), but these are unlikely
to suddenly cause symptoms in a patient
recently treated with an antibiotic.
• B is describing the mechanism of
the toxin produced by Shigella and
enterohemorrhagic E. coli
• D is describing the mechanism of the
toxin produced by Clostridium tetani
and Clostridium botulinum but these are
associated with muscle spasms or paralysis
and not diarrhea.
48

Section 4 - Clostridium perfringens

1. Clown with perfume: Clostridium perfringens 8. Smashed pumpkin: degrades tissue and cell
2. Purple background: gram positive membranes (hemolysis)
3. Rice Krispie Treats: crepitus 9. Pumpkin seeds: spore-forming
4. Man about to vomit: heat-labile enterotoxin causes 10. Crow: causes myonecrosis
food poisoning 11. Gang of kids: causes gas gangrene
5. Pot full of dirt: commonly found in soil 12. Kids wearing masks: obligate anaerobe
6. Alpha house sign: produces α toxin 13. Murderer killing two people with two streaks of
7. Dum dum suckers stuck in pumpkin: the α toxin blood: “double zone” of hemolysis on blood agar
(lecithinase) is a phospholipase 14. Janitor cleaning mess: treat with clindamycin
49

REVIEW QUESTIONS ?
1. A 27-year-old female presents to the emergency
department due to a wound from a garden rake
that penetrated her leg. Over the next several
hours she develops intense pain and edema
over the injured leg. Crepitus is present on
physical examination. The organism most likely
responsible for this patient’s condition produces
a toxin with what mechanism of action?

Photo Credit: Content Provider(s): CDC/Don Stalons [Public domain], via • Correct answer: degrades the cell
Wikimedia Commons membrane
Figure 5.2.8 - Gram stain of Clostridium perfringens • This patient sustained an injury from a
dirty garden rake (wounds contaminated
with dirt increase the risk of infection by
Clostridium perfringens)
• Crepitus is highly suggestive of Clostridium
perfringens
• Clostridium perfringens produces alpha
toxin which is a phospholipase that
degrades cell membranes

Photo Credit: Engelbert Schröpfer, Stephan Rauthe and Thomas Meyer. [CC BY
2.0 (https://creativecommons.org/licenses/by/2.0)], via Wikimedia Commons
Figure 5.2.9 - Gas gangrene
50

Section 5 - Bacillus anthracis

1. Giant ant hill: Bacillus anthracis 10. Sign with skull: lethal factor
2. Purple tents: gram positive 11. Ant “mates" glued together: polypeptide capsule
3. Mounds extending away from ant hill: “Medusa (contains D-glutamate)
head” 12. Coughing up red M&Ms: pulmonary anthrax causes
4. Wool coat and boots: increased risk in those who pulmonary hemorrhage
handle wool (woolsorter’s disease) 13. M&Ms near mediastinum: pulmonary anthrax
5. Girl holding inhaler: grows well under aerobic causes mediastinitis with widened mediastinum on
conditions CXR
6. M&Ms on ground: spore-forming 14. Black ant biting hand: ulcer with black eschar seen
7. Edamame: edema factor (exotoxin) in cutaneous anthrax
8. Tents: edema factor mimics adenylate cyclase 15. Flower staff: fluoroquinolones are an effective
(increases cAMP) treatment
9. Backpack protecting girl from ants: protective
antigen
51

REVIEW QUESTIONS ?
1. A 37-year-old male is admitted to the hospital
due to 3 days of fever, shortness of breath, and
a recent episode of hemoptysis. He works as a
shepherd during the summers, has no smoking
history, and denies any recent travel. A sputum
gram stain reveals gram-positive rods. Which of
the following is most likely true regarding the
microorganism?
A. The polysaccharide capsule is
antiphagocytic
Photo Credit: https://commons.wikimedia.org/wiki/File:Bacillus_anthracis_
Gram.jpg B. One component of the exotoxin complex
Figure 5.2.10 - Gram stain of Bacillus anthracis results in increased levels of cAMP
C. The toxin inactivates elongation factor (EF-
2)
D. It is an obligate anaerobe

• Correct answer: B
• This patient has pulmonary anthrax
• Hemoptysis, shepherd work, and a gram
stain showing gram-positive rods
• Bacillus anthracis produces edema
factor which mimics the effects of
adenylate cyclase → ↑ cAMP
• A is incorrect because while capsules are
antiphagocytic, bacillus anthracis has a
polypeptide capsule - not a polysaccharide
capsule
Photo Credit: Content Provider(s): CDC [Public domain], via Wikimedia • C is incorrect as this is describing the
Commons
Diphtheria toxin (EF-2 is associated with
Figure 5.2.11 - Cutaneous anthrax protein synthesis which is not directly
affected by bacillus anthracis)
• D is incorrect because bacillus anthracis is
an aerobe - not an anaerobe
52

Section 6 - Bacillus cereus

1. Bowl of rice: Bacillus cereus 9. Rice cereal: emetic type produces cereulide toxin
2. Purple shirt: gram positive 10. Vomiting: emetic type causes vomiting
3. Rod-shaped walking stick: bacilli 11. Five fingers: emetic type occurs 1-5 hours after
4. Hemp seeds: spore forming ingestion of toxin
5. Cooking rice and hemp seeds over fire: spores can 12. Guy sitting in puddle of water holding abdomen:
withstand being cooked in rice diarrheal type results in watery diarrhea and
6. Toxic green smoke: enterotoxin formation gastrointestinal pain
7. “EMT” badge: emetic type 13. “I voted” sticker: diarrheal type occurs 8-18 hours
8. Eating rice: emetic type is associated with rice and after ingestion of toxin
pasta
53

REVIEW QUESTIONS ?
1. An 8-year-old boy presents to the emergency
department due to 6 hours of nausea, vomiting,
and abdominal pain. He reports eating pasta
at an Italian restaurant earlier today. He
denies diarrhea, bloody stool, or recent travel.
Physical examination reveals diffuse abdominal
tenderness to palpation. Which of the following
is most likely true regarding this patient’s
condition?

A. It is caused by an ingested pathogen that is


Photo Credit: Y tambe [CC BY-SA 3.0 (https://creativecommons.org/licenses/ actively producing a toxin
by-sa/3.0/)], via Wikimedia Commons B. It is due to a fecalith obstruction
Figure 5.2.12 - Gram stain of Bacillus cereus C. It is associated with Henoch-Schӧnlein
purpura
D. Microscopy will reveal noncaseating
granulomas
E. A preformed toxin is responsible

• Correct answer: E
• This patient has Bacillus cereus food
poisoning
• Acute onset nausea, vomiting, and
abdominal pain after eating pasta
• The spores can withstand being cooked
and once the pasta is left at room
temperature the spores germinate
• A preformed toxin is produced that can
rapidly cause symptoms when ingested
• A is incorrect because organisms that
actively produce toxins after ingestion
usually don’t cause symptoms until 24 hours
after the ingestion of the organism.
• B is incorrect because this is describing
appendicitis.
• C is describing intussusception which is
classically associated with red “currant jelly”
stools
• D is describing Crohn disease. This typically
presents diarrhea as well as many other
manifestations such as malabsorption
problems, perianal disease, and
dermatologic abnormalities
54

Section 7 - Listeria monocytogenes

1. Monocle: Listeria monocytogenes 12. Ice and cold water: grows well at cool temperatures
2. Purple colors: gram positive (4-10 degrees C)
3. Rod-shaped bat: bacillus 13. Candle: a TORCHES organism
4. Narrow river and bay: narrow zone of β-hemolysis 14. Sick pregnant woman: septicemia in pregnant
5. Cat: catalase positive women
6. Tumbleweed: tumbling motility in broth 15. Vomit: self limited gastroenteritis in healthy
7. Whip cracking tail: forms “rocket tails” via actin individuals
polymerization 16. Vomit on pregnant belly: amnionitis in pregnant
8. Man with gun inside of house: rocket tails facilitate women can lead to spontaneous abortion
intracellular movement and cell-to-cell spread 17. Grandma: granulomatosis infantiseptica
across cell membranes 18. Baby wearing hat: neonatal meningitis
9. Asthma inhaler: aerobe 19. Man on stretcher wearing hat: meningitis in
10. Building with no roof: facultative intracellular immunocompromised
11. People drinking milk and eating cheese and deli 20. Old time amp: an effective treatment is ampicillin
meats: acquired by ingestion of cold deli meats and
unpasteurized dairy products
55

REVIEW QUESTIONS ?
1. A 23-year-old pregnant female is brought to
the emergency department due to nausea,
vomiting, and diarrhea. Several hours later she
spontaneously delivers a preterm infant who
has meningitis. Upon further investigation,
the mother endorses regularly drinking
unpasteurized milk throughout her pregnancy.
Blood cultures from the mother and child reveal
gram-positive rods that exhibit tumbling motility
in broth. Which of the following would be most
Photo Credit: CDC/Bobby Strong, Courtesy: Public Health Image Library useful in eliminating the causal organism?
Figure 5.2.13 - Gram stain of gram-positive rods
A. Cytotoxic T cell activity
B. Antibody secretion
C. Complement activation
D. Ceftriaxone

• Correct answer: A
• This mother and child have infections
caused by Listeria monocytogenes
• Listeria causes gastroenteritis in healthy
individuals (eg, nausea, vomiting, and
diarrhea) and meningitis in neonates
• The mother has a history of drinking
Photo Credit: By Y tambe (Y tambe's file) [GFDL (https://www.gnu.org/ unpasteurized milk which is a risk factor
copylet/fdl.html), CC-BY-SA-3.0 (https://creativecommons.org/licenses/by-
sa/3.0/) or CC BY-SA 2.5 (https://creativecommons.org/licenses/by-sa/2.5/), for Listeria
via Wikimedia Commons • Listeria is a gram-positive rod that
Figure 5.2.14 - Hemolysis on 5% sheep blood agar exhibits tumbling motility in broth
• Listeria is a facultative intracellular
organism which means that it
is protected from antibodies,
complement, and other circulating
immune factors
• Elimination of the pathogen is reliant
upon the activity of cytotoxic T cells
Photo Credit: Philippinjl at French Wikipedia. [CC BY-SA 2.0 fr (https://
creativecommons.org/licenses/by-sa/2.0/fr/deed.en), via Wikimedia
which trigger cell lysis
Commons • B and C are both wrong because these are
Figure 5.2.15 - Catalase test largely ineffective against an intracellular
pathogen
• D is incorrect because Listeria is resistant
to third generation cephalosporins such as
ceftriaxone
56

Section 8 - Corynebacterium diphtheriae

1. Haunted corn maze: Corynebacterium diphtheriae 10. Agency guy with extendable baton: the A subunit
2. Purple background: gram positive inhibits elongation factor 2 (EF-2)
3. Child with lice: must first be lysogenized by a 11. Heat lamp: fever
bacteriophage before it produces a virulent exotoxin 12. Tires: pseudomembrane formation
4. Woman using asthma inhaler: aerobic 13. Sausage-link rope: neurotoxic
5. Woman using the telephone: tellurite agar 14. FBI car: cardiotoxic (myocarditis, heart block, and
6. Alien: aniline dyes used to stain metachromatic arrhythmias)
granules 15. Elk: Elek test
7. Mist by the corn maze: transmitted through 16. Laughing guy: Löffler’s medium
aerosolized droplets 17. Syringe to pop tires: toxoid vaccine can prevent
8. Tie around neck: pharyngitis diphtheria
9. Bureau guy arresting business owner: the B subunit 18. Y-shaped suspension tools: preformed antibody
binds to host cell receptors and induces endocytosis can be administered to help prevent systemic
of the toxin complications
57

REVIEW QUESTIONS ?
1. A 12-year-old boy who recently moved to
the United States from Nepal is brought to
the emergency department due to fever,
pharyngitis, and heart palpitations. His
temperature is 38.5°C but other vital signs are
normal. Physical examination is significant for
a gray exudate on the posterior pharynx. A
swab of the exudate is obtained. On microscopy
there are gram-positive rods with granules that
Photo Credit: Content Provider(s): [Public domain], via Wikimedia Commons stain deeply with aniline dyes. He is admitted
Figure 5.2.16 - Gram stain of Corynebacterium to the hospital. Which of the following should
diphtheriae be done immediately to prevent additional
complications?
A. Careful monitoring of neurological status
B. Intubation
C. Serial electrocardiograms
D. Preformed antitoxin administration
E. Vaccine administration

• Correct answer: D
• This patient has a Corynebacterium
diphtheriae infection
• Possible lack of immunization (eg,
recently moved to the US from Nepal)
• Gray exudate on the posterior pharynx
• Gram-positive rods with granules that
stain deeply with aniline dyes
Photo Credit: Nethan Hussain [CC BY-SA 3.0 (http://creative commons.org/
licenses/by-sa/3.0)], from Wikimedia Commons • The antitoxin contains antibodies that
Figure 5.2.17 - Cystine-tellurite agar inactivate the diphtheria toxin (only
useful before the toxin enters the cell so
it must be administered immediately)
• A is incorrect because administration of
preformed antitoxin is more important (any
delay may allow more of the toxin to enter
the cells)
• B is unnecessary at this point because the
boy’s respiratory rate is normal and he
doesn’t appear to be in any respiratory
distress.
• C is a good idea because diphtheria
is cardiotoxic but just like choice A -
administration of the antitoxin is more
important.
• E should also be done but the immune
response to vaccination is slow so the
Photo Credit: User:Dileepunnikri [CC BY-SA 3.0 (https://creativecommons.
antitoxin should be given first in someone
org/licenses/by-sa/3.0)], from Wikimedia Commons who is acutely ill
Figure 5.2.18 - Pseudomembranous pharyngitis
58

Section 9 - Viridans streptococci

1. Venomous villain: Viridans streptococci 10. Spider webs adhering to buildings: produces
2. Purple background: gram positive dextrans which adhere to areas of endothelial
3. Wide open mouth and tongue out: normal trauma
oropharyngeal flora 11. Cars: subacute bacterial endocarditis
4. Mutant being: Streptococcus mutans 12. Teeth (dentistry): endocarditis after dental
5. Mighty warrior: Streptococcus mitis procedures in patients with pre-existing valvular
6. Teeth: Streptococcus mitis and Streptococcus lesions
mutans cause dental caries 13. Splinter in the foot: splinter hemorrhages
7. Spider warrior: Streptococcus sanguinis 14. Green soda with ice cubes: bile insoluble
8. Alfalfa plants: α-hemolytic 15. Pennies: treatment for endocarditis is penicillin G
9. Chin resisting web: optochin resistant 16. Trident weapon: ceftriaxone
59

REVIEW QUESTIONS ?
1. A 29-year-old female presents to the physician
due to intermittent fevers for the past 4
days. She states that she underwent a dental
procedure 3 weeks ago, but did not receive
prophylactic antibiotics. She has a history of
a pre-existing valvular lesion which occurred
several years ago. Examination of her fingers
reveals dark lesions that run vertically
underneath the nails beds. How will the causal
organism most likely be affected by optochin?
Photo Credit: Y tambe [GFDL (http://www.gnu.org/copyleft/fdl.html), CC-BY-
SA-3.0 (http://creativecommons.org/licenses/by-sa/3.0/) or CC BY0SA 2.5
(https://creativecommons.org/licenses/by-sa/2.5)], via Wikimedia Commons
• Correct answer: unaffected by optochin
• Intermittent fevers and dark lesions that run
Figure 5.2.19 - Gram stain of Streptococci
vertically underneath the nail beds (splinter
hemorrhages) are suggestive of bacterial
endocarditis
• A history of a pre-existing valvular
lesion and a recent dental procedure
are suggestive of an infection caused by
Viridans group streptococci
• Viridans group streptococci are optochin
resistant

Photo Credit: CDC: Dr. JJ Farmer (PHIL #3031), 1978. [Public domain], via
Wikimedia Commons
Figure 5.2.20 - Microbial resistance and sensitivity
to various compounds

Photo Credit: Splarka [Public domain], from Wikimedia Commons


Figure 5.2.21 - Splinter hemorrhages
60

Section 10 - Streptococcus pneumoniae

1. New mower: Streptococcus pneumoniae 10. Syringe on the sack: the vaccine is directed against
2. Mower is purple: gram positive the polysaccharide capsule
3. Alfalfa plant: α-hemolytic 11. Spilled green apple juice with melted ice cubes: bile
4. Hero that is sensitive about his chin: optochin soluble
sensitive 12. Guy cleaning juice with mop: MOPS (meningitis,
5. Lances on mower: lancet-shaped diplococci otitis media, pneumonia, and sinusitis)
6. Tridents: treat with ceftriaxone 13. Guy cutting down vegetation with a sickle: sepsis in
7. Scarecrow: treat with macrolides sickle cell disease and asplenic patients
8. Quails: quellung reaction 14. Scissors: IgA protease
9. Sacks of apples: polysaccharide capsule
61

REVIEW QUESTIONS ?
1. A 67-year-old female presents to the emergency
department due to sudden onset nausea,
vomiting, and confusion. Physical examination is
significant for a fever of 38.6°C, nuchal rigidity,
and altered mental status. Empiric treatment
is started and a lumbar puncture is performed.
CSF gram stain reveals gram-positive, lancet-
shaped diplococci. Administration of a vaccine
directed against what bacterial structure may
have been helpful in preventing infection?

• Correct answer: polysaccharide capsule


• A fever, nuchal rigidity, and altered mental
status → meningitis
• Streptococcus pneumoniae is the most
common cause of bacterial meningitis in
Photo Credit: Content Provider(s): CDC [Public domain], via Wikimedia adults
Commons
• A CSF gram stain reveals gram-positive,
Figure 5.2.22 - Quellung reaction lancet-shaped diplococci which confirms the
diagnosis of Streptococcus pneumoniae
• The vaccine targets the polysaccharide
capsule

Photo Credit: B. Welleschick [GFDL (http://www.gnu.org/copyleft/fdl.html),


CC-BY-SA-3.0 (http://creativecommons.org/licenses/by-sa/3.0/) or CC BY-SA
2.5 (https:///creativecommons.org/licenses/by-sa-2.5)], from Wikimedia
Commons
Figure 5.2.23 - Otitis media
62

Section 11 - Streptococcus pyogenes

1. Pie: Streptococcus pyogenes 19. “S” is for strawberry: streptolysin S


2. Purple floor: gram positive 20. Guy with a fever and towel on his head: rheumatic
3. House on the bay: β-hemolytic fever
4. “Put your raincoat”: PYR positive 21. “Welcome to the JONES’ sign”: JONES
5. Covered pie: hyaluronic acid capsule 22. Donut jelly flying to guy with a fever:
6. Burned hand: skin infections (cellulitis) antistreptolysin O (ASO) titers can be used to
7. Honey on the pie: impetigo diagnose rheumatic fever
8. Red mitten: erysipelas 23. Maid: M protein is a virulence factor
9. Superhero cape: pyrogenic exotoxin (a 24. Maid quieting down noisy kids: M protein inhibits
superantigen) causes toxic shock-like syndrome phagocytosis
10. Shocked baby: toxic shock-like syndrome causes 25. Maid caring for guy with fever: M protein can give
shock rise to rheumatic fever
11. Sweating baby: toxic shock-like syndrome causes 26. Toy cars by maid: M protein is associated with
fever carditis
12. Red rash on baby’s belly: toxic shock-like syndrome 27. Tangled up cord in shape of glomerulus:
causes a rash poststreptococcal glomerulonephritis
13. Woman wearing scarf: pyrogenic exotoxin causes 28. Broken bass guitar: bacitracin sensitive
scarlet fever 29. Stressed out guy pulling on tie wrapped around
14. Crumbs that resemble sand: scarlet fever causes a neck: pharyngitis
sandpaper-like rash 30. Pennies on the poker table: penicillin G used to treat
15. Scarf around neck: scarlet fever causes pharyngitis pharyngitis
16. Munching on a strawberry: strawberry tongue 31. Sapphires on the poker table: cephalexin used to
17. Burnt pie: necrotizing fasciitis treat skin infections (impetigo)
18. O-shaped donuts that are filled with red jelly:
streptolysin O is a toxin that degrades cell
membranes and lyses RBCs
63

Photo Credit: RafaelLopez at the English language Wikipedia[CC BY-SA 3.0 Photo Credit: Evanherk at Dutch Wikipedia [CC BY-SA 3.0 (http://
(http://creativecommons.org/licenses/by-sa/3.0/)] creativecommons.org/licenses/by-sa/3.0/)]

Figure 5.2.24 - Cellulitis Figure 5.2.25 - Impetigo


64

Photo Credit: CDC/Dr. Thomas F. Sellers/Emory University [Public domain] Photo Credit: Afag Azizova [CC BY-SA 3.0 (http://creativecommons.org/
licenses/by-sa/3.0/)]
Figure 5.2.26 - Erysipelas
Figure 5.2.28 - Strawberry tongue

Photo Credit: https://en.wikipedia.org/wiki/File:Pos_strep.JPG#file


Photo Credit: https://en.wikipedia.org/wiki/File:Scarlet_Fever.jpg#file
Figure 5.2.27 - Sandpaper-like rash Figure 5.2.29 - GAS pharyngitis
65

REVIEW QUESTIONS ?
1. A 12-year-old boy comes to the office due to 2
days of a sore throat. A rapid antigen detection
test is positive for group A Streptococcus (GAS).
The physician immediately prescribes penicillin
due to concerns about a complication that
would occur if the infection was left untreated.
The complication occurs due to a virulence
factor produced by the organism which
subsequently triggers an immune response.
What is the most likely function of the virulence
factor described?

• Correct answer: resists phagocytosis


• The patient has group A strep (GAS) and
is prescribed penicillin due to concerns of
developing rheumatic fever
• Rheumatic fever is caused by M-protein
which is a virulence factor that triggers an
immune response and causes the host to
produce antibodies that target bacterial
antigens
• The bacterial antigens may be similar to
antigens present in the heart which means
that host antibodies may accidently target
the heart
• Additionally, M-protein resists phagocytosis
66

Section 12 - Streptococcus agalactiae

1. Glacier: Streptococcus agalactiae 10. Hat and headband near pregnant lady: meningitis in
2. Bay: β-hemolytic neonates
3. Purple tents: gram positive 11. Pregnant lady: screen pregnant women at 35-37
4. Camping: CAMP factor weeks gestation
5. Sleep sack: polysaccharide capsule 12. Pennies: pregnant women colonized with GBS
6. Bass guitar with chains: bacitracin resistant receive intrapartum penicillin prophylaxis
7. Hippies: positive hippurate test 13. Amp: pregnant women may also be treated with
8. Pregnant lady coughing: pneumonia in neonates ampicillin
9. Guy sipping hot chocolate near pregnant lady:
sepsis in neonates
67

REVIEW QUESTIONS ?
1. A 27-year-old female presents to the physician
for a follow-up visit at 37 weeks gestation.
The physician recommends vaginal and
rectal cultures to screen for colonization of a
potentially harmful microorganism transmitted
to the neonate during childbirth. The organism
of interest most likely demonstrates what type
of hemolysis?

• Correct answer: β-hemolysis


• GBS may colonize the vagina and rectum
so vaginal and rectal cultures are typically
obtained at 37 weeks gestation
• GBS can be transmitted to the neonate
Photo Credit: By Clueiridium [CC0], from Wikimedia Commons during childbirth
Figure 5.2.30 - CAMP test • GBS exhibits β-hemolysis on sheep blood
agar
68

Section 13 - Streptococcus gallolyticus

1. Galileo: Streptococcus gallolyticus 6. Bloody wound: bacteremia


2. Purple colors: gram positive 7. Cart: endocarditis
3. Flowers near brown river: normal colonic flora 8. Cancer hope ribbon near muddy river: associated
4. Green gamma ray: γ-hemolytic with colon cancer
5. Green reflection in river: grows in bile
69

REVIEW QUESTIONS ?
1. A 33-year-old male with a history of IV drug • Streptococcus gallolyticus can cause
use comes to the office due to recent changes endocarditis and is associated with colon
in his fingernail. Examination of the fingernail cancer
is shown below. Physical examination is also • A is false - Streptococcus gallolyticus is
significant for a holosystolic high-pitched catalase-negative (Staphylococcus aureus  is
murmur that is best heard near the left sternal catalase-positive and can cause endocarditis
border. Blood cultures are positive for an but does not grow in the presence of bile)
organism that grows well in bile but fails to • B is false - Streptococcus gallolyticus is a
grow in the presence of 6.5% NaCl. Which of gram-positive coccus
the following is true regarding the organism • D is false - it’s not typically associated with
responsible for this patient’s condition? abdominal wound infections (this is true of
Enterococcus)
• E is false - Streptococcus gallolyticus exhibits
γ-hemolysis on blood agar

Photo Credit: Splarka [Public domain], from Wikimedia Commons

A. It is catalase positive
B. It is a gram-positive rod
C. It is associated with colon cancer
D. It is associated with abdominal wound
infections
E. It exhibits α-hemolysis on blood agar

• Correct answer: C
• The patient has endocarditis
• IV drug use
• Splinter hemorrhages (shown in the
image)
• Cardiac murmur
• Blood cultures reveal an organism that
grows well in bile but not in the presence
of 6.5% NaCl → organism must be
Streptococcus gallolyticus (Enterococci grow
in bile and 6.5% NaCl)
70

Section 14 - Enterococci

1. “Enter” sign: Enterococcus 10. Car: endocarditis


2. Purple colors: gram positive 11. Charging attendees a fee: Enterococcus faecium and
3. Flowers near sewer: normal intestinal flora Enterococcus faecalis
4. “Put Your Raincoat”: PYR positive 12. Banana trees: biliary tree infections
5. Salt shaker: grows in 6.5% NaCl 13. Amp: treat with ampicillin
6. Green gamma rays: γ-hemolytic 14. Line tape: treat with linezolid
7. Green soda near growing flowers: grows in bile 15. Resistant bouncer guys near van: vancomycin
8. Punched in the belly: abdominal wound infections resistant
9. Spilled fluid on crotch: urinary tract infections
71

REVIEW QUESTIONS ?
1. A 72-year-old male is admitted to the hospital
for community acquired pneumonia. A urinary
catheter is placed upon admission because the
patient is unable to ambulate to the bathroom.
After two days of appropriate treatment his
symptoms begin to resolve. However, on the
third day of admission he begins to experience
dysuria, frequency, and urgency. A urine culture
grows gram-positive cocci in pairs and chains.
Antibiotic sensitivity testing reveals that the
Photo Credit: Content Provider(s): CDC/Dr. Richard Facklam [Public domain] organism is resistant to vancomycin. The causal
Figure 5.2.31 - Gram stain of Enterococci organism will most likely demonstrate which of
the following?
A. Beta hemolysis on blood agar
B. PYR positivity
C. Absence of growth in hypertonic (6.5%)
saline
D. Water and oxygen production in the
presence of hydrogen peroxide

• Correct answer: B
• Reasons why Enterococcus is the most likely
organism:
• UTI (dysuria, frequency, and urgency
following placement of a catheter)
• Morphology of the organism (gram-
positive cocci in pairs and chains)
• Ability to resist vancomycin
• Enterococcus is PYR positive
• A is wrong because Enterococcus is
γ-hemolytic
• C is wrong because Enterococcus grows well
in the presence of 6.5% NaCl
• D is describing the catalase test and
Enterococcus is catalase-negative
72

Section 15 - Staphylococcus aureus

1. Merlin’s staff: Staphylococcus aureus 13. Tissue packed in nose: classically associated with
2. Purple background: gram positive prolonged use of a tampon or nasal packing
3. Merlin’s cat: catalase positive 14. Guy vomiting: vomiting
4. Bay: β-hemolytic 15. Guy is getting shocked: shock
5. Tall man: ferments mannitol 16. Rash with skin falling off: desquamating rash
6. “A” on shield: protein A prevents opsonization and 17. Dragon scales: osteomyelitis
phagocytosis 18. Red knees: septic arthritis
7. Fire coming out of dragon’s nose: colonizes the 19. Guy near mosquitos is coughing due to smoke: post-
nares viral bacterial pneumonia
8. Coagulated blood: coagulase positive 20. Burns on arm: skin infections and abscesses
9. Fire scalding man’s skin: scalded skin syndrome 21. Chest of myrrh: MRSA
10. Man vomiting: rapid-onset food poisoning 22. Bricks (i.e. altered wall) on ground causing myrrh
11. Guy getting shocked: toxic shock syndrome chest to fall off of cart: MRSA have altered penicillin
12. Guy getting shocked is wearing a superhero cape: binding proteins
TSST-1 is a superantigen (causes binding of MHC 23. Caravan: vancomycin is effective against MRSA
class II and T cell receptors → ↑ cytokine release → 24. Sapphires: cephalexin for non-MRSA infections
shock) 25. Vials of drugs on cart: endocarditis in IV drug users
73

REVIEW QUESTIONS ?
1. A 27-year-old female presents to the emergency
department due to a 5-hour history of fever,
rash, and altered mental status. Her blood
pressure is 76/58 and she is tachycardic. She
is examined thoroughly and a used tampon is
removed from the vagina. A gram stain of a
nasal swab specimen reveals gram-positive cocci
Photo Credit: By Y Tambe (Y Tambe) [GFDL (http://www.gnu.org/copyleft/fdl. in clusters. The organism most likely responsible
html) or CC-BY-SA-3.0 (http://creativecommons.org/licenses/by-sa/3.0/)], via for this patient’s presentation produced the
Wikimedia Commons
systemic syndrome through which of the
Figure 5.2.32 - Gram stain of Staphylococcus
following mechanisms?
A. Overactivation of cAMP
B. Lecithinase-mediated degradation of tissue
C. Toxin B-mediated disruption of the
cytoskeleton
D. Excessive release of cytokines

• Correct answer: D
• Reasons why Staphylococcus aureus is the
most likely organism:
• The patient has toxic shock syndrome
(fever, rash, altered mental status,
hypotension, and the presence of a
used tampon in the vagina)
• A nasal swab showed gram-positive
Photo Credit: Navaho [CC BY-SA 4.0 (https://creativecommons.org/licenses/ cocci in clusters
by-sa/4.0)]
• Toxic shock syndrome is caused by toxic
Figure 5.2.33 - Mannitol fermentation test
shock syndrome toxin 1 (TSST-1)
• This is a superantigen which causes binding
of MHC class II and T-cell receptors resulting
in overactivation of T-cells → excessive
release of cytokines
• A is true of several toxins but
Staphylococcus aureus does not affect cAMP
• B is describing the alpha toxin of Clostridium
perfringens
• C is describing the toxin of Clostridium
difficile

Photo Credit: Philippinji [GFDL (http://www.gnu.org/copyleft/fdl.html) or CC-


BY-SA-3.0 (http://creativecommons.org/licenses/by-sa/3.0/)], via Wikimedia
Commons
Figure 5.2.34 - Coagulase test tube
74

Section 16 - Staphylococcus epidermidis

1. Dermatology clinic: Staphylococcus epidermidis 6. Cat: catalase positive


2. Purple background: gram positive 7. Urine: urease positive
3. Flowers: normal skin flora 8. Biofilm on the pipe: produces an adherent biofilm
4. Bison vulnerably sitting on operating table: 9. Foreign body stuck in bison: infects foreign bodies
novobiocin sensitive 10. Tools with tubes: infects IV catheters
5. Spattered bison blood: frequently contaminates 11. Car: can cause prosthetic valve endocarditis
blood cultures 12. Van: vancomycin is an effective treatment
75

Photo Credit: https://www.flickr.com/photos/36349503@N08/3368611155


Figure 5.2.35 - Urease test

REVIEW QUESTIONS ?
1. A 39-year-old male is admitted to the hospital • Staphylococcus epidermidis produces an
due to a fever of 38.7 C and a cardiac murmur. adherent biofilm that is made up of an
He recently had his aortic valve replaced due extracellular polysaccharide matrix that
to degeneration of a congenital bicuspid aortic acts as a barrier to antibiotics and the
valve. Blood cultures are obtained and grow immune system
coagulase-negative staphylococci. Which of • B is false because Staphylococcus
the following is most likely true regarding this epidermidis doesn’t produce an exotoxin
organism? • C is D are false because Staphylococcus
epidermidis is urease positive and
A. It produces an adherent biofilm
novobiocin sensitive (Staphylococcus
B. It releases an exotoxin into the bloodstream
C. It is urease negative saprophyticus is novobiocin resistant)
D. It is novobiocin resistant

• Correct answer: A
• This patient has a Staphylococcus
epidermidis infection causing prosthetic
valve endocarditis
• Coagulase-negative staphylococcus on
gram stain
• The patient has a fever, heart murmur,
recent aortic valve replacement,
and a positive blood culture →
prosthetic valve endocarditis caused by
Staphylococcus epidermidis
76

Section 17 - Staphylococcus saprophyticus

1. Sap: Staphylococcus saprophyticus 7. Flowers covered in urine: normal flora of the female
2. Purple shirt: gram positive genital tract
3. Young attractive girl near urine: UTI in sexually 8. Fury toes: nitrofurantoin is a first line agent for
active women uncomplicated UTIs
4. Bison and chain: novobiocin resistant 9. Meth crystals in sap: TMP-SMX is an effective
5. Cat: catalase positive treatment
6. Urine: urease positive
77

REVIEW QUESTIONS ?
1. A 19-year-old sexually active female presents to
the office with dysuria, urgency, and frequency.
Urine cultures reveal gram-positive cocci.
What two lab tests would be most helpful in
distinguishing the causal organism from other
members of the genus?
A. The catalase test and the urease test
B. The catalase test and the type of hemolysis
observed on blood agar
C. The type of hemolysis observed on blood
agar and the coagulase test
D. The coagulase test and the catalase test
E. The coagulase test and the novobiocin test

• Correct answer: E
• This patient has a UTI caused by
Staphylococcus saprophyticus
• Dysuria, urgency, and frequency → UTI
• Young, sexually active, and urine
cultures have revealed gram-positive
cocci → Staphylococcus saprophyticus
• Staphylococcus saprophyticus,
Staphylococcus aureus, and
Staphylococcus epidermidis belong
to the same genus and can be
distinguished from each other using the
coagulase test and the novobiocin test
• See picture below (A, B, C, & D)
78

Section 18 - Nocardia

1. Notecards: Nocardia 9. Urine: urease positive


2. Purple colors: gram positive 10. Cat: catalase positive
3. Tangled up cords: branching filamentous 11. Stretcher: predominantly occurs in
morphology immunocompromised individuals
4. Dirt: endemic in soil 12. Burned skin: cutaneous involvement
5. Inhaler: obligate aerobe 13. Cough: pneumonia
6. Microphone: mycolic acid 14. Burning red car parts on top of the building: brain
7. Acid melting door rapidly: weakly acid-fast abscesses
8. Car bomb: carbol fuchsin 15. Meth crystals: treat with TMP-SMX
79

REVIEW QUESTIONS ?
1. A 33-year-old homeless male is brought to
the emergency department due to a severe
headache and altered mental status. He has a
history of HIV and has not been compliant with
his medications. The patient also has had a
fever, night sweats, and a productive cough for
the past several weeks. Physical examination
reveals bilateral papilledema and crackles heard
throughout all lung fields. An MRI of the brain
reveals an abscess. Which of the following is
Photo Credit: CDC/Dr. Lucille K. George, Courtesy: Public Health Image Library true regarding the most likely causal organism?
Figure 5.2.36 - Gram stain of Nocardia A. It is a gram positive coccus
B. It is a gram negative rod
C. It is catalase positive
D. It is an obligate anaerobe

• Correct answer: C
• This patient has a brain abscess caused by
Nocardia
• Immunocompromised
• Noncompliant with medications
• Fever
• Night sweats
• Cough
Photo Credit: CDC/Dr. George P. Kubica [Public domain], via Wikimedia • Brain abscess
Commons • Nocardia is catalase positive
Figure 5.2.37 - Acid-fast stain of Mycobacterium • A is false because while Staphylococcus
Tuberculosis (similar to Nocardia) aureus is a more common cause of brain
abscess, the patient’s other symptoms (eg,
history of HIV, cough) make Nocardia more
likely
• B is false because Nocardia is a filamentous,
gram-positive rod
• D is false because Nocardia is an obligate
aerobe
80

Section 19 - Actinomyces israelii

1. Actor: Actinomyces israelii 7. Skull with open mouth: colonizes the mouth, is
2. Purple colors: gram positive found in dental caries, and can cause cervicofacial
3. Tangled up cords: branching filamentous disease
morphology 8. Movie director guy aspirating coffee: pulmonary
4. Mask: anaerobic actinomyces may develop following aspiration
5. Yellow sand in air: yellow “sulfur” granules 9. Devices: intrauterine device infection can result in
6. Flowers: part of the normal oral, reproductive, and PID
GI flora 10. Pennies: treat with penicillin
81

REVIEW QUESTIONS ?
1. A 42-year-old female comes to the office due to
a chronic, slowly progressive, indurated mass
on the left side of her face. Physical examination
reveals a nontender mass near the left aspect of
the mandible. An aspiration of the mass is sent
for further analysis and a gram stain is shown
below.

Photo Credit: Yale Rosen from USA [CC BY-SA 2.0 https://creativecommons.
org/licenses/by-sa/2.0)]
Figure 5.2.38 - Gram stain of Actinomyces

Photo Credit: Yale Rosen from USA [CC BY-SA 2.0 https://
creativecommons.org/licenses/by-sa/2.0)]

A. It should be urgently treated with surgery


B. It is caused by an aerobic organism
C. The causal organism is part of the normal
oral flora
D. It stains weakly acid fast

Photo Credit: Yale Rosen from USA [CC BY-SA 2.0 https://creativecommons.
• Correct answer: C
org/licenses/by-sa/2.0)] • This patient has a cervicofacial abscess
Figure 5.2.39 - Sulfur granule caused by Actinomyces israelii
• Cervicofacial abscess affecting the
mandible is likely Actinomyces israelii
which comes from normal oral flora
• The gram stain shows a gram-positive
organism with a central sulfur granule
which is characteristic of Actinomyces
israelii
• A is false because the description of her
infection wouldn’t require urgent surgery
• B is false because Actinomyces israelii is an
anaerobic species
• D is false because Actinomyces israelii
doesn’t stain weakly acid-fast (Nocardia
does)
82

Section 20 - Neisseria overview

1. Knife: Neisseria 10. Glue: ferments glucose


2. Pink colors: gram negative 11. Oompa loompas: opa-proteins
3. Two pieces of candy being dipped into the chocolate 12. “OPEN 9-5” sign: C5-C9 complement deficiency
bowl: diplococci increases susceptibility to recurrent neisseria
4. Prominent tall hat: Neisseria meningitidis infections
5. Gondola: Neisseria gonorrhoeae 13. Blue necklace: oxidase positive
6. Chocolate candy in chocolate factory: grows on 14. Thayer-Martin chocolate food van: grows in Thayer-
chocolate agar Martin agar
7. Lips: contains lipooligosaccharides (strong 15. Van: vancomycin is present in Thayer-Martin agar
endotoxins) 16. Nice Satan: nystatin is present in Thayer-Martin agar
8. Pillars and packages of various shapes and sizes: 17. Mixer: polymyxin is present in Thayer-Martin agar
displays antigenic variation of pili 18. Meth candy bar with three segments:
9. Scissors: IgA protease trimethoprim  is present in Thayer-Martin agar

Photo Credit: Content Provider(s): CDC/Dr. Norman Jacobs [Public domain], Photo Credit: CDC/Megan Mathias and J. Todd Parker [Public domain], via
via Wikimedia Commons Wikimedia Commons
Figure 5.2.40 - Gram stain of Neisseria Figure 5.2.41 - Chocolate agar
83

Photo Credit: Jeff Dahl [GFDL (http://www.gnu.org/copyleft/fdl.html) or CC BY-SA 4.0 (https://creative commons.org/licenses/by-sa/4.0)], from Wikimedia
Commons
Figure 5.2.42 - Gram negative bacterial cell wall
REVIEW QUESTIONS ?
1. A 26-year-old male presents to the office due
to urethral discharge. One of his partners was
recently diagnosed with a sexually transmitted
infection. A swab of the discharge is sent for
further laboratory analysis. The pathologist
cultures the sample on a selective medium in an
attempt to isolate the causal organism. Which of
the following compounds is likely present in the
medium?
Photo Credit: http://commons.wikimedia.org/wikiFile: Cytoxydase.
jpg#filehistory A. Cephalexin
Figure 5.2.43 - Oxidase test B. Penicillin
C. Azithromycin
D. Vancomycin
E. Caspofungin

• Correct answer: D
• The patient presentation is suggestive of a
Neisseria gonorrhoeae infection
• A selective medium (which isolates
a particular organism) for Neisseria
gonorrhoeae is the Thayer-Martin agar
• Thayer-Martin agar contains
vancomycin, nystatin, polymyxin, and
Photo Credit: https://commons.wikimedia.org/wiki/File:Neisseria_ trimethoprim
gonorrhoeae_01.png
• A, B, C, and E are false because they are not
Figure 5.2.44 - Chocolate agar and Thayer-Martin
present in Thayer-Martin agar
agar
84

Section 21 - Neisseria meningitidis

1. Knife and top hat: Neisseria meningitidis 14. Chimney shaped like a triangle: Waterhouse-
2. Pink colors: gram negative Friderichsen syndrome causes adrenal insufficiency
3. Picture of Neisseria overview image: don’t forget 15. Lamp next to house: Waterhouse-Friderichsen
the overview information syndrome is associated with a fever
4. Mist: transmitted through respiratory secretions 16. Coagulated jello: Waterhouse-Friderichsen
5. Sack: polysaccharide capsule syndrome is associated with DIC
6. Syringe next to ammo sacks: the vaccine contains 17. Stray wire and shock symbol: Waterhouse-
antigens from the polysaccharide capsule Friderichsen syndrome is associated with shock
7. Horse with prominent nostrils: colonizes the nares 18. Union soldier stabbing guy in the spleen: asplenic
8. Malt beer: maltose fermenter patients are at an increased risk of developing an
9. Sipping: sepsis infection
10. Red spots on horse: petechial rash 19. Pennies: treat with penicillin G
11. Hats: meningitis 20. Trident: treat with ceftriaxone
12. Black foot and toes: gangrene of the toes 21. Rifles with bayonets for close combat: close
13. Putting fire out with a bucket of water: Waterhouse- contacts are prophylactically treated with rifampin
Friderichsen syndrome
85

REVIEW QUESTIONS ?
1. A 45-year-old female is brought to the
emergency department due to fever, a petechial
rash, and altered mental status. Her past
medical history is significant for an ATV accident
in which she sustained trauma to the abdomen
that required surgery due to internal bleeding.
Her blood pressure is 72/41 and heart rate is
129/min. Gram stain from a lumbar puncture
reveals gram-negative diplococci. Impairment of
which of the following most likely contributed
to this patient’s condition?
A. Reactive oxygen species
B. Immunoglobulin production
C. Complement production
D. Microtubule formation

• Correct answer: B
• The patient has a history of abdominal
trauma with significant internal bleeding
that required surgery - this suggests she had
a splenectomy following splenic rupture
• Her current presentation is suggestive of
Waterhouse-Friderichsen syndrome from an
infection with Neisseria meningitidis
• The absence of a spleen makes infection
from encapsulated organisms more
likely (Neisseria meningitis has a
polysaccharide capsule)
• The splenic B-lymphocytes produce
a large percentage of the body’s
immunoglobulins which help with
opsonization (IgG) and removal of
encapsulated organisms
• A is false because an inability to form
reactive oxygen species makes patients
more susceptible to catalase positive
organisms - as in NADPH deficiency
• C is false because the liver is where
complement proteins are produced
• D is false because microtubule dysfunction
is associated with drug effects
86

Section 22 - Neisseria gonorrhoeae

1. Gondola: Neisseria gonorrhoeae 9. Pregnant woman outside of the gondola: increased


2. Pink colors: gram negative risk of ectopic pregnancy
3. Neisseria overview image: don’t forget the overview 10. Uterus-looking chandelier: pelvic inflammatory
information disease
4. Unique pillars: antigenic variation of pili (evades 11. Tendons about to burst: tenosynovitis
immune system and has prevented vaccine 12. Glass breaking around knees: septic arthritis
development) 13. Clam: chlamydial coinfection
5. Scarlet letter: sexually transmitted infection 14. Gnat bugs: diagnosis made with nucleic acid
6. Peeing off ledge: dysuria and penile discharge in amplification test (NAAT)
males 15. Trident: treated with ceftriaxone
7. Pee hitting eye: neonatal conjunctivitis 16. Scarecrow: treated with macrolides (for chlamydial
8. Fist: Fitz-Hugh-Curtis syndrome coinfection)
87

REVIEW QUESTIONS ?
1. A 24-year-old male presents to the clinic due
to dysuria. He states that his symptoms began
two days ago. He is sexually active with multiple
partners. Physical examination reveals a white
discharge expressed from the penis. Gram
stain of the discharge reveals gram-negative
diplococci seen in the cytoplasm of neutrophils.
Which of the following is the recommended
treatment regimen for the causal organism?
A. Doxycycline and cephalexin
B. Ceftriaxone and tigecycline
C. Trimethoprim-sulfamethoxazole
D. Ceftriaxone and azithromycin
E. Penicillin and clavulanic acid

• Correct answer: D
• The patient has urethritis caused by
Neisseria gonorrhoeae
• Dysuria
• Multiple partners
• White discharge expressed from the
penis
• Gram stain showing gram-negative
diplococci in the cytoplasm of
neutrophils
• Effective treatment involves
using both ceftriaxone and
azithromycin  (ceftriaxone is used
to cover gonococcal infection and
azithromycin is used to cover a possible
chlamydia coinfection)
• A is incorrect because doxycycline
can be used to treat the possible
Chlamydia coinfection but cephalexin is
a 1st generation cephalosporin and not
commonly used to treat STIs
• B is false because tigecycline is used
as a broad-spectrum antibiotic for
Staphylococcus aureus skin and soft tissue
infections as well as MRSA and VRE
• C and E are false because both
trimethoprim-sulfamethoxazole and
penicillin with clavulanic acid are not
used to treat Neisseria gonorrhoeae or
Chlamydia
88

Section 23 - Moraxella catarrhalis

1. Cactus: Moraxella catarrhalis 6. Colorado Police Department: COPD exacerbations


2. Red sunset: gram negative 7. Earmuffs on child: otitis media in children
3. Flowers on top of cacti: normal flora of the 8. Signs: sinusitis
respiratory tract 9. Ammo: amoxicillin is an effective treatment
4. Dusty mist: transmitted through respiratory droplets 10. Cleaver: an effective treatment is amoxicillin and
5. Inhaler: aerobe clavulanic acid
89

REVIEW QUESTIONS ?
1. A 16-month-old boy is brought to the physician
due to increased irritability, fever, and poor
feeding. Physical examination reveals an
erythematous bulging tympanic membrane.
Which of the following is the most likely causal
organism?
A. Moraxella catarrhalis
B. Streptococcus pneumoniae
C. Haemophilus influenzae
D. Staphylococcus aureus
Photo Credit: CDC/Dr. W.A. Clark, Courtesy: Public Health Image Library
Figure 5.2.45 - Gram stain of Moraxella • Correct answer: B
• Streptococcus pneumoniae is the most
common cause of otitis media in children
(approximately 50% of cases)
• Moraxella catarrhalis accounts for less than
10% of cases (A)
• Haemophilus influenzae accounts for about
45% of cases (C)
• Staphylococcus aureus is a rare cause of
otitis media in children (D)

Photo Credit: Welleschik [GFDL (http://www.gnu.org/copyleft/fdl.html),


CC-BY-SA-3.0 (http://creativecommons.org/licenses/by-sa/3.0) or CC BY-SA
2.5 (https://creativecommons/org/licenses/by-sa/2.5)], from Wikimedia
Commons
Figure 5.2.46 - Otoscopic view of otitis media
90

Section 24 - Haemophilus influenzae

1. Office: Haemophilus influenzae 13. Michael with red knees: Haemophilus influenzae
2. Pink walls: gram negative serotype b (Hib) causes septic arthritis
3. Scissors: IgA protease 14. Hood: Haemophilus influenzae serotype b (Hib)
4. Jar of chocolate: grows on chocolate agar causes meningitis
5. Staff: can be grown near Staphylococcus aureus on 15. Sickle: asplenia increases the risk of Haemophilus
blood agar influenzae serotype b (Hib)  infections
6. Star fish with five points: factor V is necessary for 16. Trident: ceftriaxone can be used to treat
the growth of Haemophilus influenzae Haemophilus influenzae serotype b (Hib) infections
7. “10 RULES OF THE OFFICE” sign: factor X is 17. Rifle on shirt: rifampin prophylaxis for close contacts
necessary for the growth of Haemophilus influenzae 18. Broken keyboard: nontypeable strains (not
8. “PLEASE RESPECT PROPERTY” sign and stapler encapsulated)
surrounded by jello: encapsulated strains have a 19. Smoke: aerosolized transmission
polyribosylribitol phosphate (PRP) polysaccharide 20. Signs: nontypeable strains of Haemophilus
capsule influenzae cause sinusitis
9. Tape dividing the room in half: there are typeable 21. Ear muffs: nontypeable strains of Haemophilus
and nontypeable strains of Haemophilus influenzae influenzae cause acute otitis media
10. Syringe: there is a vaccine for Haemophilus 22. Red eyes: nontypeable strains of Haemophilus
influenzae serotype b (Hib) influenzae cause conjunctivitis
11. Dwayne opening mouth and yelling “Michael”: 23. Ammo and a cleaver: nontypeable strains of
Haemophilus influenzae serotype b (Hib) causes Haemophilus influenzae are treated with amoxicillin
epiglottitis and clavulanate
12. Computer screen that has the “thumb sign”:
“thumb sign” seen on lateral neck x-ray
91

Photo Credit: Dimitrios Malamos [CC BY 4.0 (https://creativecommons.org/


licenses/by/4.0)]
Figure 5.2.49 - Epiglottis
Photo Credit: Bobjgalindo [GFDL (http://www.gnu.org/copyleft/fdl.html)
or C BY-SA 4.0 (https://creativecommons.org/licenses/by-sa/4.0)], from
Wikimedia Commons
Figure 5.2.47 - Gram Stain of Haemophilus
influenzae

Photo Credit: Deminorwood [CC BY-SA 4.0 (https://creativecommons.org/


licenses/by-sa/4.0)]
Figure 5.2.48 - Satellite colonies of Haemophilus
influenzae growing near Staphylococcus aureus on
blood agar
92

Photo Credit: [CC BY-SA 3.0 (https://creativecommons.org/licenses/by-


sa/3.0)]
Figure 5.2.51 - Cherry red epiglottis

Photo credit: Med Chaos [CC0]


Figure 5.2.50 - Thumb sign
93

REVIEW QUESTIONS ?
1. A researcher is studying an organism that
demonstrates poor growth on 5% sheep
blood agar. However, the same organism
grows well when incubated next to colonies of
Staphylococcus aureus. The organism described
is known to cause invasive disease through the
use of what virulence factor?
A. M protein
B. Polyribosylribitol phosphate
C. Erythrogenic exotoxin A
D. Pyrrolidonyl arylamidase

• Correct answer: B
• The organism being described is
Haemophilus influenzae
• Demonstrates poor growth on 5% sheep
blood agar but grows well when grown
next to colonies of Staphylococcus
aureus
• Staphylococcus aureus is β-hemolytic
so it lyses red blood cells and provides
nearby colonies of Haemophilus
influenzae with NAD (factor V) and
hematin (factor X)
• The encapsulated strains of
Haemophilus influenzae bacterium
have a polyribosylribitol phosphate
polysaccharide capsule
• A is incorrect because M protein is
a virulence factor associated with
Streptococcus pyogenes (β-hemolytic and
would grow well on blood agar)
• C is incorrect because the erythrogenic
exotoxin A is also associated with
Streptococcus pyogenes and results in shock
after it stimulates an overwhelming release
of cytokines
• D is false because pyrrolidonyl arylamidase
(known as PYR) is associated with
Streptococcus pyogenes and Enterococcus
and is not a virulence factor
94

Section 25 - Bordetella pertussis

1. Border: Bordetella pertussis 11. Catapult: catarrhal stage


2. Pink colors: gram negative 12. Guy coughing due to smoke: a mild cough is
3. Boards blocking entrance: Bordet-Gengou agar commonly seen during the catarrhal stage
4. Rays: Regan-Lowe medium 13. Lamp: a fever is commonly seen during the
5. G.I. Joe guy falling down with an arrow stuck in catarrhal stage
chest: the pertussis toxin disables the G subunit
i 14. Watery eyes next to onions: coryza is commonly
6. Camping tent: ribosylation of the G subunit
i seen during the catarrhal stage
results in increased levels of cAMP 15. Rocks: paroxysmal stage
7. Another camping tent: adenylate cyclase toxin 16. Dust and severe coughing: severe coughing spells
increases levels of cAMP (paroxysmal cough) are commonly seen during
8. Tray and toxic gas: tracheal cytotoxin (damages the paroxysmal stage
respiratory epithelial cells) 17. Vomiting after coughing: post tussive vomiting may
9. Rope pulling catapult: filamentous hemagglutinin be seen during the paroxysmal stage
(FHA) facilitates the attachment of the pathogen 18. Convicts: convalescent stage
to ciliated epithelial cells 19. Zipper: treat with azithromycin
10. Spherical rocks with a purple hue: causes a 20. Syringe: vaccine (DTaP and Tdap) given to prevent
lymphocytosis infection
95

REVIEW QUESTIONS ?
1. A 2-year-old unvaccinated boy is brought to
the emergency department due to difficulty
breathing. His mother states that she first
noticed his symptoms approximately two weeks
ago which included a mild cough, a runny
nose, and intermittent sneezing. Recently the
cough has become so severe that he has had
trouble breathing and has vomited several
times afterward. Physical examination reveals
episodes of severe coughing spells followed by
a forced expiratory grunt. Laboratory analysis
reveals a lymphocytosis. The most likely causal
organism produces a toxin that:
A. Inactivates the 60S ribosome
B. Increases the activity of guanylate cyclase
C. Increases the activity of adenylate cyclase
Photo Credit: CDC Public Health Image Library [Public domain] D. Inactivates elongation factor 2
Figure 5.2.52 - Gram stain of Bordetella
• Correct answer: C
• This patient has a Bordetella pertussis
infection
• Difficulty breathing
• Cough
• Coryza
• Posttussive emesis
• Lymphocytosis
• The pertussis toxin over activates
adenylate cyclase by disabling G which
i

results in increased levels of cyclic AMP


• A is false because inactivation of the 60S
ribosome is indicative of Shiga toxin and
Shiga-like toxin from Shigella and E. coli
• B is false because it is the heat-stable toxin
of E. coli that increases the activity of
guanylate cyclase
• D is false because the toxins that inactivate
Photo Credit: https://www.flickr.com/photos/54976525@N08/5905471297 elongation factor 2 are the diphtheria toxin
Figure 5.2.53 - Regan-Lowe agar of Corynebacterium diphtheriae and the
exotoxin A of Pseudomonas aeruginosa
96

Section 26 - Pasteurella multocida, Brucella spp., and Francisella tularensis

1. Bruce Wayne: Brucella 15. Flies: deer flies are reservoirs for Francisella
2. Eiffel tower: Francisella tularensis 16. Minnow fish: Francisella is treated with
3. Pastor: Pasteurella multocida aminoglycosides
4. Pink colors: gram negative 17. Pastor with blue necklace: Pasteurella is oxidase
5. Ejecting out of batmobile: Brucella species are positive
facultative intracellular organisms 18. Sack of bibles: Pasteurella has a polysaccharide
6. Rolling hills: Brucella is associated with an undulant capsule
fever 19. Cat: Pasteurella is catalase positive
7. Fresh farmer’s market booth and milk: Brucella is 20. Dog biting pastor: Pasteurella is transmitted through
associated with ingestion of unpasteurized dairy dog bites
8. Dirt bike: Brucella is treated with doxycycline 21. Blood from bite wound: Pasteurella grows on 5%
9. Rabbit: rabbits are reservoirs for Francisella blood agar
10. Tic tacs: ticks are a common reservoir for Francisella 22. Wound on arm: Pasteurella is associated with
11. Pavilion: Francisella is facultative intracellular animal bites and skin infections
12. Joker cutting granny’s arm: Francisella causes 23. Fish bones and scales: Pasteurella causes
painful skin lesions osteomyelitis
13. Hand on head as if hot and sweating: Francisella 24. Penny: Pasteurella is treated with penicillin
causes a fever
14. Granny: Francisella is associated with granuloma
formation
97

REVIEW QUESTIONS ?
1. A 32-year-old female comes to the office due
to a red swollen arm. She recently purchased
an extremely hyper dog that bit her on the arm
last night as they were playing. Immediately
after the injury she felt fine so she went to sleep
without much concern. However, when she
woke up this morning she noticed the swelling
and decided to come to the office. Physical
examination is significant for an edematous and
erythematous wound on the left arm that is
tender to palpation. Cultures of the wound grow
gram-negative coccobacilli that are catalase-
positive. Which of the following is the definitive
treatment for the most likely causal organism?
A. Daptomycin
Photo credit: CDC [Public domain]
B. Doxycycline
Figure 5.2.54 - Gram stain of gram-negative C. Gentamicin
coccobacilli D. Penicillin

• Correct answer: D
• The patient has a Pasteurella infection
• Recent dog bite
• Culture of the wound shows gram-
negative coccobacilli that are catalase-
positive
• Standard treatment for Pasteurella is
penicillin
• A is false because daptomycin is commonly
used to treat skin infections (MRSA),
bacteremia, endocarditis, and VRE
• B is false because Brucella is often treated
with doxycycline but the commonly seen
Photo Credit: Sanjay Mukhopadhyay [Public domain], via Wikimedia
symptoms of undulant fever and weight
Commons loss, as well as a history of drinking
Figure 5.2.55 - Granuloma unpasteurized milk are absent
• C is false because gentamicin is used to treat
Francisella tularensis
98

Section 27 - Salmonella spp.

1. Salmon: Salmonella 12. Lemons: acid-labile (large inoculum required)


2. Red and pink colors: gram negative 13. Guy writing: may cause Reiter syndrome (reactive
3. Flag: flagellated arthritis)
4. Fish surrounded by a net: encapsulated 14. Vomiting: causes vomiting
5. Trail of blood: spreads hematogenously 15. Bloody water: causes bloody diarrhea
6. Pink bubbles that resemble neutrophils: 16. Three guys secreting the vomit: utilizes a type III
polymorphonuclear neutrophils (PMNs) are seen in secretion system
disseminated disease 17. Toxic smoke cloud: one of the virulence factors of
7. Black cloud: hydrogen sulfide production on TSI agar Salmonella is an endotoxin
8. Meat and eggs: eggs and meat (poultry) are 18. Fish with prominent scales: causes osteomyelitis in
reservoirs patients with sickle cell disease
9. Turtle: turtles are reservoirs 19. Trident: treat sickle cell osteomyelitis with
10. Raspberry patch: invades the gastrointestinal tract ceftriaxone
through M cells (found in Peyer’s patches) 20. “DO NOT PASS” sign: do not give antibiotics to
11. Pavilion: facultative intracellular patients with GI-related Salmonella infections
99

REVIEW QUESTIONS ?
1. A 17-year-old female presents to the emergency
department with a 4-hour history of nausea,
vomiting, and diffuse abdominal pain. She
also states that she has had several loose
bowel movements and one appeared slightly
red. Physical examination is significant for
diffuse abdominal pain and a fever of 38.7°C. A
stool sample is obtained and reveals oxidase-
negative, nonlactose-fermenting organisms
that produce a black color when placed in
TSI agar. Treating the patient’s condition with
ciprofloxacin would most likely result in:
Photo Credit: Roinujus [CC BY-SA 4.0 (https://creaivecommons.org/licenses/
by-sa/4.0)] A. Rapid improvement of the patient’s
Figure 5.2.56 - Gram stain of Salmonella condition
B. Infertility
C. An increased duration of bacterial fecal
excretion
D. Hepatosplenomegaly
E. Osteomyelitis

• Correct answer: C
• The patient has gastroenteritis caused by
Salmonella
• Nausea, vomiting, and diffuse
abdominal pain → gastroenteritis
• Bloody diarrhea, oxidase-negative,
nonlactose-fermenting, and black color
on TSI agar → Salmonella
• Treatment of Salmonella gastroenteritis
with ciprofloxacin would increase
Photo credit: Witmadrid [Public domain]
the duration of the illness and an
Figure 5.2.57 - TSI agar increased duration of bacterial fecal
excretion would be seen (Salmonella
gastroenteritis is usually self-limiting)
• A is false because antibiotic treatment of
Salmonella gastroenteritis is considered
ineffective with respect to the duration of
the illness
• B and D are false because infertility and
hepatosplenomegaly are not known side
effects of ciprofloxacin (which can be toxic
to the liver but does not affect the spleen)
• E is false because osteomyelitis is a
complication of Salmonella infection
especially in patients with sickle cell disease
who have areas of infarction and necrosis in
Photo Credit: User:Plainpaper [CC BY-SA 3.0 (https://creativecommons.org/ certain bone and bone marrow locations
licenses/by-sa/3.0)]
Figure 5.2.58 - Peyer’s patches
100

Section 28 - Salmonella typhi

1. Thai restaurant: Salmonella typhi 11. Lamp: typhoid fever causes a fever
2. Pink colors: gram negative 12. Blood stains: typhoid fever causes a rash
3. Flag: flagellated characterized by rose spots on the abdomen
4. Black smoke rising from eggs: hydrogen sulfide 13. Guy sitting on toilet next to muddy brown river:
production on TSI agar typhoid fever causes abdominal pain, constipation,
5. Green smoke: one of the virulence factors of and diarrhea
Salmonella typhi is an endotoxin 14. Smelling flowers next to “LIVE FISH” sign: live
6. Hiker guy: associated with a history of traveling attenuated vaccine
7. Sleep sack: has a Vi polysaccharide capsule 15. Syringe injecting juice into fish and juice spraying on
8. Lemons: acid-labile (large) inoculum required sleep sack: the IM vaccine contains the Vi antigen
9. Balloon that resembles a monocyte: immune from the polysaccharide capsule
response is primarily monocytes 16. Flower staff: treat with fluoroquinolones
10. Banana trees: can colonize the gallbladder in a 17. Trident: treat with ceftriaxone
chronic carrier state
101

REVIEW QUESTIONS ?
1. A 27-year-old male comes to the clinic due to
fever, abdominal pain, and diarrhea. He first
noticed the symptoms one week ago after
returning from a humanitarian trip to Nepal.
Physical examination reveals a temperature of
38.5°C, an erythematous maculopapular rash on
his abdomen, and diffuse abdominal tenderness
upon palpation. Blood and stool cultures are
pending. This patient is most likely at risk of
developing:
Photo Credit: Content Provider(s): CDC/Armed Forces Institute of Pathology,
Charles N. Farmer [Public domain] A. Continued fecal bacterial shedding
Figure 5.2.59 - Rose spots B. Rapid resolution of his symptoms without
antibiotic administration
C. Worsening of the rash which may involve
the hands and palms
D. Hemolytic uremic syndrome

• Correct answer: A
• The patient has an infection caused by
Salmonella typhi
• Fever, abdominal pain, diarrhea, recent
travel history to an endemic area, and
erythematous maculopapular rash on
his abdomen
• Patients who develop Salmonella typhi
are at risk of developing a chronic
carrier state of infection where the
bacteria colonize in the gallbladder →
continued fecal shedding of the bacteria
as proliferation continues without
treatment
• B is false because the rapid resolution
of symptoms without treatment is
characteristic of nontyphoidal Salmonella
species
• C is false because of the rash of Salmonella
typhi lasts for 10-14 days generally and is
commonly distributed on the chest, back,
and abdomen
• D is false because Shigella and E. coli
produce toxins that can cause HUS and
damage the kidneys
102

Section 29 - Shigella

1. Shit: Shigella 12. Red candy on top of poop: Shiga toxin causes
2. Red sunset: gram negative damage to the GI mucosa resulting in dysentery and
3. Pooper scooper team of three: utilizes a type III bloody diarrhea
secretion system 13. Tangled up strings that resemble the glomerulus
4. Poop thrown from one person and landing on and red balloons: Shiga toxin causes hemolytic
another person: human-to-human transmission uremic syndrome
only 14. Guy writing: may cause Reiter syndrome (reactive
5. Raspberry patches: invades the gastrointestinal arthritis)
tract through M cells (found in Peyer’s patches) 15. Shit and disco ball: Shigella dysenteriae (most
6. Horse reins that resemble actin: manipulates the severe)
host’s actin filaments which allows the organism to 16. Flexing guy: Shigella flexneri (2nd most severe)
move intracellularly 17. Body builder guy: Shigella boydii (3rd most severe)
7. Hoof prints that resemble the nuclei of neutrophils: 18. Body builder’s son: Shigella sonnei (4th most
immune response is primarily neutrophils severe)
8. Stationary handcuffed guy: immotile (no flagella) 19. Amp: treated with ampicillin
9. 60 minute news crew: Shiga toxin inhibits the 60S 20. Flower staff: treated with fluoroquinolones
ribosomal subunit 21. Meth crystals: treated with TMP-SMX
10. Coal covered in blood: enterohemorrhagic E. coli 22. Trident: treated with ceftriaxone
(EHEC) has a toxin that closely resembles Shiga toxin
(Shiga-like toxin)
11. Orange juice being blocked: acid resistant (low
inoculum required)
103

Photo Credit: Paulo Henrique Orlandi Mourao [CC BY-SA 3.0 (https://
creativecommons.org/licenses/by-sa/3.0)]
Figure 5.2.63 - Schistocytes

REVIEW QUESTIONS ?
1. A 12-year-old girl is brought to the emergency
Photo Credit: CDC/Armed Forces Institute of Pathology (AFIP) department due to bloody diarrhea which
Figure 5.2.60 - Gram stain of Shigella began 2 days ago. Stool cultures grow non-
lactose fermenting gram-negative bacilli that
do not produce hydrogen sulfide when grown
on triple sugar iron agar. It is determined that
her infection is caused by an extremely invasive
organism that can cause disease with exposure
to as few as 10 organisms. The pathogenesis of
this patient’s condition is most likely caused by:
A. Hematogenous spread
B. Invasion of cells that overlie intestinal
lymphatic aggregates
C. Aerosolized inhalation
Photo Credit: Jeff16 [CC BY-SA 4.0 (https://creativecommons.org/licenses/
D. Excess proliferation of normal bacterial flora
by-sa/4.0)] E. Spore germination
Figure 5.2.61 - Actin
• Correct answer: B
• This patient has an infection caused by
Shigella
• Bloody diarrhea
• Stool cultures that grow non-lactose
fermenting gram-negative bacilli that do
not produce HS on triple sugar iron agar
• A very low inoculum is required for
Shigella to cause disease
• Shigella infects M cells which overlie the
intestinal lymphatic aggregates (Peyer’s
patches)
• A is false because Shigella spreads from cell
to cell - not hematogenously
Photo Credit: Dr. Graham Beards [CC BY-SA 3.0 (https://creativecommons. • C is false because Shigella infects after
org/licenses/by-sa/3.0)] ingestion into the GI tract
Figure 5.2.62 - Neutrophils • D is false because Shigella doesn’t form part
of the normal GI flora
• E is false because Shigella isn’t spore-
forming
104

Section 30 - E. coli

1. Coal mine: Escherichia coli (E. coli) 17. Pregnant woman and hat: serotypes of E. coli
2. Pink reflection: gram negative with the K antigen are commonly associated with
3. Bay: β-hemolytic neonatal meningitis
4. Flowers: normal flora of the colon 18. Pregnant lady coughing: serotypes of E. coli with the
5. Green machine: green metallic sheen on eosin K antigen are commonly associated with pneumonia
methylene blue (EMB) agar 19. Torch: a TORCHES infection
6. Monkey animal crackers: E. coli is a lactose 20. Lips, sack, sipping, and shock: has a
fermenter (pink color on MacConkey agar) lipopolysaccharide virulence factor that causes
7. “Speed 50 MPH” sign: fast lactose fermenter septic shock
8. Cat: catalase positive 21. Coal and blood: enterohemorrhagic E. coli (EHEC)
9. Dole whip: indole positive 22. Pool of blood: EHEC causes bloody diarrhea
10. Three guys helping carry water: utilizes a type III 23. Microphone shaped like the number one: EHEC is
secretion system the most common serotype of E. coli in the United
11. Guy with bacteria spontaneously growing over his States
abdomen: causes spontaneous bacterial peritonitis 24. Smashed cup of sorbet: EHEC does not ferment
12. Pillar pipe on fire going to outhouse: E. coli has sorbitol
virulent pili that are associated with UTIs, cystitis, 25. Hamburger and green leafy lettuce: EHEC is
and pyelonephritis transmitted via undercooked meat and raw leafy
13. Thin wires that resemble fimbriae: E. coli has vegetables
virulent fimbriae that are associated with cystitis 26. O157:H7 written on shirt: O157:H7
and pyelonephritis 27. Shit hitting 60 minutes camera: EHEC produces
14. Prostitute: causes prostatitis shiga-like toxin that inhibits the 60S ribosomal
15. Ammo near outhouse: amoxicillin can be used to subunit
treat E. coli UTIs 28. Shit, strings that resembles the glomerulus, and red
16. Sack and letter “K” on hat: K antigen is present in balloons: shiga-like toxin causes hemolytic-uremic
the polysaccharide capsule syndrome
105

29. Broken plate: shiga-like toxin causes 39. Rock on fire and rock with black spots:
thrombocytopenia enteroinvasive E. coli invades the intestinal mucosa
30. Rock smashing balloon: hemolytic-uremic syndrome and causes inflammation and necrosis
causes mechanical hemolysis resulting in anemia 40. Toxic cloud: enterotoxigenic E. coli
31. Helmet shaped like a schistocyte: shiga-like toxin 41. Travelers: enterotoxigenic E. coli causes travelers’
causes microthrombi formation --> schistocytes and diarrhea
renal failure 42. Water: enterotoxigenic E. coli causes watery
32. Rock on fire and rock with black spots: shiga-like diarrhea
toxin causes necrosis and inflammation 43. Warm and stable guy: enterotoxigenic E. coli
33. Rope pathway: enteropathogenic E. coli produces a heat-stable enterotoxin
34. Smashed plants that resemble intestinal villi: 44. Heat generator near warm and stable guy: the heat-
enteropathogenic E. coli adheres to apical surfaces stable enterotoxin increases levels of cGMP
of intestinal cells, flattens villi, and prevents 45. Emotionally-labile guy crushed by a rock:
absorption enterotoxigenic E. coli produces a heat-labile
35. Child’s clothes covered in mud: enteropathogenic E. enterotoxin
coli causes diarrhea in children 46. Camping tent near emotionally labile guy: the heat-
36. Happy child with thumbs up on shirt: labile enterotoxin increases levels of cAMP
enteropathogenic E. coli does not produce a toxin 47. Fire getting put out by water: enterotoxigenic E. coli
37. Vase and coal: enteroinvasive E. coli does not cause inflammation or invasion
38. Shit falling: the clinical manifestations of 48. Travelers with prominent collars: enterotoxigenic E.
enteroinvasive E. coli are similar to Shigella coli causes watery diarrhea that is similar to cholera

Photo Credit: Bobjgalindo [CC BY-SA 4.0(https://creativecommons.org/


licenses/by-sa/4.0)]
Figure 5.2.64 - Gram stain of E. coli Photo Credit: Medimicro [Public domain]
Figure 5.2.66 - MacConkey agar

Photo Credit: Witmadrid [Public domain]


Figure 5.2.65 - Eosin methylene blue aga
106

Photo Credit: Botolph [Public domain]


Photo Credit: Microrao [Public domain]
Figure 5.2.67 - Indole test Figure 5.2.68 - Lipopolysaccharide
107

REVIEW QUESTIONS ?
1. A 22-year-old male presents to the emergency
department due to abdominal pain and
diarrhea that started yesterday. He states that
he recently ate an undercooked hamburger
from a restaurant and is worried he may have
developed an infection. Physical examination
is significant for diffuse abdominal tenderness
and guaiac-positive bloody stools. Stool cultures
reveal a gram-negative organism that is indole-
positive and does not ferment sorbitol on
sorbitol-containing MacConkey agar. Additional
laboratory analysis will most likely reveal which
of the following?
A. Thrombocytosis
B. Elevated serum creatinine and blood urea
nitrogen
C. Fragmented platelets on a peripheral blood
smear
D. A positive Coombs test

• Correct answer: B
• The patient has an infection by
enterohemorrhagic E. coli (EHEC)
• Guaiac-positive bloody diarrhea
• Does not ferment sorbitol
• EHEC can cause HUS because the
Shiga-like toxin causes microthrombi
formation and results in schistocytes
and kidney damage
• Kidney damage results in elevated
serum creatinine and blood urea
nitrogen
• A is false because E. coli does not cause
thrombocytopenia via platelet consumption
• C is false because the platelets are not
fragmented, the RBCs are (schistocytes)
• D is false because a positive Coombs test
indicates the presence of antibodies that
are causing RBC hemolysis which is not seen
in E. coli infections
108

Section 31 - Klebsiella

1. Club: Klebsiella 10. Molotov cocktail: associated with lobar pneumonia


2. Pink sunset: gram negative in alcoholics when aspirated
3. Flowers next to dirty river: part of the normal 11. Strangled: may cause struvite kidney stones
gastrointestinal flora 12. Monkey eating liver: causes hepatic abscesses
4. Fast monkey: ferments lactose on MacConkey agar 13. Bacteria spontaneously growing on abdomen:
(fast lactose fermenter) causes spontaneous bacterial peritonitis
5. Sacks: polysaccharide capsule 14. Young attractive girl next to urine: causes urinary
6. Urine: urease positive tract infections
7. Spilled container of pills: associated with multidrug 15. Red and green cubes of jello: classically associated
resistance with red “currant jelly” sputum
8. Mucous: mucoid colonies 16. Black river: lung abscesses and liquefactive necrosis
9. Dyed beads and guy coughing: associated with lobar of lung tissue
pneumonia in diabetics when aspirated
109

REVIEW QUESTIONS ?
1. A 53-year-old homeless male with a history
of alcoholism is brought to the emergency
department due to 3 days of chest pain,
chills, and shortness of breath. He quickly
destabilizes and passes away despite aggressive
intervention. A sputum culture obtained upon
admission to the emergency department
revealed gram-negative bacilli that formed
mucoid colonies. An autopsy revealed
Photo Credit: Microrao, JJMMC, Davangere, Kamataka, India [Public domain] the presence of lung abscesses filled with
Figure 5.2.69 - Mucoid colonies liquefactive necrosis. The organism most likely
responsible for this patient’s death:
A. Slowly produces a pink color on MacConkey
agar
B. Rapidly produces a pink color on
MacConkey agar
C. Slowly produces a yellow color on
MacConkey agar
D. Rapidly produces a yellow color on
MacConkey agar

• Correct answer: B
• The patient has aspiration pneumonia due
to Klebsiella
• History of alcoholism, chest pain,
Photo Credit: Doruk Salanci [CC BY-SA 3.0 (https://creativecommons.org/
and shortness of breath → aspiration
licenses/by-sa/3.0)] pneumonia
Figure 5.2.70 - Struvite crystals • Sputum culture shows gram-negative
bacilli that form mucoid colonies and
there are lung abscesses filled with
liquefactive necrosis → Klebsiella
infection
• Klebsiella is a fast lactose fermenter on
MacConkey agar (turns pink)
• A is false because Klebsiella is a fast lactose
fermenter
• C and D are false because Klebsiella is a
lactose fermenter and produces a pink color
when plated on MacConkey agar
110

Section 32 - Proteus mirabilis

1. Proteus god: Proteus mirabilis 7. Strangled: may cause struvite kidney stones
2. Pink and red colors: gram negative 8. Horns: staghorn calculi may be seen
3. Flipper: exhibits swarming motility when plated 9. Water splashing on girl’s crotch: causes urinary tract
4. Urine: urease positive infections
5. Black smoke near eggs: hydrogen sulfide production 10. Green stench cloud rising from fish: causes a distinct
on TSI agar fishy scent
6. Stones: kidney stones 11. Ammo: treated with amoxicillin
111

REVIEW QUESTIONS ?
1. A 37-year-old male comes to the emergency
department because of a 3-hour history of
dysuria and hematuria. His temperature is
38.4°C, pulse is 112/min, and blood pressure is
158/84. Physical examination is significant for
left costovertebral angle tenderness. Urinalysis
reveals the presence of blood and a pH of 6.7
(normal: <5.3). An X-ray of the abdomen (KUB)
reveals the presence of a staghorn calculus in
the left kidney. This patient’s condition is most
likely caused by:
A. Dehydration
B. An organism that produces ammonia from
urea
C. High levels of uric acid
D. Impaired cystine reabsorption in the
proximal convoluted tubule

• Correct answer: B
• Patient has a kidney stone caused by
Proteus mirabilis
• Dysuria
Photo Credit: © Nevit Dilmen [CC BY-SA 3.0 (https://creativecommons.org/ • Hematuria
licenses/by-sa/3.0)]
• CVA tenderness
Figure 5.2.71 - Staghorn calculus • Staghorn calculus
• The urinalysis results of a pH of
6.7 indicates a struvite or calcium
phosphate kidney stone rather than uric
acid or cystine kidney stones (where a
low pH would be present)
• The staghorn calculus suggests infection
with an organism that produces
ammonia from urea - a urease positive
organism - like Proteus mirabilis
• A is false because it is referring to calcium
oxalate stones
• C is false because it is referring to uric acid
kidney stone
• D is false because it is referring to cystine
kidney stones
112

Section 33 - Pseudomonas aeruginosa

1. Painting of Mona Lisa: Pseudomonas aeruginosa 15. Grenade releasing green smoke and causing an
2. Pink color of the building: gram negative electrical fire that shocks a burglar: produces an
3. Flag: flagellated (motile) endotoxin which causes shock
4. Statue of a cat: catalase positive 16. Flames burning the burglar: causes skin infections in
5. Three holes in the wall and robbers throwing down burn victims
stolen valuables: utilizes a type III secretion system 17. Guard putting salt in the water softener is coughing
6. Stuffing valuables into a sack: polysaccharide on grenade smoke: causes pneumonia (especially in
capsule cystic fibrosis patients)
7. Woman wearing a blue necklace: oxidase positive 18. SWAT team with night-vision goggles: keratitis and
8. Panicked woman using inhaler: obligate aerobe corneal ulcers (contact lens wearers)
9. Scared woman who urinated: causes urinary tract 19. Red and black bullet wound: causes ecthyma
infections gangrenosum (especially in immunocompromised
10. Oblivious guard eating grapes: produces a grape-like patients)
fruity odor 20. Special headset covering the ears: causes otitis
11. Modern art showing blue and green paint: produces externa (swimmer’s ear)
a blue-green pigment (pyocyanin and pyoverdin) 21. Burglar with dyed beads in his hair: more common
12. Exploding tank of oxygen near the blue and green in diabetics
paint: pyoverdin and pyocyanin generate reactive 22. Decorative fountain: causes hot tub folliculitis
oxygen species 23. Guard sipping a drink: causes sepsis
13. Officer from the “Agency” wielding an elongating 24. A nurse making things worse by running toward
baton: exotoxin A inactivates elongation factor 2 an active robbery: associated with nosocomial
(EF-2) infections
14. Clothespins hanging the “Come see” sign: produces 25. Drug addict stealing medications from the
phospholipase C (a toxin that degrades cell unattended ambulance: associated with drug
membranes) addicts
113

26. Security guard with scaly bullet-proof vest: causes 29. Pine tree car freshener: treated with cefepime
osteomyelitis 30. Leaky pipe: treated with piperacillin
27. Getaway car: causes endocarditis 31. Minnows swimming in the fountain: treated with
28. Robber offering flowers: treated with aminoglycosides
fluoroquinolones 32. Tiger painting on the wall: treated with ticarcillin

Photo Credit: CDC [Public domain]


Figure 5.2.72 - Gram stain of Pseudomonas

Photo Credit: James Hellman, MD [CC BY-SA 3.0 (https://creativecommons.


org/licenses/by-sa/3.0)]
Figure 5.2.74 - Corneal abrasion

Photo Credit: Y tambe [CC BY-SA 3.0 (http://creativecommons.org/licenses/


by-sa/3.0)]
Figure 5.2.73 - Pyoverdin and pyocyanin (bluish-
green color) of Pseudomonas (left)
114

Photo Credit: James Heilman, MD [CC BY-SA 3.0 (https://creativecommons.


org/licenses/by-sa/3.0)]
Figure 5.2.76 - Hot tub folliculitis

Photo Credit: James Heilman, MD [CC BY 3.0 (https://creativecommons.org/


licenses/by/3.0)]
Figure 5.2.75 - Otitis externa
115

REVIEW QUESTIONS ?
1. A 24-year-old female is admitted to the
hospital following a house fire in which she
sustained third-degree burns over her trunk
and arms. Several days after aggressive
fluid administration, pain management, and
surgical debridement she develops a diffuse
skin infection. She is successfully treated with
cefepime and tobramycin and discharged
from the hospital. The organism most likely
responsible for this patient’s skin infection is:
A. Oxidase negative
B. Urease positive
C. Motile
D. A slow lactose fermenter

• Correct answer: C
• This patient has an infection by
Pseudomonas aeruginosa
• Burn victim
• Developed a diffuse skin infection
• Treated successfully with cefepime and
tobramycin
• Pseudomonas aeruginosa is a
flagellated motile organism
• A, B, and D are not characteristic of
Pseudomonas aeruginosa which is oxidase
positive, urease negative, and does not
ferment lactose
116

Section 34 - Yersinia pestis and enterocolitica

1. Yerba mate: Yersinia pestis 9. Sacks of food for those in need: Yersinia
2. “DO NOT ENTER” sign: Yersinia enterocolitica enterocolitica is encapsulated
3. Pink sunset: gram negative 10. Pork and milk: Yersinia enterocolitica is transmitted
4. Sacks of yerba mate: encapsulated through contaminated milk and pork
5. Outdoors but also has shelter from a large canopy: 11. Dog pooping into the sewer: Yersinia enterocolitica
facultative intracellular is transmitted through pet feces
6. Fleas, rats, and prairie dogs roaming the alley: 12. Fancy pen: Yersinia enterocolitica causes
Yersinia pestis is transmitted through rats, prairie pseudoappendicitis
dogs, and fleas 13. Writing on a notepad: Yersinia enterocolitica causes
7. Plague doctor mask: Yersinia pestis causes the Reiter syndrome
bubonic plague 14. Vomiting: Yersinia enterocolitica causes vomiting
8. Man with bleeding armpits: Yersinia pestis causes and diarrhea
buboes (erythematous, swollen, and painful lymph 15. Flower staff: Yersinia pestis and Yersinia
nodes) enterocolitica can be treated with fluoroquinolones
117

REVIEW QUESTIONS ?
1. A 27-year-old male presents to the emergency
department while on vacation in California due
to a fever, chills, and axillary pain. Additional
history reveals that while driving through
Nevada he accidentally ran over a rodent.
He was very distressed by the situation so
he spent 30 minutes burying it on the side of
the road. Physical examination reveals tender
lymphadenopathy in the left axilla. A sample
of fluid from the lymph node reveals gram-
Photo Credit: CDC [Public domain] negative bacilli. This patient’s condition is most
Figure 5.2.77 - Gram stain gram of Yersinia likely caused by:
enterocolitica
A. Coccidioides immitis
B. Yersinia pestis
C. Blastomyces dermatitidis
D. Yersinia enterocolitica
E. Cryptococcus neoformans

• Correct answer: B
• This patient has a Yersinia pestis infection
• Rodent exposure
• Tender lymphadenopathy in left axilla
(buboes)
• Gram-negative bacilli
• A is incorrect due to the lymphadenopathy
although it is found in the same U.S. region
Photo Credit: CDC [Public domain] • C is incorrect because of the location and
Figure 5.2.78 - Axillary Bubo symptoms (Yersinia enterocolitica is found
more commonly in the Eastern and Central
part of the U.S. and does not cause buboes)
• D is incorrect as it is associated with GI
symptoms and has different transmission
• E is incorrect as this is a fungal infection that
is commonly transmitted through pigeon
droppings and presents with neurological
findings
118

Section 35 - Enterobacter, Citrobacter, & Serratia marcescens

1. Ptero (Greek for having wings): Enterobacter 10. Prominent red aura surrounding Mars: Serratia
2. Mars: Serratia marcescens marcescens produces a red pigment
3. Orange (citrus fruit) pods: Citrobacter 11. Cat constellation near Mars: Serratia marcescens is
4. Pink and red atmosphere: gram negative catalase positive
5. Red tube coming off the winged ship: Enterobacter 12. Monkey in slow, damaged ship near Mars: Serratia
is associated with intravascular catheter infections marcescens is a slow lactose fermenter
6. Shields protecting the winged ship from laser 13. Attractive pilot in damaged ship losing urine fuel:
beams: Enterobacter is drug resistant Serratia marcescens causes UTIs
7. Monkey piloting a fast ship near the winged ship: 14. Two cephalopods: Serratia marcescens is treated
Enterobacter is a fast lactose fermenter with second generation cephalosporins
8. Attractive pilot in a ship that burns urine for fuel: 15. Monkeys in slow, damaged ship with orange pods:
Enterobacter causes nosocomial UTIs Citrobacter is a slow lactose fermenter
9. Two cephalopods: Enterobacter is treated with
second generation cephalosporins
119

REVIEW QUESTIONS ?
1. A 6-year-old girl is admitted to the ICU while
undergoing treatment for leukemia. An
intravascular catheter is placed to provide
access for fluid and medication administration.
However, two days later she develops erythema,
tenderness, and drainage surrounding the
access site. The catheter is removed, but the
medical team is concerned about possible
bacteremia. Blood cultures are drawn and
grow gram negative bacilli that rapidly ferment
lactose when placed on MacConkey agar. The
Photo Credit: Riraq25 [CC BY-SA 3.0 (https://creativecommons.org/licenses/ most likely causal organism:
by-sa/3.0)]
Figure 5.2.79 - Gram stain of Enterobacter A. Can be effectively treated with ceftriaxone
B. Produces a red pigment
C. May cause bloody diarrhea
D. Also causes urinary tract infections

• Correct answer: D
• This patient has an Enterobacter infection
• Erythema, tenderness, and drainage
surrounding the access site
• Gram negative bacilli that rapidly
ferment lactose on MacConkey agar
• Enterobacter causes urinary tract
infections
• A is incorrect because ceftriaxone is a third
generation cephalosporin and enterobacter
is resistant to this antibiotic
• B is incorrect because Serratia produces a
Photo Credit: https://en.wikipedia.org/wiki/File:Bloody_bread_-_Serratia_
marcescens_in_action.JPG#file red pigment but is a slow lactose fermenter
Figure 5.2.80 - Red pigment from Serratia on a • C is incorrect because enteric organisms
piece of bread such as Shigella or E. coli may cause bloody
diarrhea but are not associated with
intravascular catheter infections
120

Section 36 - Bartonella henselae

1. Hens: Bartonella henselae 9. Scalpel and shirt with circular blood stains: biopsy
2. Pink sky: gram negative will show neutrophilic inflammation
3. Starry sky: Warthin-Starry stain 10. Shattered petri dish near car: blood culture-negative
4. Farm with animals: zoonotic organism endocarditis
5. Cat scratching hen: cat scratch fever 11. Scarecrow leaning up against barn: treat with
6. Granny defending the hen: granuloma formation macrolides
7. Hairy armpits: painful axillary lymphadenopathy 12. Dirt bike: treat with doxycycline
8. Man using a $100 bill to patch up injured hen on 13. Using a rifle to shoot the cat: treat with rifampin
a red-stained stretcher: immunocompromised
patients (AIDS patients with CD4 count < 100) may
develop bacillary angiomatosis
121

REVIEW QUESTIONS ?
1. A 27-year-old male presents to the physician
due to pain in the left axilla. Physical
examination reveals an enlarged and tender
lymph node that is palpable under the left axilla.
There is also several scratch marks seen on the
posterior aspect of the left hand. When asked
about the scratches, he states that they were
caused by a new cat he recently purchased.
Which of the following is the best treatment for
this patient’s condition?
A. Azithromycin
Photo Credit: [CC BY 2.0 (https://creativecommons.org/licenses/by/2.0)], via B. Ceftriaxone
Wikimedia Commons
C. Daptomycin
Figure 5.2.81 - Warthin-Starry stain D. Metronidazole
E. Amoxicillin

• Correct answer: A
• This patient has an infection caused by
Bartonella henselae (cat scratch disease)
• Recent cat purchase
• Multiple scratches
• Lymphadenopathy in the axilla proximal
to the scratches
• Effective treatment includes using a
macrolide (eg, azithromycin)
• B-E are not used in the treatment of
Bartonella henselae infections
122

Section 37 - Legionella pneumophila

1. Roman legion: Legionella pneumophila 9. Water spilling into the sinking ship: causes watery
2. Pink sky and water: gram negative diarrhea
3. Soldier falling out of the boat and swimming to get 10. Man coughing from smoke: causes pneumonia
back in: facultative intracellular 11. Confused soldier smacked in the head: causes CNS
4. Boats surrounded by lots of water: transmitted symptoms (eg, confusion, headaches)
through water sources (eg, air conditioning systems, 12. Hot fire burning the ship: high fever (>39°C)
hot water tanks) 13. Ramming spike shaped like the Pontiac symbol:
5. Fire with smoke rising in the air: common in causes Pontiac fever
smokers and in chronic lung disease 14. Salt spilling out of the damaged ship: associated
6. Helen of Troy wearing expensive blue necklace: with hyponatremia
oxidase positive 15. Oil and gems spilling out of damaged ship:
7. Piles of silver: can be identified using the silver stain diagnosed by detecting antigens in the urine
8. Legion shields decorated like the Sistine chapel, 16. Soldier using zip line: can be treated with
black-sooted faces, and iron javelins: Legionella is azithromycin
grown on charcoal yeast extract medium buffered 17. Flowers wrapped around the mast: can be treated
with iron and cysteine with fluoroquinolones
123

REVIEW QUESTIONS ?
1. A 51-year-old male with a history of chronic
bronchitis is brought to the emergency
department by his wife due to strange behavior
for the past two days. She states that this
morning he got lost driving home from the
grocery store which is only a few blocks away
from their home. She also states that he
complained of diarrhea and a headache the
day before. Physical examination is significant
for a fever of 40.3°C (104°F). The patient is also
Photo Credit: CDC-PHIL [Public domain] mildly disoriented, but the remainder of the
Figure 5.2.82 - Gram stain of Legionella neurological exam is normal. A sputum gram
stain reveals an abundance of neutrophils but
no organisms. What additional finding will most
likely be present in this individual?
A. Gram-positive cocci colonization of the
nares
B. Antigens of the pathogen in the urine
C. Hemolysis due to IgM autoantibodies
D. Antibodies directed against hemagglutinin
and neuraminidase

• Correct answer: B
• The patient has a Legionella infection
• History of chronic bronchitis
• CNS symptoms
Photo Credit: William Cherry [Public domain] • GI symptoms
Figure 5.2.83 - Silver stain of Legionella • High fever
• Abundance of neutrophils but no
organisms seen on gram stain
• Legionella can be diagnosed by
detecting urine antigens
• A is false because it is characteristic of
Staphylococcus aureus
• C is false because it describes the
pathogenesis of Mycoplasma pneumonia
• D is false because it refers to the mechanism
of the Influenza virus

Photo Credit: CDC/Megan Mathias and J. Todd Parker, Courtesy: Public Health
Image Library
Figure 5.2.84 - Buffered charcoal yeast extract agar
124

Section 38 - Campylobacter jejuni

1. Jiu jitsu at the battle of Iwo Jima: Campylobacter 10. Stray cat: cats are reservoirs
jejuni 11. Eating raw meat and drinking raw milk: transmitted
2. Pink sky: gram negative through the ingestion of undercooked meat and
3. Curved mustache: curved bacilli unpasteurized milk
4. Blue necklace: oxidase positive 12. Injured men bleeding: causes bloody diarrhea
5. Man looking at poop on his shoe: fecal-oral 13. Green berets climbing hill, each one getting more
transmission exhausted toward the top: associated with Guillain-
6. American flag: flagellated (motile) Barré syndrome (ascending paralysis)
7. Hot explosion: grows at 42°C 14. Reporter writing things down: associated with Reiter
8. Pig symbol on aircraft: pigs are reservoirs syndrome (reactive arthritis)
9. Puppy: puppies are reservoirs
125

REVIEW QUESTIONS ?
1. A 6-year-old girl is brought to the physician
due to bloody diarrhea that was first noticed
yesterday. She has no significant past medical
history and has not traveled recently. A stool
culture reveals gram-negative curved bacilli
that are oxidase-positive. The causal organism
is most likely transmitted through which of the
following?
A. Contaminated water
Photo Credit: CDC [Public domain]
B. Sheep
C. Sexual contact
Figure 5.2.85 - Gram stain Campylobacter D. Pigs
E. Respiratory droplets

• Correct answer: D
• This patient has a Campylobacter jejuni
infection
• Bloody diarrhea
• Stool culture reveals gram-negative
curved bacilli that are oxidase-positive
• Campylobacter jejuni can be transmitted
through pigs
• A is incorrect because although Vibrio
cholera is a gram-negative curved bacilli
that is oxidase positive and causes diarrhea,
it doesn’t cause bloody diarrhea and is
spread through contaminated water
• B is a reference to Bacillus anthracis which
is not associated with bloody diarrhea
• C is incorrect because sexual contact is not
typically associated with bloody diarrhea
• E is a reference to many organisms such as
Legionella
126

Section 39 - Vibrio cholerae & Vibrio vulnificus

1. Popped collar: Vibrio cholerae 12. Knocking over pot of rice: causes “rice-water”
2. Pink sky: gram negative diarrhea
3. Curved mustache: curved bacillus 13. Waterfall: causes watery diarrhea
4. Flag: flagellated 14. Large jug of water: treat with oral rehydration
5. Image of Africa: endemic to developing countries solution
6. Acidic lemon juice: acid-labile (large inoculum 15. Volvo logo: Vibrio vulnificus
required) 16. Monkey crackers: Vibrio vulnificus ferments lactose
7. Air mattress pump: proton pump inhibitors increase 17. Oysters: Vibrio vulnificus is associated with raw
the risk of infection oyster consumption
8. Blue necklace: oxidase positive 18. Pack of alcohol: Vibrio vulnificus is associated with
9. Contaminating water with old food: transmitted via alcohol consumption
ingestion of contaminated water 19. Sipping near Volvo: Vibrio vulnificus can cause sepsis
10. Flashlight powered by alkaline batteries: found in 20. Dalmatian dog with a liver-shaped spot: Vibrio
alkaline environments vulnificus can cause liver damage
11. Camping with tents: cholera toxin overactivates
adenylate cyclase resulting in increased levels of
cAMP
127

REVIEW QUESTIONS ?
1. A resident physician is working in Ghana on
a humanitarian trip when a 19-year-old male
presents to the clinic with a 3-day history of
watery diarrhea. He states that he recently
drank some water that may have been
contaminated. A stool sample reveals gram-
negative, oxidase-positive, flagellated bacilli.
The resident informs the medical team that his
condition is caused by a toxin that:
A. Invades the gastrointestinal mucosa
B. Results in an absence of neutrophils in the
stool
C. Spreads hematogenously to vital organs
D. Increases levels of cyclic GMP

Photo Credit: CDC [Public domain]


• Correct answer: B
• This patient has a Vibrio cholerae infection
Figure 5.2.86 - Gram stain of Vibrio cholerae
• Recent travel exposure
• Watery diarrhea
• History of drinking contaminated water
• Stool sample with gram-negative,
oxidase-positive flagellated bacilli
• Vibrio cholerae produces a toxin that
results in rice-water diarrhea
• The toxin is non-inflammatory and
results in an absence of neutrophils
in the stool as it does not invade the
gastrointestinal mucosa
• A is characteristic of the inflammatory
diarrheal illnesses caused by Shigella or
Salmonella
• C is incorrect because this is describing
Salmonella which can disseminate
hematogenously
• D is incorrect because Vibrio cholerae
increases the levels of cyclic AMP - not cyclic
GMP
Photo Credit: F1jmm [CC BY-SA 3.0 (https://creativecommons.org/licenses/
by-sa/3.0)]
Figure 5.2.87 - “Rice-water” diarrhea
128

Section 40 - Helicobacter pylori

1. Pile of dirty clothes: Helicobacter pylori 10. Mattress dipping in the middle with a red spot:
2. Pink walls: gram negative causes peptic ulcer disease
3. Curved mustache: curved bacillus 11. Green fart clouds: can cause chronic gastritis
4. American flag: flagellated (motile) 12. Cancer hope ribbon: can cause gastric carcinoma
5. Silver plate and silverware: can be diagnosed using 13. Malt shake: can cause MALT lymphoma
silver-based staining methods 14. Green burp cloud: diagnosed with a urease breath
6. Antlers: colonizes the antrum test
7. Cat on pile: catalase positive 15. Vacuum pump: treated with proton pump inhibitors
8. Cat urinating on dirty pile: urease positive 16. Ammo belt: treated with amoxicillin
9. Blue necklace: oxidase positive 17. Clarinet: treat with clarithromycin

Photo Credit: CDC/Dr. Edwin P. Ewing, Jr. [Public domain]


Photo Credit: Ed Uthman, MD [CC BY-SA 2.0 (https://creativecommons.org/
licenses/by-sa/2.0)] Figure 5.2.89 - Silver based stain
Figure 5.2.88 - Hematoxylin and eosin (H&E) stain
of the gastric mucosa
129

Figure 5.2.90 - Stomach anatomy and physiology

REVIEW QUESTIONS ?
1. A 27-year-old male with a history of chronic • Correct answer: B
arthritis is brought to the emergency • This patient has a peptic ulcer caused by
department after an episode of syncope which chronic NSAID use
occurred an hour ago. He quickly regained • Chronic arthritis → likely taking NSAIDs
consciousness, but has felt lightheaded since • Recent syncope, black tarry stools,
that time. Additional history reveals that he tachycardia, and hypotension → peptic
began having black tarry stools several days ago. ulcer and an upper GI bleed
His temperature is 37°C (98.6°F), pulse is 120/ • Chronic NSAID use in a patient with a
min, and blood pressure is 80/60. This patient’s painful chronic condition is likely, and
condition is most likely caused by which of the there is no information indicating an
following? H. pylori infection in the question stem
(answer A)
A. Helicobacter pylori
• C is incorrect because while high
B. NSAID use
C. Alcohol use concentrations of alcohol may damage the
D. Hypertriglyceridemia gastric mucosal barrier, there is no evidence
that alcohol causes peptic ulcer disease
• D is suggestive of pancreatitis
130

Section 41 - Mycobacterium tuberculosis

1. Tubers: Mycobacterium tuberculosis 17. Fibrous ends of the blanket: the infected lung tissue
2. Microphone: mycolic acid becomes fibrotic
3. Green acid: acid-fast bacillus 18. TNT remote: TNF-α inhibitors disrupt granuloma
4. Car bomb: carbol fuchsin formation and may “awaken” the infection from its
5. LeBron James signature: can be grown in latent state
Löwenstein-Jensen medium 19. Emaciated kid: secondary TB occurs as health
6. Smoke: enters the lower lobes of the lungs via declines and the immune system becomes weaker
aerosolized droplets 20. Upstairs: secondary TB affects the upper lobes of
7. Cage: engulfed by alveolar macrophages and the lungs
replicates intracellularly 21. Skinny: secondary TB presents with weight loss
8. Television cord: cord factor creates a “serpentine 22. Sweating: secondary TB presents with night sweats
cord” appearance under the microscope 23. Red beet juice near mouth: secondary TB presents
9. Tide bleach: produces a virulence factor known as with hemoptysis
sulfatide 24. Heater: secondary TB presents with a fever
10. Bleach on the floor in separate puddles: sulfatides 25. Millet: miliary TB can occur following secondary TB
prevent fusion of the phagosome and lysosome or primary progressive TB
11. Guy purifying clothes: purified protein derivative 26. Pot of water: Pott disease may be seen in miliary TB
(PPD) test 27. Hat with cavity due to weight of millet seeds:
12. Syringe: the BCG vaccine causes false positive PPD cavitary brain lesions may be seen in miliary TB
tests 28. Smear of ripe food: treat primary, secondary, and
13. Green rays: the IFN-γ release assay (IGRA) has fewer miliary TB infections with RIPE therapy
false positives among people with the BCG vaccine 29. Rifle and grandma holding a glass of ice: treat latent
14. Gong: a Ghon complex may form during primary TB TB with rifampin and isoniazid
15. Person sleeping: latent TB
16. Granny: organisms are walled off via granuloma
formation
131

Photo Credit: CDC/Dr. George P. Kubica [Public domain], via Wikimedia


Commons
Figure 5.2.91 - Acid-fast stain of Mycobacterium
tuberculosis

Photo Credit: https://en.wikipedia.org/wiki/File:LJ_medium.JPG#file


Figure 5.2.92 - Lowenstein-Jensen medium
132

Figure 5.2.93 - Pathophysiology of Mycobacterium tuberculosis


133

REVIEW QUESTIONS ?
1. A 57-year-old woman who emigrated to the U.S.
from Thailand several years ago presents to the
emergency department due to night sweats,
fevers, and weight loss. She states that she
first noticed her symptoms approximately one
week ago. Her past medical history is significant
Photo Credit: Yale Rosen from USA [CC BY-SA 2.0 (https://creativecommons.
org/licenses/by-sa2.0)] for rheumatoid arthritis that is well controlled
Figure 5.2.94 - Miliary tuberculosis with infliximab. Physical examination shows
axillary lymphadenopathy. A chest x-ray reveals
a pulmonary infiltrate in the upper lobe of the
left lung. This patient’s condition can be best
explained by which of the following?
A. Idiopathic pulmonary fibrosis
B. Uncontrolled proliferation of Reed-
Sternberg cells
C. Granuloma dysfunction
D. Polymorphonuclear invasion of alveolar
tissue
Photo Credit: CDC/Ronald K. Smithwick [Public domain]
• Correct answer: C
Figure 5.2.95 - “Serpentine cords”
• This patient has reactivation of latent TB
• From Thailand, night sweats, fever,
weight loss, pulmonary infiltrate in the
upper lobe of the left lung → suggestive
of TB
• Infliximab is a TNF-α inhibitor →
granuloma breakdown → reactivation of
latent TB
• Granulomas form around latent TB
Photo Credit: Greg Knobloch [Public domain] through the action of macrophages
Figure 5.2.96 - PPD test which secrete TNF-α which helps form
epithelioid macrophages and giant cells
• Anti-TNF-α medications can cause
sequestered granulomas to break down
which results in the dissemination of
the bacteria
• A is incorrect because while idiopathic
pulmonary fibrosis can cause shortness of
breath, it’s unlikely to cause fever, night
sweats, weight loss
• B is describing cells that are associated with
Hodgkin lymphoma
• D is incorrect because this is describing
pneumonia

Photo Credit: Basem Abbas Al Ubaidi [CC BY 4.0 (https://creativecommons.


org/licenses/by/4.0)]
Figure 5.2.97 - Ghon complex
134

Section 42 - Mycobacterium leprae

1. Leopards: Mycobacterium leprae 13. Lion near the two lazy leopards: the lepromatous
2. Acid rain: acid-fast bacillus form is associated with leonine facies
3. Car bombs: carbol-fuschin 14. Skull near the two lazy leopards: the lepromatous
4. Microphone: mycolic acid form can be lethal
5. Armadillo: armadillos are reservoirs in the U.S. 15. Big tubercle: tuberculoid form
6. Snow-capped mountains: prefers cool temperatures 16. Caged leopard: the tuberculoid form is associated
7. Prominent red gloves: causes superficial nerve with a strong immune response (T helper type 1
infections resulting in a glove and stocking loss of cells are inactivated)
sensation 17. The cage and tubercle-ridden man are low on
8. Broken petri dish: cannot be grown in vitro the hill near small amounts of green mold: the
9. Biopsy dart: diagnosed with a skin biopsy tuberculoid form is associated with a low bacterial
10. Three-chain fence: diagnosed using polymerase load
chain reaction 18. Bald patches near tubercle: the tuberculoid form is
11. Two lazy leopards: the lepromatous form is associated with hairless skin plaques
associated with a weak immune response (T helper 19. Deputy: dapsone is used to treat both forms of
type 2 cells are activated) leprosy
12. Two leopards high on the hill near high amounts 20. Rifle: rifampin is used to treat both forms of leprosy
of green mold: the lepromatous form is associated 21. Lazy leopard eating a clover: clofazimine is added
with a high bacterial load for patients who have the lepromatous form
135

Photo Credit: U.S. Department of Health and Human Services [Public domain]
Figure 5.2.98 - Acid-fast stain of Mycobacterium leprae

Figure 5.2.99 - Pathophysiology of Mycobacterium leprae


136

Photo Credit: Rosemary A. Jones [CC BY-SA 4.0 (https://creativecommons.org/licenses/by-sa/4.0)]


Figure 5.2.100 - Leonine Facies
137

REVIEW QUESTIONS ?
1. A 41-year-old male who recently emigrated • Correct answer: B
from Nigeria comes to the office due to • The patient has the tuberculoid form of
a skin lesion on his head. He states that leprosy
he first noticed the area a few weeks ago. • From Nigeria
Physical examination reveals a hypoesthetic, • Hypoesthetic, hypopigmented lesion
hypopigmented lesion on the right side of the as well as with an area of baldness →
patient’s forehead. Adjacent to the lesion is an tuberculoid form of leprosy
area of baldness extending several centimeters • Mycobacterium leprae invades the skin
into his hairline. A biopsy is obtained and nerves
and light microscopy reveals numerous • The cell wall of the bacterium has many
pathogens invading the nerve tissue. Which long fatty acids which is describing
of the following is true regarding this patient’s mycolic acid found in Mycobacterium
condition? leprae
• A is incorrect because this patient has
A. There is an abundance of Th2 cells in
the tuberculoid form of leprosy which is
response to the infection
B. The cell wall of the pathogen contains many characterized by a Th1 response - not a Th2
long fatty acids response
C. The causal organism prefers warm • C is not characteristic of Mycobacterium
temperatures leprae (it prefers cool temperatures)
D. IL-4 is the cytokine involved in this immune • D is incorrect because IL-4 is more involved
response with Th2 responses
138

Section 43 - Mycobacterium avium-intracellulare, Scrofulaceum, and Marinum

1. Aviary: Mycobacterium avium-intracellulare (MAC) 11. Man using a zip line and wearing a shirt with all 50
2. Child with a scruffy beard: Mycobacterium states: azithromycin is given when the CD4 count is
scrofulaceum < 50
3. Marine biologist bitten by piranha: Mycobacterium 12. Spilled medicine container: MAC is often resistant to
marinum multiple drugs
4. Acid pond: acid-fast 13. Scruffy beard on a child: M. scrofulaceum is
5. Microphone: mycolic acid common in children
6. Man in the aviary has a Band-Aid: MAC causes 14. Scruffy child holding a beaded net near his neck:
disease in AIDS patients M. scrofulaceum causes cervical lymphadenitis in
7. Bead-like segments in the aviary: MAC causes children
lymphadenitis 15. Marine biologist working closely with piranhas:
8. Heat lamp in the aviary: MAC causes a fever M. marinum causes hand infections in aquarium
9. Sweating man within the aviary: MAC causes night handlers
sweats
10. Skinny man within the aviary: MAC causes weight
loss
139

REVIEW QUESTIONS ?
1. A 33-year-old male with a history of HIV
presents to the physician due to weight loss,
fever, and a swollen lump in his neck. His
temperature is 38.8°C (101.8°F). Physical
examination reveals a swollen and tender
cervical lymph node. A tissue aspirate of the
swollen lymph node reveals acid-fast bacilli. A
CD4 count is obtained and will most likely reveal
which of the following?
A. 43 cells/microL
Photo Credit: CDC/Dr. Edwin P. Ewing, Jr. [Public domain] B. 97 cells/microL
Figure 5.2.101 - Acid-fast stain of Mycobacterium C. 439 cells/microL
avium-intracellulare D. 991 cells/microL
E. 1,503 cells/microL

• Correct answer: A
• The patient has MAC
• Weight loss
• Fever
• Cervical lymphadenitis
• Tissue aspirate revealed acid-fast bacilli
• HIV diagnosis makes an AIDS-defining
illness such as MAC more likely than the
rare M. scrofulaceaum
• MAC infections are most common when
the CD4 count drops below 50 which
is why these patients may be given
prophylactic azithromycin
• B-E are all >50 cells/microL
140

Section 44 - Borrelia burgdorferi

1. Burglar: Borrelia burgdorferi 10. Bell: stage 2 is associated with Bell’s palsy
2. Spiral staircase: spirochete 11. Car on fire: stage 2 is associated with carditis
3. “Northeast corner” sign in the northeast part of the 12. Blocking customers from car store: stage 2 is
image: common in the northeastern part of the U.S. associated with atrioventricular block
4. Tic tacs near exit sign: transmitted through the 13. Red knees and elbows: stage 3 is associated with
Ixodes tick migratory arthritis
5. Anne of Green Gables: the Ixodes deer tick is also a 14. Water all over guy’s head: stage 3 is associated with
vector for Anaplasma and Babesia encephalitis
6. Games: can be visualized using the Giemsa stain 15. Dirt bike: treat with doxycycline
7. Deer chasing a mouse: deer and mice are reservoirs 16. Ammo near pregnant woman and child: treat with
and important to the tick life cycle amoxicillin (pregnant women and children)
8. Target: stage 1 is associated with a “bulls-eye” rash 17. Cephalopod toy: treat with 2nd generation
known as erythema migrans cephalosporins (pregnant women and children)
9. Sweat: stage 1 is associated with flu-like symptoms

Photo Credit: Content Provider(s): CDC [Public domain] Photo Credit: Courtesy of the U.S. Department of Health and Human Services
Figure 5.2.102 - Spirochetes via dark-field Figure 5.2.103 -  Ixodes deer tick
microscopy
141

Photo Credit: Alan R. Walker [CC BY-SA 3.0 (https://creativecommons.org/ Photo Credit: https://upload.wikimedia.org/wikipedia/commons/1/1d/
licenses/by-sa/3.0)] Bellspalsy.JPG

Figure 5.2.104 - Giemsa stain of Borrelia Figure 5.2.106 - Bell’s palsy

REVIEW QUESTIONS ?
1. A 50-year-old female presents to the physician
due to a rash on her leg that she first noticed
yesterday. She also states that she has felt
general malaise since returning home from a
hiking trip to the Appalachian mountains three
days ago. Physical examination reveals a rash
on the left leg with a central area of erythema
and a surrounding erythematous ring. If left
untreated, which of the following complications
is most likely to occur in this patient?
A. Pneumonia
B. Meningitis
C. Arthritis
D. Hepatitis
E. Aplastic anemia
Photo Credit: James Gathany, Content Provider(s): CDC/James Gathany
[Public domain] • Correct answer: C
Figure 5.2.105 - Erythema migrans • This patient has erythema migrans
associated with stage 1 of Lyme disease
• Recently travel to the Appalachian
mountains (east coast of the U.S.)
• Rash on the left leg with a central
area of erythema and a surrounding
erythematous ring → erythema migrans
(bulls-eye rash) → stage 1
• Migratory arthritis is a common
complication if left untreated
• A, B, D, and E are not commonly associated
with Lyme disease
142

Section 45 - Leptospira interrogans

1. Spear: Leptospira interrogans 9. Sweating surfer: leptospirosis causes flu-like


2. Spiral waves: spirochete symptoms including myalgias
3. Fishing hook near contaminated water: 10. Whale: Weil disease
hook-shaped ends commonly found in water 11. Dalmatian with liver-shaped spot: Weil disease can
contaminated with animal urine cause liver dysfunction
4. Tropical trees: leptospirosis is prevalent in the 12. Yellow surfboard: Weil disease can cause jaundice
tropics 13. Heat lamp: Weil disease can cause a fever
5. Surfer: leptospirosis is prevalent among surfers 14. Uncontrolled bleeding: Weil disease can cause
6. Yellow surfboard: leptospirosis can cause jaundice hemorrhage and anemia
7. Sunglasses and looking away from the sun: 15. Kidney-shaped shrimp: Weil disease can cause renal
leptospirosis can cause photophobia failure
8. Blood spattered on sunglasses: leptospirosis can
cause conjunctival erythema
143

REVIEW QUESTIONS ?
1. A 24-year-old male presents to the physician
due to myalgias and a headache. He states that
he first noticed his symptoms after returning
home from a surfing trip to Hawaii two days
ago. Physical examination reveals conjunctival
erythema. The physician suspects a spirochete
infection and immediately begins antibiotic
therapy. Which of the following complications
is most likely associated with this patient’s
condition?
Photo Credit: CDC Public Health Image Library. CDC/NCID/HIP/Janice Carr A. A painful genital ulcer
(PHIL #1220) B. A maculopapular rash on the trunk
Figure 5.2.107 - Electron microscopic image of C. A painless genital ulcer
Leptospira interrogans D. Renal failure

• Correct answer: D
• This patient has an infection caused by
Leptospira interrogans
• Surfing in Hawaii
• Developed myalgias, a headache, and
conjunctival erythema
• A spirochete infection is suspected
→ Leptospira interrogans is the likely
cause
• Weil disease can cause renal failure in
individuals infected with Leptospira
interrogans
• A is associated with Haemophilus ducreyi
• B is incorrect as it is associated syphilis
which has a rash that affects the palms and
soles
• C is describing the classic lesion seen in
syphilis (painless chancre)
144

Section 46 - Treponema pallidum

1. Paladin: Treponema pallidum 14. Lumpy flail: secondary syphilis may present with a
2. Spiral staircase: spirochete wart-like lesion on the genitals known as condyloma
3. Fighting dark creature with magic: dark field lata
microscopy can identify Treponema pallidum 15. Beaded net: secondary syphilis may present with
4. Armor with the letter “S”: sexually transmitted lymphadenopathy
infection 16. Bald patches on mangy monster: secondary syphilis
5. Hammer: Jarisch-Herxheimer reaction may present with patchy hair loss
6. Skull symbol on the hammer: Jarisch-Herxheimer 17. Grandpa-looking wizard: tertiary syphilis may
reaction occurs following the destruction of the present with gummas
organism 18. Gannondolf wearing a hat: neurosyphilis is a
7. Hammer embedded with copper pennies: common complication of tertiary syphilis
symptoms associated with the Jarisch-Herxheimer 19. Damaged columns: tertiary syphilis causes tabes
reaction begin following treatment with penicillin dorsalis (demyelination of the nerves within the
8. Shielding cane attack: primary syphilis presents with dorsal column)
a painless genital chancre 20. Aorta-shaped tree: tertiary syphilis may cause
9. Spilling vitriol: VDRL test aortitis resulting in a tree barking appearance of the
10. Tombstone reading “RIP”: RPR test aorta
11. Heart and bones that resemble phospholipids are 21. Eyes appearing angry and focused: tertiary syphilis
poking up from the ground near the vitriol and may present with Argyll Robertson pupils
“RIP” tombstone: the presence of anticardiolipin 22. Hobbit with swollen red knees: tertiary syphilis may
antibodies indicates a positive test result present with neuropathic arthropathy (Charcot
12. Fire arrow shot between the goblin’s eyes: the FTA- joint)
ABS test is the “confirmatory” test 23. Baby Balrog: congenital syphilis
13. Palms and soles with hair and bumps: secondary 24. Balrog’s notched teeth: congenital syphilis may
syphilis may present with a maculopapular rash cause notched teeth
involving the trunk, palms, and soles 25. Intricate horns near the ears: congenital syphilis
may cause hearing loss
145

26. Odd-shaped nose: congenital syphilis may cause 29. Deformed shins: congenital syphilis may cause saber
saddle nose shins
27. Fire coming out of mouth, nose, and head: 30. Fighting the Balrogs using a penny-based spell:
congenital syphilis may cause snuffles (nasal congenital syphilis can be prevented if the mother is
discharge laden with Treponema pallidum) treated with penicillin early during pregnancy
28. Wide openings near the sides of the mouth: 31. Pennies on the hammer and used against the
congenital syphilis may cause rhagades (wrinkled Balrog: syphilis is treated with penicillin
skin and linear scars at the angle of the mouth)

Photo Credit: Herbert L. Fred, MD, Hendrik A. van Dijk [CC BY-SA 3.0 (https://
creativecommons.org/licenses/by-sa/3.0)]
Photo Credit: Content Providers: CDC/C.W. Hubbard [Public domain] Figure 5.2.110 - Maculopapular rash
Figure 5.2.108 - Dark-field photomicrograph of
Treponema pallidum

Photo Credit: CDC/Robert Sumpter [Public domain]


Figure 5.2.111 - Maculopapular rash on the palms
Photo Credit: Courtesy of the U.S. Department of Health and Human Services
and M.Rein
Figure 5.2.109 - Chancre
146

Photo Credit: Courtesy of the U.S. Department of Health and Human Services
and Susan Lindsley
Figure 5.2.112 - Condyloma lata Photo Credit: Courtesy of the U.S. Department of Health and Humna Services
and Susan Lindsley
Figure 5.2.115 - Notched teeth

Photo Credit: CDC [Public domain]


Figure 5.2.113 - Gumma

Figure 5.2.114 - Cross section of the spinal cord


147

REVIEW QUESTIONS ?
1. A 23-year-old male comes to the physician due
to a small painless lesion on his penis that he
noticed yesterday. He is concerned he may have
a sexually transmitted infection because 4 days
ago he had unprotected sexual intercourse with
a prostitute. Physical examination reveals a
painless ulcer on the shaft of the penis. A serum
Venereal Disease Research Laboratory (VDRL)
test is negative. Which of the following should
be performed to make an accurate diagnosis of
this patient’s condition?
A. The rapid plasma reagin (RPR) test
B. Repeat the Venereal Disease Research
Laboratory (VDRL) test
C. The Toluidine Red Unheated Serum Test
(TRUST)
D. The fluorescent treponemal antibody
absorption (FTA-ABS) test

• Correct answer: D
• This patient has a painless lesion on his
Photo Credit: https://en.m.wikipedia.org/wiki/File:Saddle_nose_38.jpg penis (chancre), which is a feature of
Figure 5.2.116 - Saddle nose primary syphilis
• The VDRL is negative but this does not
rule out Treponema pallidum infection
due to the time it takes to develop an
immune response (about 4 weeks)
• False negatives are common early in the
course of infection
• The fluorescent treponemal antibody
absorption (FTA-ABS) test is a
confirmatory test that detects the
presence of Treponema pallidum
antigens and should be performed in
this case
• Always remember to perform both
Photo Credit: Courtesy of the U.S. Department of Health and Humnan a nontreponemal test (VDRL or RPR)
Services and Robert Sumpter and a treponemal test (FTA-ABS) when
Figure 5.2.117 - Rhagades diagnosing cases of syphilis
• Answers A-C are incorrect because they
are all nontreponemal tests which are
based upon the detection of anticardiolipin
antibodies (the patient already had the
nontreponemal VDRL test so it would be
pointless to perform another test with a
similar mechanism)
148

Section 47 - Chlamydia trachomatis, Chlamydophila pneumoniae, and Chlamydophila


psittaci

1. Clams: Chlamydia 17. Red car fragments and coughing babies: serotypes
2. Letter “S” on shirt: most serotypes of  Chlamydia D-K cause neonatal pneumonia with eosinophilia
trachomatis are sexually transmitted 18. Two babies with red eyes: serotypes D-K cause
3. Cage: obligate intracellular (cannot make own ATP) neonatal conjunctivitis 1-2 weeks after birth
4. Hugging elementary school kids: elementary bodies 19. Limping granny: serotypes L1, L2, and L3 cause
enter the cell via endocytosis lymphogranuloma venereum
5. Rakes inside school: Reticulate bodies replicate 20. Cup: serotypes L1, L2, and L3 cause painless genital
inside of the cell by fission ulcers
6. Kids leaving elementary school: Reticulate bodies 21. Swollen lesions on side of legs: serotypes L1, L2, and
are reorganized into elementary bodies, released L3 cause painful inguinal buboes
from the cell, and then infect adjacent cells 22. Dirt bike: serotypes L1, L2, and L3 are treated with
7. Broken mirror: the cell wall lacks muramic acid doxycycline
8. Games: identified using the Giemsa stain 23. Writing: Reiter syndrome
9. Gnats: diagnosed with NAAT (amplifies bacterial 24. Gondola toys: Gonorrhoeae coinfection
DNA or RNA sequences using PCR) 25. New mower with clam logo: Chlamydophila
10. Apples: serotypes A, B, and C pneumoniae
11. Dog: serotypes D-K 26. Pistachios: Chlamydophila psittaci
12. Lamp: serotypes L1, L2, and L3 27. Smoke: C. pneumoniae and C. psittaci are
13. Holding hands: serotypes A, B, and C exhibit hand- transmitted through aerosolized droplets
to-hand transmission 28. Coughing guys: C. pneumoniae and C. psittaci cause
14. Blind African American man: serotypes A, B, and C pneumonia
are common in Africa and can cause trachoma 29. Blue bird: birds are reservoirs for C. psittaci
15. Pelvis on fire: serotypes D-K cause urethritis and PID 30. Trident: treat with ceftriaxone
16. Pregnant woman partially inside and outside of car: 31. Zip-line: treat with azithromycin
serotypes D-K cause PID which increases the risk of
ectopic pregnancy
149

REVIEW QUESTIONS ?
1. A 12-day-old infant is brought to the emergency
department by his mother due to red eyes.
She states that she did not have prenatal care
due to financial concerns, but that the boy
was delivered at home without complications.
Physical examination reveals erythema and
mucopurulent discharge surrounding both eyes.
The infectious serotype responsible for this
patient’s condition may also cause:
A. Groin pain and a painless genital ulcer
B. Trachoma
C. Septic arthritis
D. Pelvic inflammatory disease

Photo Credit: Huckfinne [Public domain] • Correct answer: D


Figure 5.2.118 - Life cycle of Chlamydia trachomatis • This patient has neonatal conjunctivitis
caused by Chlamydia trachomatis
• Normal onset is 5-14 days after birth
• Specifically caused by serotypes D-K
• Serotypes D-K also cause pelvic
inflammatory disease
• A is incorrect because it describes
lymphogranuloma venereum which is
caused by serotypes L1, L2, and L3
• B is incorrect because this is the leading
cause of blindness in the world and is
caused by serotypes A, B, and C
• C is incorrect because it is a complication
associated with Gonorrhea but Gonorrhea
Photo Credit: Doctors E. Arum and N. Jacobs [Public domain]
causing neonatal conjunctivitis typically
Figure 5.2.119 - Giemsa stain of Chlamydia occurs 2-5 days after birth
150

Section 48 - Coxiella burnetii

1. Brunette queen: Coxiella burnetii 9. Coughing stepsister: patients with Q fever may
2. Pink sunset: gram negative present with pneumonia
3. Queens crown: causes Q fever 10. Liver-shaped spot: patients with Q fever may
4. Cow placenta: transmitted through the aerosols of present with hepatitis and transaminitis
cattle amniotic fluid 11. Sweating stepsister: patients with Q fever may
5. Sheep wool: transmitted through the aerosols of present with flu-like symptoms
sheep amniotic fluid 12. Broken plate: patients with Q fever may present
6. Plate of snails: a sporulation-like process protects with thrombocytopenia
the organism from harsh environmental conditions 13. Broken petri dish near cart: patients with Q fever
7. Caged sheep: obligate intracellular organism may present with culture-negative endocarditis
8. Black horses: a rash is rarely seen in patients with Q 14. Dirt bike: treat with doxycycline
fever
151

REVIEW QUESTIONS ?
1. A 27-year-old male presents to the emergency
department due to a subjective fever, myalgias,
and shortness of breath for the past several
days. He has also noticed strange bumps on the
pads of his fingers that are quite tender. When
asked about recent changes in his life he states
that approximately one month ago he was hired
to work on a dairy farm. Physical examination
reveals a cardiac murmur and tender raised
lesions on his finger pads. Blood cultures are
Photo Credit: CDC Public Health Image Library [Public domain]
obtained but no organisms are grown. Which
Figure 5.2.120 -  Gram stain of Coxiella burnetii of the following additional findings would most
likely be observed in this patient?
A. An AST of 19 U/L (normal: 8-20 U/L)
B. A platelet count of 73,000/mm3 (normal:
150,000-400,000/mm3)
C. A hemoglobin of 10.4 g/dL (normal: 13.5-
17.5 g/dL)
D. A chest radiograph with bilateral hilar
adenopathy

• Correct answer: B
• This patient has Q fever
• Subjective fever
• Myalgias
Photo Credit: Roberto J. Galindo [CC BY-SA 4.0 (https://creativecommons.org/ • Shortness of breath
licenses/by-sa/4.0)] • History of working on a dairy farm
Figure 5.2.121 -  Osler’s nodes • Osler nodes and negative blood
cultures → culture-negative bacterial
endocarditis
• Patients with Q fever may have
thrombocytopenia
• A is incorrect because Q fever also causes
hepatitis so a normal AST would be less
likely than an elevated AST
• C is incorrect because anemia is an
extremely rare finding in patients with Q
fever
• D is incorrect because while fever can cause
pneumonia, this description is suggestive of
sarcoidosis - not pneumonia
152

Section 49 - Gardnerella vaginalis

1. Garden: Gardnerella vaginalis 7. Roses: inflammation caused by overgrowth of


2. Clue: clue cells normal vaginal flora
3. Decayed skin edges: stippled edges 8. Blue and red garden bugs: gram variable
4. Gray skin: grayish vaginal discharge 9. Cleaning lady: clindamycin as treatment
5. Fish: fishy smell 10. Bleach: vaginal pH>4.5 (alkaline)
6. Vagina shaped flower on shield: painless vaginitis 11. Metro: metronidazole as treatment
153

REVIEW QUESTIONS ?
1. A 29-year-old female presents to the physician
complaining of vaginal discharge with a fishy
odor for the past 4 days. The physician obtains
a sample of the discharge for laboratory tests.
Following microscopic examination, the lab does
not identify any fungi, protozoa, or bacteria
foreign to the vagina. Epithelial cells covered
with normal vaginal flora were seen. What
is the likely pH range of the patient’s vaginal
discharge?

• Correct answer: pH > 4.5


• Gardnerella vaginalis infection is suspected
due to the following:
• Vaginal discharge with a fishy odor
• Clue cells (epithelial cells covered with
Photo Credit: CDC/M. Rein [Public domain], via Wikimedia Commons normal vaginal flora)
Figure 5.2.122 - Clue cells • Lack of fungi makes Candidal vaginitis
unlikely
• Lack of protozoa makes Trichomonas
vaginalis unlikely
154

Section 50 - Mycoplasma pneumoniae

1. Plasma alien: Mycoplasma pneumoniae 8. Snow and cold: agglutination and hemolysis occurs
2. Blobby mass: Mycoplasma lacks a cell wall at cold temperatures
3. Coughing: causes pneumonia 9. X-ray machine damaging alien blob: chest
4. Eaten man: grows on Eaton agar radiographs reveal nodular densities
5. College dorm: outbreaks common with young adults 10. Wincing monster: infection causes headaches
(<30) in dormitories 11. Pennies going right through the alien: the absence
6. Military: outbreaks common with military recruits of a cell wall confers beta-lactam resistance
(and prisons) 12. Stick with flowers: treat with fluoroquinolones
7. Snowflakes and bleeding alien: IgM antibodies cause 13. Dirtbike: treat with doxycycline
cold agglutination and hemolysis 14. Scarecrow: treat with macrolides

 
Photo Credit: Martin Brandli (brandlee86) [CC BY-SA 2.5 (https://creativecommons.org/licenses/by-sa/2.5)]
Figure 5.2.123 - IgM antibodies
155

REVIEW QUESTIONS ?
1.  A 23-year-old female presents to her
primary care physician with persistent
fatigue, headaches and a cough. She states
she is a freshman at the local college and
recently moved into a nearby dormitory.
Further history demonstrates that several
of her classmates have also had similar
symptoms. Her sputum is collected for
staining and culturing. The gram stain does
not reveal the presence of any organisms,
and the organism only grows when cultured
on Eaton agar. Based on the information
above, which of the following is most likely
true regarding her infection?
A. A broad-spectrum antibiotic such as
vancomycin should be used
B. Her blood would agglutinate if collected in a
test tube and warmed
C. A chest radiograph would show
enhancement of an entire lobe
D. Azithromycin will likely kill the suspected
infectious organism

• Correct answer: D
• Azithromycin is a type of macrolide,
a class of drugs that effectively kill
Mycoplasma pneumoniae
• A is incorrect because vancomycin works
by targeting the cell wall, something that
Mycoplasma pneumoniae lacks
• B is incorrect because her blood would
agglutinate at cold temperatures, not hot
temperatures
• C is incorrect because a chest radiograph
would show scattered nodules, not lobar
pneumonia
156

Section 51 - Rickettsia prowazekii, Rickettsia rickettsii, and Rickettsia typhi

1. Rocky Mountains: Rocky mountain spotted fever 14. Prowling bug: Rickettsia prowazekii
2. Destroyed cabin walls: no cell wall 15. Prowling bug and fleas on belly: Rickettsia
3. Spots: Rocky mountain spotted fever prowazekii can also cause typhus
4. Man dressed as a tick: spread by tick bites 16. “Pedicure party”: Pediculus species transmit
5. Tied at wrists and ankles: rash starts at wrists and Rickettsia prowazekii
ankles 17. Eggs carried by boat to a distant land: Rickettsia
6. Delirious: confusion prowazekii causes epidemic typhus
7. Sweating: can cause fevers and sweats 18. Typhoon sign: Rickettsia typhi causes endemic
8. Weary muscles: cause muscle aches typhus
9. Vomiting: can cause projectile vomiting 19. Caged monster: Rickettsia species are obligate
10. Tennis racket: Rickettsia rickettsii causes Rocky intracellular bacteria
mountain spotted fever 20. Chlorine: can treat with chloramphenicol
11. Typhoon warning sign: Rickettsia typhi 21. Dirtbike: first-line treatment is doxycycline
12. Flea-phoon: Rickettsia typhus is spread by flea bites 22. Pregnant woman on dirt bike: caution when
13. Fleas on belly: Typhus rash starts on the trunk prescribing doxycycline to pregnant individuals
before spreading to the rest of the body (spares 23. Whale: diagnosed with Weil-Felix test
palms and soles) 24. Games: diagnosed with Giemsa stain

Photo Credit: CDC [Public domain]


Figure 5.2.124 - Rocky Mountain Spotted Fever
157

Figure 5.2.125 - Weil Felix Test

REVIEW QUESTIONS ?
1. A 19-year-old female presents to the physician • Correct answer: D
with a pruritic rash on her abdomen, arms • The patient likely has typhus from R.
and legs. She states that the rash started on typhi for the following reasons:
her abdomen soon after she brought a new • Her rash started on her trunk and
dog home from the local pound. The physician spread to her arms and legs
recognizes that the rash is likely caused by a • The infection is endemic to the
rickettsial infection. The physician tells the region
patient that he sees this type of infection three • Doxycycline is a tetracycline that
or four times each year and has for the past 20 effectively treats Rickettsial infections.
years. The physician also indicates that he has • A is incorrect because Rickettsial species
never seen this infection prior to moving to the do not gram stain well, which is why the
area. Which of the following is true regarding Giemsa stain or the Weil-Felix test are
the most likely causal organism? required
• B is incorrect because fleas transmit
A. It will appear purple on a gram stain
endemic typhus, not ticks
B. It is spread by ticks endemic to the area
C. It is spread by an organism belonging to the • C is incorrect because lice (pediculus
pediculus genus species) transmit epidemic typhus (R.
D. A tetracycline should be administered prowazekii), not endemic typhus (R. typhi)
158

Section 52 - Ehrlichia & Anaplasma

1. Early alarm clock: Ehrlichia 8. Completely black blankets: a rash is uncommon


2. Anne of Green Gables: Anaplasma 9. Balloon that resembles monocytes: Ehrlichia may be
3. Destroyed wall: no cell wall seen in the cytoplasm of monocytes on a peripheral
4. Hamster cage: obligate intracellular blood smear
5. 2E on the east side of picture: common in the East 10. Grain crumbs: Anaplasma may be seen in the
Coast of the U.S. cytoplasm of granulocytes on a peripheral blood
6. Tic Tacs: transmitted by the Ixodes deer tick smear
7. Sick with thermometers in mouths: Ehrlichia and 11. Dirt bikes: treat with doxycycline
Anaplasma cause nonspecific flu-like symptoms
159

REVIEW QUESTIONS ?
1. A 12-year-old boy is brought to the physician
24 hours after developing chills, myalgias,
and a subjective fever. He recently returned
home from a scouting trip to Connecticut.
His temperature is 38.9°C (102°F), pulse is
114/min, and blood pressure is 112/79. A
peripheral blood smear is obtained and reveals
an organism present within the cytoplasm of
granulocytes. The most likely causal organism
shares a vector with which of the following?
A. Treponema pallidum
B. Coxiella burnetii
C. Leptospira interrogans
Photo Credit: CDC [Public domain] D. Borrelia burgdorferi
Figure 5.2.126 - Ehrlichia within a monocyte
• Correct answer: D
• Reasons why this patient has an Anaplasma
infection:
• The patient has chills, myalgias, and a
fever since returning home from a trip
to Connecticut
• A peripheral blood smear has revealed
an organism present in the cytoplasm of
granulocytes
• Anaplasma and Borrelia burgdorferi are
both transmitted through the Ixodes deer
tick
• A is incorrect because this is a sexually
transmitted infection
• B is incorrect because this is transmitted
Photo Credit: Alan R. Walker [CC BY-SA 3.0 (https://creativecommons.org/ through aerosols of cattle and sheep
licenses/by-sa/3.0)] amniotic fluid
Figure 5.2.127 - Anaplasma within a granulocyte • C is incorrect because this is transmitted
through water contaminated with animal
urine
160

Section 53 - Miscellaneous Organisms

1. B​ lue sky: gram positive 15. Wearing protective mouth mask: Fusobacterium is
2. Whipping out eyeball: Tropheryma whipplei anaerobic​
3. Stealing many small purple balls: periodic acid-Schiff 16. Using surgical tubing as funnel: Fusobacterium is
(PAS) positive associated with surgical drain infections​
4. Man lacking several toes: Lactobacillus 17. Molotov cocktails near fuse: Fusobacterium is
5. Man spraying potent antibiotics and killing green in associated with alcoholic aspiration
the area: antibiotics can reduce the population of 18. Molotov cocktails burning cube-shaped ulcers in the
Lactobacilli in the vagina grass: Fusobacterium is associated with decubitus
6. Planting Canadian flag in the now open area: a ulcers
decreased population of Lactobacilli increases the 19. Wheel of provolone cheese: Prevotella spp.
risk of vaginal Candidiasis 20. Wearing protective mouth mask: Prevotella is
7. Garden growing in the now open area: a decreased anaerobic
population of Lactobacilli increases the risk of 21. Molotov cocktails burning cube-shaped ulcers in the
bacterial vaginosis (Gardnerella vaginalis) grass: Prevotella is associated with decubitus ulcers​
8. Pepto-bismol: Peptostreptococcus spp. 22. Molotov cocktails near fuse: Prevotella is associated
9. Wearing protective mouth mask: with alcoholic aspiration
Peptostreptococcus is anaerobic 23. Using surgical tubing as funnel: Prevotella is
10. Pouring Pepto-bismol into Molotov cocktail: associated with surgical drain infections​​
Peptostreptococcus is associated with alcoholic 24. Box labeled “FRAGILE”: Bacteroides fragilis
aspiration 25. Cube-shaped grass ulcers near the “FRAGILE” box:
11. Toxic fluid creating cube-shaped ulcers in the grass: Bacteroides fragilis is associated with decubitus
Peptostreptococcus is associated with decubitus ulcers
ulcers 26. Molotov cocktails within “FRAGILE” box: Bacteroides
12. Using surgical tubing as funnel: Peptostreptococcus fragilis is associated with alcoholic aspiration
is associated with surgical drain infections 27. Surgical tubing as funnels: Bacteroides fragilis is
13. Red colors: gram negative associated with alcoholic aspiration
14. Lighting a fuse: Fusobacterium spp.
161

28. “FRAGILE” box sitting on sewer drain: Bacteroides 40. Club-wielding granny​wearing African apparel:
fragilis is not part of the normal colonic flora Klebsiella granulomatis is most prevalent in Africa
29. Blast from “FRAGILE” box that exposes abdominal 41. Beef and blood on crotch: Klebsiella granulomatis
organs: Bacteroides fragilis is associated with intra- causes beefy, red genital ulcers (granuloma
abdominal infections inguinale)
30. Bird holes: Burkholderia cepacia complex 42. Stains do not cause pain: ulcers are painless
31. Water softener with dysregulated, lost salt: 43. Club-wielding granny​wearing a scarlet “S”:
Burkholderia infections occur in patients with cystic Klebsiella granulomatis is sexually transmitted
fibrosis 44. Half-bloody (bipolar) cow: Klebsiella granulomatis
32. Water softener dust causing a cough: Burkholderia demonstrates bipolar Donovan bodies on
causes pneumonia in cystic fibrosis patients microscopy
33. Cat ready to attack bird: Burkholderia is catalase 45. Rubber duckies: Haemophilus ducreyi
positive 46. Painful groin hit from ducky impact: Haemophilus
34. Timeshares in the Bahamas: Acinetobacter ducreyi​causes painful genital ulcers
baumannii 47. Green decorative beads lining the groin:
35. Cat biting Bahamas salesman: Acinetobacter Haemophilus ducreyi​causes inguinal
baumannii is catalase positive lymphadenopathy
36. Coughing from smoke due to overheating car: 48. Mom wearing scarlet “S”: Haemophilus ducreyi​is
Acinetobacter baumannii causes pneumonia sexually transmitted
37. Computer hacker: HACEK (Haemophilus 49. Urinating on plasma television: Ureaplasma spp.
haemolyticus + parahaemolyticus, Aggregatibacter, 50. Urine pooling on the ground: Ureaplasma is urease
Cardiobacterium, Eikenella, and Kingella) positive
38. Car running over the petri dish: HACEK organisms 51. Electric shock to the urethra: Ureaplasma causes
are often the cause of culture negative endocarditis urethritis
39. Club-wielding granny: Klebsiella granulomatis 52. Electric shock destroying brick wall: Ureaplasma has
no cell wall and does not gram stain well
162

Organism Morphology Other Features Clinical Significance


• Diagnose with
Gram positive
Tropheryma whipplei periodic acid-Schiff • Whipple disease
bacilli
(PAS) stain
• Population decreased by antibiotics and
Gram positive • Part of normal
Lactobacillus douching → predisposes to Candida and
bacilli vaginal flora
Gardnerella vaginalis infections
• Decubitus ulcers
Peptostreptococcus Gram positive
• Anaerobic • Can infect surgical drains
spp. cocci
• Associated with alcoholic aspiration
• Decubitus ulcers
Gram negative
Fusobacterium spp. • Anaerobic • Can infect surgical drains
bacilli
• Associated with alcoholic aspiration
• Decubitus ulcers
Gram negative
Prevotella spp. • Anaerobic • Can infect surgical drains
bacilli
• Associated with alcoholic aspiration
• Decubitus ulcers
Gram negative • Anaerobic • Can infect surgical drains
Bacteroides fragilis
bacilli • Normal colonic flora • Associated with alcoholic aspiration
• Intra-abdominal infections
Burkholderia cepacia Group of gram
• Catalase positive • Pneumonia in cystic fibrosis patients
complex negative bacilli
• Community acquired pneumonia
Acinetobacter Gram negative
• Catalase positive • Associated with mechanical ventilation/
baumannii coccobacilli
intubation
Gram negative • Sexually transmitted • Chancroid (painful genital ulcers)
Haemophilus ducreyi
coccobacilli infection • Inguinal lymphadenopathy
• Sexually transmitted
infection
Klebsiella • Granuloma inguinale (painless beefy red
Gram negative • Bipolar staining
(calymmatobacterium) ulcer that easily bleeds)
bacilli seen on microscopy
granulomatis • Rare in USA
(Donovan bodies in
cytoplasm)
HACEK (Haemophilus
haemolyticus +
parahaemolyticus, Gram negative • Associated with culture negative
Aggregatibacter, bacilli endocarditis
Cardiobacterium,
Eikenella, and Kingella)
Pleomorphic (no
Ureaplasma
cell wall: poor • Urease positive • Urethritis
urealyticum
gram staining)
Table 5.2.1 - Miscellaneous organisms
163

Photo Credit: By Ed Uthman via Flickr (CC SA 2.0)


Figure 5.2.128 - Intestinal biopsy in Whipple disease

Photo Credit: Public domain via Flickr


Figure 5.2.131 - Bipolar Donovan bodies from
granuloma inguinale ulcer biopsy

Photo Credit: GerardM via Wikimedia Commons


Figure 5.2.129 - Granuloma inguinale caused by
Klebsiella granulomatis (male)

Photo Credit: GerardM via Wikimedia Commons


Figure 5.2.130 - Granuloma inguinale caused by
Klebsiella granulomatis (female)
164

REVIEW QUESTIONS ?
1. A 45-year-old male with a history of alcoholism
presents to the emergency department with a
painful, productive cough. He reports that his
symptoms began shortly after a visit to a local
bar. Upon awakening, he discovered that he was
face down in a puddle of beer. A radiograph of
his chest is obtained which is suggestive of lobar
pneumonia. His sputum is cultured and a gram
stain reveals a gram negative bacillus. Which
of the following is the most likely infectious
organism?
A. Burkholderia cepacia complex
B. Tropheryma whipplei
C. Haemophilus ducreyi
D. Peptostreptococcus spp.
E. Fusobacterium spp.

• Correct answer: E
• The following items are consistent with
a Fusobacterium infection:
• Pneumonia
• Alcoholic aspiration
• Gram negative organism
• Choice A is incorrect because Burkholderia
cepacia complex occurs almost exclusively
with cystic fibrosis, a disease not suspected
in this patient
• Choice B is incorrect because Tropheryma
whipplei is a gram positive organism that
causes Whipple disease
• Choice C is incorrect because Haemophilus
ducreyi causes painful genital ulcers, not
pneumonia
• Choice D is incorrect because
Peptostreptococcus is gram positive
165

FUNGI

Figure 5.3.1 - Fungi overview figure


166

Section 1 - Histoplasma capsulatum

1. Watching the History channel: Histoplasma 13. Shawl with an “A” on it: treat local infections with
capsulatum azole medications (eg, itraconazole, fluconazole,
2. Butterfly: dimorphic (mold at 20°C and yeast at voriconazole)
37°C) 14. Amphibian running from dog: treat systemic
3. Ohio state shirt and Mississippi mud pie: common in infections with amphotericin B
the Mississippi and Ohio River valleys
4. Pet bird and pet bat: found in droppings from the
starling bird or bats
5. Caves shown on television: commonly found in
caves
6. Injured man on stretcher: can disseminate in
immunocompromised individuals
7. Coughing grandson: can cause pneumonia
8. Specks of chocolate on tongue: can cause palatal
and tongue ulcers
9. Dalmatian dog with liver-shaped spot: can cause
hepatosplenomegaly
10. Granny: can form granulomas Photo Credit: Histoplasmosis by Yal Rosen via Flickr.com

11. Bird cage with bird food: microscopy may reveal Figure 5.3.2 - Histoplasma capsulatum yeast in
macrophages filled with Histoplasma yeast macrophages
12. Gems on the counter near the urinating man:
diagnosed by detecting antigens in the urine or
serum
167

Photo Credit: Mold (left): Bob Blaylock [CC BY-SA 3.0 (https://creative commons.org/licenses/by-sa/3.0)] Yeast (right): Microrao [Public domain]
Figure 5.3.3 - Mold (left) and yeast (right)

REVIEW QUESTIONS ?
1. A 24-year-old female presents to the physician • Correct answer: C
due to a cough and chest pain that began two • Findings suggestive of Histoplasma
days ago. She works at a pet store cleaning capsulatum:
bird cages. She also has smoked a pack of • A cough and chest pain which is
cigarettes per day for the past 8 years. Her suggestive of pneumonia
temperature is 38.8°C (101.8°F). A chest • She cleans bird cages
x-ray reveals parenchymal infiltrates and • Histopathological examination has
hilar lymphadenopathy. Histopathological revealed the presence of yeast-phase
examination of a mediastinal lymph node organisms inside of macrophage
reveals the presence of yeast-phase organisms • Histoplasma capsulatum also exhibits
inside of macrophages. What other pathological granuloma formation (an organized
finding is most likely present in this individual? collection of macrophages)
• A is a reference to hepatomegaly which
A. Enlarged hepatocytes
can be seen in patients with disseminated
B. Septate hyphae that branch at acute angles
C. An organized collection of macrophages histoplasmosis (this patient has a relatively
D. Spherules filled with endospores healthy immune system so it’s unlikely that
the fungus has disseminated)
• B is a reference to Aspergillus which is a
monomorphic fungus and forms septate
hyphae that branch at acute angles
• D is a reference to coccidioidomycosis
168

Section 2 - Blastomyces dermatitidis

1. Big blast: Blastomyces dermatitidis 8. Gray and black burn spots: can cause verrucous
2. Butterfly patch for the butterfly squad: dimorphic skin lesions in immunocompromised patients (may
(mold at 20°C and yeast at 37°C) appear similar to squamous cell carcinoma)
3. GPS showing central and eastern parts of the U.S.: 9. Large bump on the stretcher: can cause
endemic to the central and eastern parts of the U.S. granulomatous nodule formation in
4. Big lake: commonly seen in the Great Lakes area immunocompromised patients
5. Soldier coughing on smoke from the blast: 10. Two grenades right next to each other: exhibits
causes flu-like symptoms and pneumonia in broad-based budding
immunocompromised individuals 11. Soldier with a tattoo of his granny: associated with
6. White flag with burn pattern resembling the lungs: granuloma formation
may cause chronic inflammation of the lungs 12. Granny wearing a shawl with the letter “A” on it:
(chronic pneumonia) treat local infections with azole medications (i.e.
7. Injured man on a stretcher with bones protruding itraconazole)
from skin: can disseminate to the bone and skin in 13. Amphibian on the soldier’s gun: treat systemic
immunocompromised patients infections with amphotericin B
169

Photo Credit: Kelly Nelson (Photographer) [Public domain]


Figure 5.3.5 - Squamous cell carcinoma

Photo Credit: Alborz Fallah [CC BY-SA 3.0 (https://creativecommons.org/


licenses/by-sa/3.0)]
Figure 5.3.4 - Verrucous skin lesion
170

Photo Credit: [Public domain]


Figure 5.3.6 - Granulomatous nodule

Photo Credit: Medmyco at English Wikipedia [CC0]


Figure 5.3.7 - Broad-based budding by Blastomyces
dermatitidis
171

REVIEW QUESTIONS ?
1. A 23-year-old female with a history of HIV • Correct answer: A
presents to the physician due to a red sore on • This patient has an infection caused by
her left arm. She states that she first noticed it Blastomyces dermatitidis
3 days ago but that it has been getting worse. • The chronic cough is alluding to
She has also had a cough for the past several pulmonary blastomycosis which is likely
months but didn’t think much of it. She is not around the time that the patient first
compliant with antiretroviral therapy. Physical became infected
examination reveals a violet-colored verrucous • Her history of HIV  is likely what allowed
lesion with irregular borders. A biopsy is the infection to disseminate to the skin
obtained and shown below. • She’s presenting with a disseminated
Blastomyces skin infection resulting
in the characteristic violet-colored
verrucous skin lesion
• This, along with the pathological
specimen showing broad-based
budding, is enough information to
conclude that the causal organism is
Blastomyces dermatitidis
• B is incorrect because this is describing the
geographic location of coccidioidomycosis
which does not exhibit broad-based
budding as seen in the picture
Photo Credit: CDC [Public domain] • C is incorrect because this is describing the
Which of the following is true regarding the transmission of histoplasmosis which does
causal organism? not exhibit broad-based budding
• D is incorrect because this is a reference to
A. It may also cause osteomyelitis the morphology of Aspergillus fumigatus
B. It is endemic to the Southwestern U.S. which does not exhibit broad-based
C. It is transmitted through bat droppings budding
D. Microscopy may also reveal septate hyphae
that branch at 45 degree angles
172

Section 3 - Coccidioides immitis

1. Cocking a gun: Coccidioides immitis 13. Hammock that looks like a stretcher:
2. Picture of the Southwestern U.S. inscribed on gun: can disseminate to the skin and bone in
endemic to the Southwestern U.S. immunocompromised individuals
3. Butterfly on balloon: dimorphic 14. Man on hammock wearing a hat: can cause
4. Butterfly balloon is spherical-shaped: a mold form meningitis in immunocompromised individuals
at cold temperatures and a spherule (not yeast) at 15. Granny wearing a shawl with the letter “A” on it:
warm temperatures treat local infections with azole medications (eg,
5. Popped balloon releasing confetti and butterfly fluconazole, itraconazole)
balloon is larger than red balloon: the spherules 16. Desert amphibian underneath the hammock: treat
are much larger than RBCs and are filled with disseminated infections with amphotericin B
endospores
6. Silver bullet: can be identified using the silver stain
7. Bullets stirring up dust: transmission occurs via
inhalation of spores that are released from the soil
(eg, earthquakes, archeological excavations)
8. Shooting target: can cause erythema multiforme
(target-like lesions)
9. Dust causing the granny to cough: can cause
pneumonia
10. Granny here to support her son: associated with
granuloma formation
11. Red and injured elbow from bullet fragment: can
cause arthralgias
12. Remnants of ruptured balloons on the desert
ground: can cause erythema nodosum (desert Photo Credit: CDC [Public domain]

bumps) Figure 5.3.8 - Endospore of Coccidioides immitis


173

REVIEW QUESTIONS ?
1. A 33-year-old female presents to her physician
due to shortness of breath, chest pain, and a
cough that has steadily worsened over the past
week. Her temperature is 38.7°C (101.7°F).
Physical examination reveals crackles at the
base of the lungs bilaterally. A pathological
specimen is obtained and shown below.

Photo Credit: CDC [Public domain]


Figure 5.3.9 - Coccidioides using the methenamine
silver stain

Photo Credit: CDC [Public domain]

Based on the information above, where does


this patient most likely live?
A. Ohio
B. Michigan
C. Nevada
D. Brasil

• Correct answer: C
• This patient has an infection caused by
Coccidioides immitis
• She has pneumonia (shortness of
breath, chest pain, a cough, a fever
of 38.7°C, and crackles heard on
Photo Credit: Grook Da Oger [CC BY-SA 3.0 (https://creativecommons.org/ examination)
licenses/by-sa/3.0)]
• The pathological specimen reveals a
Figure 5.3.10 - Erythema multiforme
spherule which is unique to Coccidioides
immitis
• A is the endemic location of histoplasmosis
• B is the endemic location of blastomycosis
• D is the location of paracoccidioidomycosis

Photo Credit: Medicalpal [CC BY-SA 4.0 (https://creativecommons.org/


licenses/by-sa/4.0)]
Figure 5.3.11 - Erythema nodosum
174

Section 4 - Paracoccidioides brasiliensis

1. Brazilian embassy: Paracoccidioides brasiliensis 7. Injured civilian on a stretcher: immunocompromised


2. Captain South America’s wheel-shaped shield: individuals are at risk of developing disseminated
budding yeast may exhibit a “captain wheel” disease
appearance 8. Liver-shaped spot on dalmatian service dog: causes
3. Image of South America on the shirt of Captain hepatosplenomegaly
South America: endemic to Latin America 9. Beaded net full of medical supplies: causes
4. Butterfly-like aliens attacking: dimorphic (mold at lymphadenopathy
20°C and yeast at 37°C) 10. Granny wearing a shawl with an “A” on it: treat local
5. Granny hostage: associated with granuloma infections with azole medications (eg, itraconazole,
formation voriconazole)
6. Granny coughing: causes a pulmonary infection 11. Amphibians jumping near the civilian on
(often asymptomatic) the stretcher: treat systemic infections with
amphotericin B
175

REVIEW QUESTIONS ?
1. A 27-year-old male with a history of HIV
presents to the physician due to swollen bumps
in his neck, armpits, and groin that began 2
days ago. He states that he just returned home
from a trip to South America and is worried he
may have contracted an illness while traveling.
His temperature is 38.9°C (102.0°F). Physical
examination reveals lymphadenopathy and
hepatosplenomegaly. Microscopic visualization
of which of the following will most likely be
Photo Credit: CDC/Dr. Lucille K. Georg [Public domain]
present in a tissue sample taken from this
Figure 5.3.12 - Budding cells of Paracoccidioides
patient?  
brasiliensis
A. Broad-based budding
B. Spherules filled with endospores
C. Macrophages filled with fungi
D. Budding yeast protruding from a ring-like
structure

• Correct answer: D
• The patient has an infection caused by
Paracoccidioides brasiliensis
• He has a compromised immune system
(eg, HIV)
• He recently traveled to South America
• He is presenting with lymphadenopathy
and hepatosplenomegaly
• Paracoccidioides brasiliensis exhibits
a microscopic “captain’s wheel”
appearance which may be described as
budding yeast protruding from a ring-
like structure
• A is incorrect because this is describing
Blastomyces but this is not endemic to
South America
• B is incorrect because this is describing
coccidioidomycosis which is endemic to the
Southwestern U.S.
• C is incorrect because this is describing
histoplasmosis which is endemic to the
Mississippi and Ohio River valleys
176

Section 5 - Microsporum, Trichophyton, and Epidermophyton

1. Dermatologists fighting: dermatophytes 14. Athlete with infected foot: Tinea pedis (athlete’s
2. Image of the epidermis: Epidermophyton foot)
3. Microscope: Microsporum 15. Athlete’s moccasins on nearby table: moccasin-type
4. A child’s “trike”: Trichophyton tinea pedis
5. Dermatologist receiving a “K.O.” punch causing him 16. Fruit splattered on athlete’s foot: vesiculobullous
to knock over a little plant with branches and fruit: tinea pedis
a KOH preparation will reveal branching septate 17. Intertwined fingers with champion rings: interdigital
hyphae (and possibly spores) tinea pedis
6. A patient holding a baseball cap: Tinea capitis 18. Worker nailing up decorations: Tinea unguium
7. Bald patches on baseball coach: Tinea capitis is is a fungus that causes infection of the nail
associated with alopecia (onychomycosis)
8. Beaded net holding baseballs: Tinea capitis is 19. Image of little girl’s bathing suit getting tugged on
associated with lymphadenopathy by a dog: Tinea cruris (jock itch) is a dermatophyte
9. Baseball coach’s son reading a book about dragons: infection that occurs in the inguinal area
Tinea capitis is associated with scaling 20. Dermatologist wearing a shawl with an “A” on it:
10. Marine Corps tattoo: Tinea corporis most dermatophyte infections should be treated
11. Marine Corps tattoo on the torso: Tinea corporis with topical azoles
lesions are commonly seen on the torso 21. Worker applying grease to the window: griseofulvin
12. Red Marine Corps tattoo with a white center: the can be used to treat dermatophyte onychomycosis
rash is characterized by an erythematous scaly ring 22. Worker wearing a turban: terbinafine can be used to
with a central area of clearing treat dermatophyte onychomycosis
13. Marine Corps patient holding his two pets: Tinea
corporis can be acquired by infected animals (eg,
pets, farm animals)
177

Photo Credit: CDC/Dr. Lucille K. George [Public domain]


Figure 5.3.13 - Hyphae and spores of a
dermatophyte

Photo Credit: Teresa Trimm [Flickr]


Figure 5.3.16 - Tinea pedis

Photo Credit: myself [CC BY-SA 4.0 (https://creativecommons.org/licenses/


by-sa/4.0)]
Figure 5.3.14 - Tinea capitis

Photo Credit: Soodleksw [Public domain]


Figure 5.3.17 - Vesicular athlete’s foot
Photo Credit: Corina G. [Public domain]
Figure 5.3.15 - Tinea corporis
178

Photo Credit: Robertgascoin [CC BY-SA 3.0 (https://creativecommons.org/


licenses/by-sa/3.0)]
Photo Credit: User Falloonb on en.wikipedia [Public domain]
Figure 5.3.20 - Tinea cruris
Figure 5.3.18 - Interdigital tinea pedis

Photo Credit: Medical334 [CC BY-SA 4.0 (https://creativecommons.org/


licenses/by-sa/4.0)]
Figure 5.3.19 - Onychomycosis
179

REVIEW QUESTIONS ?
1. A 17-year-old male presents to the physician
due to a pruritic rash on his leg that he noticed
yesterday. He is on the high school football
team and states that he sweats excessively
during practices. Physical examination reveals
an erythematous scaling patch on the medial
aspect of the left inner thigh. Scales are scraped
from the lesion and a potassium hydroxide
(KOH) preparation reveals branching septate
hyphae. This patient’s condition is best treated
with which of the following medications?
A. Selenium sulfide
B. Amphotericin B
C. Fluconazole
D. Nystatin

• Correct answer: C
• This patient has tinea cruris (jock itch)
• An erythematous scaling patch on the
medial aspect of the left inner thigh
• A pathological sample revealed
branching septate hyphae from a KOH
prep
• The treatment for jock itch is
fluconazole
• A is incorrect because this is the treatment
for malassezia furfur
• B is the treatment for several severe fungal
infections such as systemic histoplasmosis
and blastomycosis
• D is true of infections caused by
Candida such as oral thrush or vaingal
candidiasis but this is not effective against
dermatophyte infections
180

Section 6 - Malassezia furfur

1. Malodorous and furry armpits: Malassezia furfur 6. Black and white, checkered shirt: the rash is
2. Armpits: causes pityriasis versicolor characterized by areas of hyperpigmentation and
3. “Summer Special” sign and margarita: more hypopigmentation (pink patches may also be seen)
common in the summer when it’s hot and humid 7. Spaghetti and meatballs: may exhibit a “spaghetti
(the yeast is converted into the pathogenic mycelial and meatballs” appearance on microscopy
form under these conditions) 8. “FOR SALE” on the menu: treat with topical
4. Greasy hands squeezing lemon acid onto a melon: selenium sulfide
the degradation of lipids produces acids that inhibit 9. A shawl with the letter “A” on it: treat extensive
melanin synthesis disease with oral azole medications
5. Scratching her head in awe: results in pruritic lesions
181

REVIEW QUESTIONS ?
1. A 24-year-old female presents to the physician
due to a pruritic rash which she noticed several
weeks ago. Physical examination reveals
a rash on the patient’s trunk with areas of
hypopigmentation and hyperpigmentation. A
sample of tissue is obtained and a potassium
hydroxide (KOH) preparation reveals the
presence of both yeast cells and hyphae.
Which of the following risk factors is most likely
associated with this patient’s condition?
 
Photo Credit: Saracrosenau on Flickr.com [CC BY-SA 2.0 (https:// A. Chronic alcohol use
creativecommons.org/licenses/by-sa/2.0)] B. Poor hygiene
Figure 5.3.21 - Tinea versicolor skin lesions C. Hyperhidrosis
D. Recent exposure to antibiotics

• Correct answer: C
• This patient has a Malassezia furfur
infection
• A pruritic rash with areas
of hypopigmentation and
hyperpigmentation
• KOH prep has revealed both yeast cells
and hyphae is describing the “spaghetti
and meatballs” appearance of the
organism
• This infection is more common in the
Photo Credit: CDC/Dr. Lucille K. Georg [Public domain] summer when the weather is hot and
Figure 5.3.22 - Microscopic image of Malassezia humid or under similar conditions (eg,
furfur “Spaghetti and meatballs” hyperhidrosis leading to hot and humid
conditions)
• Under hot and humid conditions,
the yeast form is converted to the
pathogenic mycelial form which can
cause disease.
• A, B, and D are not known risk factors for
developing tinea versicolor
• A and B are red herrings and D is often
associated with Candida albicans but not
Malassezia furfur
182

Section 7 - Sporothrix schenckii

1. Prison shank: Sporothrix schenckii 6. Linear skin scrape: presents as erythematous lesions
2. Branch with thorns: lives on vegetation and is along draining lymphatics that later ulcerate
transmitted through thorn pricks 7. Injured prisoner on a stretcher: disseminated
3. Butterfly tattoo: dimorphic (mold at 20°C and yeast disease is possible in an immunocompromised host
at 37°C) 8. Pot plants: treat with potassium iodide
4. Cigar: exhibits cigar-shaped budding in the yeast 9. Shawl with an “A” on it: treat with azole medications
form (i.e. itraconazole)
5. Plants with branches and fruit: exhibits branching
hyphae with rosettes of conidia in the mold form
183

REVIEW QUESTIONS ?
1. A 27-year-old female presents to the physician
due to a rash on her leg. She states that she first
noticed the rash two days ago after working in
her garden. Physical examination reveals mildly
tender erythematous papules extending from
the ankle up to the knee. A tissue sample is
obtained and viewed under the microscope.
What morphological feature will most likely be
seen?
Photo Credit: CDC/Dr. Lucille K. Georg [Public domain] A. Branching hyphae with terminal conidia
Figure 5.3.23 - Sporothrix schenckii (yeast form) B. Elongated oval-shaped yeast cells
  C. Spherules filled with endospores
D. Budding yeast that resemble a “captain’s
wheel”

• Correct answer: B
• The patient has a Sporothrix schenckii
infection
• Developed a rash on her leg after
working in the garden (exposed to
thorns)
• Has mildly tender erythematous
papules from the ankle up to the knee
(lesions draining along lymphatic
Photo Credit: CDC/Dr. Libero Ajello [Public domain] channels)
Figure 5.3.24 - Sporothrix schenckii (mold form) • Elongated oval-shaped yeast cells is
describing cigar-shaped budding
• A is wrong because this is describing the
mold form of Sporothrix schenckii (the mold
form is seen outside of the human body but
not in the warmer environment within a
host)
• C is incorrect because this is describing the
morphology of coccidioidomycosis
• D is a reference to paracoccidioidomycosis
which is unlikely to cause lesions after
gardening

Photo Credit: CDC/Dr. Lucille K. Georg [Public domain]


Figure 5.3.25 - Sporothrix schenckii lesion along
draining lymphatic channels
184

Section 8 - Candida albicans

1. Canadian flag: Candida albicans 13. Nice boss with a picture of Satan on shirt: oral and
2. Building a snowman out of cotton: reproduces esophageal candida infections can be treated with
through budding nystatin
3. Butterfly: dimorphic 14. A girl with dyed beads and a container of pills
4. Fruit tree with branches near cold window with who has spilled cotton onto her crotch: causes
icicles: forms pseudohyphae and budding yeast at vulvovaginitis (especially in patients with diabetes
20°C or those who are taking antibiotics)
5. Drinking hot coffee through straw: germ tube 15. Cotton appearing like thick clumps of cottage
formation at 37°C cheese: thick “cottage cheese” discharge
6. Flowers: part of the normal flora (eg, mouth, skin, 16. Tub of acid with a pH in the range of 4.0 - 4.5: the
vagina, intestines) vaginal pH will be normal (4.0 - 4.5)
7. Cat: catalase positive 17. A shawl with an “A” on it: candida infections causing
8. Worker with Band-Aid is eating cotton balls on a vulvovaginitis can be treated with azoles
conveyor belt with a sign reading “Max rate = 200”: 18. Cotton spilled onto baby’s red diaper: can cause an
causes oral thrush in immunocompromised patients erythematous diaper rash in children
(an AIDS patient with a CD4 count < 200) 19. Little toy car: can cause endocarditis in IV drug users
9. Scraping cotton balls off the mat: the lesions in oral 20. Boss with a Canadian flag tattoo salivating over
thrush can be easily scraped off his food: associated with chronic mucocutaneous
10. Cotton balls traveling down the conveyor belt with candidiasis (an immunodeficiency)
a “Max rate = 200” sign: causes esophagitis in AIDS 21. Fish with scales: can cause osteomyelitis
patients with a CD4 count < 200 22. Tubes bursting off machine: patients receiving
11. A shawl with an “A” on it: oral and esophageal parenteral nutrition through a catheter may develop
candida infections can be treated with azoles candidemia
12. Bag of Funyuns: oral and esophageal candida 23. Wall blasted with a uniquely-shaped dent:
infections can be treated with echinocandins neutropenic patients are at an increased risk of
developing disseminated disease
185

24. Worker receiving a “K.O.” blast from the machine: a


KOH preparation can be used to identify candida
25. Pet amphibian jumping around machine:
amphotericin B can be used to treat systemic
infections
26. Girl with “A” shawl and bag of Funyuns near the
exploding machine: echinocandins followed by an
oral azole medication can also be used for systemic
infections

Photo Credit: Y tambe [CC BY-SA 3.0 (https://creativecommons.org/licenses/


by-sa/3.0)]
Figure 5.3.26 - Pseudohyphae and budding yeast

Photo Credit: CDC/Dr. Lucille K. Georg [Public domain]


Figure 5.3.27 - Germ tube formation
186

Photo Credit: Mikael Häggström [CC0]


Figure 5.3.30 - Vulvovaginitis caused by Candida
Photo Credit: James Heilman, MD [CC BY-SA 3.0 (https://creativecommons.
albicans
org/licenses/by-sa/3.0)]
Figure 5.3.28 - Oral thrush

Photo Credit: James Heilman, MD [CC BY-SA 3.0 (https://creativecommons.


org/licenses/by-sa/3.0)]
Figure 5.3.29 - Esophageal candidiasis
187

REVIEW QUESTIONS ?
1. A 33-year-old female is found to have chills
and a persistent fever 3 days after undergoing
extensive excision of metastatic breast cancer.
Her temperature is 38.7°C (101.7°F). Physical
examination reveals a well-placed central
venous catheter through which she is receiving
parenteral nutrition. Blood cultures are
obtained and reveal budding yeast. Which of
the following is true regarding the most likely
causal organism?
A. It will exhibit “germ tubes” at 20°C
B. It is part of the normal skin flora
C. It is catalase-negative
D. It is monomorphic

• Correct answer: B
• This patient has candidemia
• She is receiving parenteral nutrition
through a central venous catheter
• Blood cultures have shown budding
yeast
• Candida is part of the normal skin flora
• A is wrong because Candida exhibits germ
tubes at 37°C
• C is wrong because Candida is catalase-
positive
• D is wrong because Candida is dimorphic
188

Section 9 - Aspergillus fumigatus

1. Asparagus jungle: Aspergillus fumigatus 12. Funyuns: invasive disease can be treated with
2. Branches and chop mark: hyphae branch at 45 echinocandins
degree angles (V-shaped) and have septations 13. Crater with asparagus and tubers: can cause
3. Cat: catalase positive aspergillomas which form due to fungal colonization
4. Man on stretcher near blast: cause invasive disease of old lung cavities (eg, following a TB infection)
in immunocompromised patients 14. Red beet juice: aspergillomas may cause hemoptysis
5. 100 dollar bill Band-Aid: invasive disease seen in 15. Sweaty skinny guy: secondary TB presents with
AIDS patients with a CD4 count <100 night sweats and weight loss (patients with an
6. Commando grandpa: invasive disease may be seen aspergilloma have an absence of these symptoms)
in patients with neutrophil dysfunction (eg, chronic 16. Aflac life insurance guy with iconic duck: some
granulomatous disease) species produce aflatoxins
7. Hope ribbon covered in blood: invasive disease may 17. Duck eating nuts and grains in bowl: aflatoxins can
be seen in patients with leukemia be found in grains and nuts
8. Medic with lamp and cough: invasive pulmonary 18. Dog with liver shaped spot: aflatoxins are associated
disease may present with a fever and cough with hepatocellular carcinoma
9. Medic coughing up blood and holding chest: 19. Coughing soldiers with inhaler and bag of salt:
invasive pulmonary disease may present with chest allergic bronchopulmonary aspergillosis (ABPA) is a
pain and hemoptysis hypersensitivity response that occurs when fungus
10. “Vietnam this way” sign: invasive disease may colonizes the bronchial mucosa of asthmatics or CF
present as fungal rhinosinusitis (similar to Mucor patients
and Rhizopus spp.) 20. Whittling soldier: allergic bronchopulmonary
11. Women wearing “A” shawls: treat local and invasive aspergillosis (ABPA) may cause bronchiectasis
disease with azoles (eg, voriconazole) 21. Red circular debris: allergic bronchopulmonary
aspergillosis (ABPA) is associated with eosinophilia
189

REVIEW QUESTIONS ?
1. A 59-year-old male is brought to the emergency
department due to a cough and bloody sputum.
He states that he has had a cough for the
past few days, but only noticed the blood this
morning. He has a history of tuberculosis (TB)
that was successfully treated several years ago.
He denies recent night sweats, fevers, or weight
loss. Imaging is obtained and shown below.
This patient’s condition is most likely caused by
Photo Credit: CDC [Public domain] which of the following?
Figure 5.3.31 - Microscopic morphology of
Aspergillus fumigatus

Stockholm [CC BY-SA 4.0 (https://creativecommons.org/licenses/by-


sa/4.0)]

A. A hypersensitivity reaction
Photo Credit: Yale Rosen from USA [CC BY-SA 2.0 (https://creativecommons.
org/licenses/by-sa/2.0)] B. Reactivation of a latent organism
Figure 5.3.32 - Aspergilloma (gross image) C. Invasive fungal disease involving the
vasculature
D. Fungal colonization of a prior lesion
E. Pulmonary invasion by neoplastic cells

• Correct answer: D
• This patient has an aspergilloma
• Cough, hemoptysis (bloody sputum),
and a history of TB
• Imaging has shown an aspergilloma
• An aspergilloma is Aspergillus fumigatus
colonization of a prior cavitary TB lesion
• A is describing allergic bronchopulmonary
aspergillosis
• B is describing reactivation TB which
Photo Credit: Stockholm [CC BY-SA 4.0 (https://creativecommons.org/ presents with night sweats, fevers, and
licenses/by-sa/4.0)]
weight loss which this patient does not have
Figure 5.3.33 - Aspergilloma (imaging)
• C is describing invasive aspergillosis
(presents in severely neutropenic and
immunocompromised patients and presents
with a cough, hemoptysis, fevers, and chest
pain)
190

Section 10 - Cryptococcus neoformans

1. Crypt: Cryptococcus neoformans 11. “50-year-old male” with Band-Aid: causes disease in
2. Orbs on staff separated by a narrow stock: exhibits immunocompromised patients (AIDS patients with a
narrow-based budding CD4 count less than 50)
3. 5-10 micrometers: 5-10 micrometers 12. Cough: the primary site of infection is in the lungs
4. Ink drawing of angel: can be identified using (most patients are asymptomatic)
the India ink stain which will show a clear halo 13. Religious hat: can cause meningitis
surrounding the pathogen 14. Urine dripping on head: can cause encephalitis
5. Pigeon and droppings: found in soil and pigeon 15. Soap bubbles: imaging may reveal “soap bubble”
droppings lesions in the brain
6. Dust: acquired through inhalation 16. Flute and sai: treat with flucytosine
7. Pool of urine: urease positive 17. Shawl with letter “A”: treat cryptococcal meningitis
8. “Keep the Sabbath day holy”: can be grown on with azoles (fluconazole)
Sabouraud dextrose agar 18. Amphibian drawing: treat cryptococcal meningitis
9. Red sack and music: can be identified using the with amphotericin B
mucicarmine stain which reveals a red capsule
10. Latex gloves: the latex agglutination test can be
used to identify the polysaccharide capsular antigen
(more specific and sensitive than other tests)
191

REVIEW QUESTIONS ?
1. A 50-year-old female with a history of HIV is
brought to the emergency department by her
husband due to sudden onset confusion. He
states that this morning she got lost driving her
car and had difficulty remembering the year.
Her temperature is 38.8°C (101.8°F). Physical
examination reveals nuchal rigidity. A lumbar
puncture is performed and cerebrospinal fluid
is sent to the laboratory for further analysis. A
latex agglutination test is positive. This patient’s
Photo Credit: CDD/Dr. Leanor Haley [Public domain] neurological findings most likely occurred
Figure 5.3.34 - India ink stain following hematogenous dissemination from
which of the following regions?
A. Skin
B. Nares
C. Lungs
D. Ear canal
E. Urinary tract

• Correct answer: C
• This patient has meningitis caused by
Cryptococcus neoformans
• HIV, sudden onset confusion, a fever of
38.8°C, and a positive agglutination test
• The organism is acquired through
Photo Credit: CDD/Dr. William Kaplan [Public domain]
inhalation
Figure 5.3.35 - Sabouraud dextrose agar • Cryptococcus neoformans can cause
lung infections and this is the primary
site of infection before hematogenous
spread occurs
• Many bacterial pathogens can cause
meningitis and can disseminate from the
nares (B) or other locations (A, D, and E),
but these would not have a positive latex
agglutination test

Photo Credit: CDD/Dr. Edwin P. Ewing, Jr. [Public domain]


Figure 5.3.36 - Mucicarmine stain
192

Section 11 - Mucor & Rhizopus spp.

1. Rising zombie: Mucor & Rhizopus species 10. Witches hair: may cause cranial nerve damage
2. Inhaling smell from snails: transmitted via inhalation 11. Red lava cracks: proliferate in blood vessel walls
of spores 12. Cave with signs: cavernous sinus thrombosis
3. Sorcerer’s broad-angled stance: exhibit irregular, 13. “CRIPS” graffiti: can penetrate the cribriform plate
broad, and nonseptate hyphae that branch at wide 14. Flowers covered in lava: can disseminate to the
angles brain and cause rhinocerebral abscesses as well as
4. Dyed beads: diabetic ketoacidosis is a risk factor frontal lobe abscesses
5. Tombstones: neutropenia is a risk factor 15. Cutting out the oral mucosa: mucosal biopsy
6. Zombie with hope ribbon: leukemia is a risk factor confirms the diagnosis
7. Deformed face: can cause facial infections resulting 16. Scalpel: treatment includes surgical debridement of
in necrotic eschar infected tissue
8. Hand on head: may present with headache and 17. Shawl with letter “A”: treat with azoles
facial pain 18. Amphibians: treat with amphotericin B
9. Eye: there may be orbital involvement
193

Photo Credit: Mucormycosis by Yale Rosen via Flickr.com


Figure 5.3.37 - Microscopy of mucormycosis

REVIEW QUESTIONS ?
1. A 7-year-old girl who is undergoing • C is true of deficits involving the temporal
chemotherapy for acute lymphoblastic leukemia lobe, but the frontal lobe is more commonly
is brought to the emergency department due affected with these types of infections
to a fever and right eye pain. Her father states • D is true of deficits involving the parietal
that the pain began two days ago, but several lobe (location of the primary sensory
hours ago she was unable to close her right cortex)
eye. Physical examination reveals periorbital
tenderness, hyperacusis, and an inability to
close the right eye lid. This patient is most likely
at risk for developing which of the following?
A. Diabetic ketoacidosis
B. Paralysis of the left arm
C. Wernicke’s aphasia
D. Decreased sensation over the left foot

• Correct answer: B
• This patient has a facial nerve deficit caused
by Mucor & Rhizopus spp.
• She has leukemia, a fever, and eye pain
• The inability to close the right eye along
with hyperacusis is suggestive of a facial
nerve deficit
• Mucor & Rhizopus spp. can also cause
a frontal lobe abscess and the primary
motor cortex is located in the frontal
lobe → contralateral paralysis
• A is incorrect because a Mucor & Rhizopus
spp. infection doesn’t increase her risk of
diabetic ketoacidosis (DKA increases the risk
of an infection)
194

Section 12 - Pneumocystis jirovecii

1. Hero: Pneumocystis jiroveci 9. Water spraying axe: can be diagnosed by lung biopsy
2. Yeast beer: a yeast-like fungus or bronchoalveolar lavage
3. Plate: exhibits disc-shaped cysts that can be 10. 3-pronged chain: can be diagnosed with PCR
identified using the methenamine silver stain 11. Fiery arrow: diagnosed with a fluorescent antibody
4. Happy thugs: most infections are asymptomatic stain
5. Man on stretcher: most patients who develop 12. Crystal meth: first-line treatment and prophylaxis is
symptoms are immunocompromised TMP-SMX (give when CD4 count drops <200)
6. Bag of Band-Aids and “Max Occupancy 200”: AIDS 13. Deputy: an alternative prophylactic medication is
patients with a CD4 count <200 are at an increased dapsone
risk of disease (start prophylactic treatment at this 14. Pentagonal sign: an alternative prophylactic
CD4 count) medication is pentamidine
7. Axe and broken glass: causes interstitial pneumonia 15. Ice-cream cone on toe: an alternative prophylactic
with ground glass opacities seen on chest imaging medication is atovaquone
8. Red spots on lungs: imaging may reveal
pneumatoceles
195

Photo Credit: CDC/Dr. Russell K. Brynes [Public domain] Photo Credit: CDC/Dr. Govinda S. Visvesvara [Public domain]
Figure 5.3.38 - Silver stain of Pneumocystis jirovecii Figure 5.3.41 - Fluorescent antibody stain
cysts

Photo Credit: Carolyn M. Allen, Hamdan H. AL-Jahdali, Klaus L. Irion, Sarah Al Ghanem, Alaa Gouda, and Ali Nawaz Khan [CC BY 4.0 (https://creativecommons.
org/licenses/by/4.0)]
Figure 5.3.39 - Chest x-rays of Pneumocystis pneumonia with “ground glass” opacities

Photo Credit: Carolyn M. Allen, Hamdan H. AL-Jahdali, Klaus L. Irion,


Sarah Al Ghanem, Alaa Gouda, and Ali Nawaz Khan [CC BY 4.0 (https://
creativecommons.org/licenses/by/4.0)]
Figure 5.3.40 - Pneumatoceles
196

REVIEW QUESTIONS ?
1. A 27-year-old male comes to the emergency • Correct answer: C
department due to a cough and shortness • This patient has a Pneumocystis jirovecii
of breath. He states that the cough has been infection
present for one week, but the shortness of • He has a cough, shortness of breath,
breath began earlier today. He has a history and a history of HIV
of HIV and has been non-compliant with • The silver stain shows cysts that
antiretroviral therapy. A bronchoalveolar lavage resemble yeast cells
is performed and a silver stain of the sample is • Pneumocystis jirovecii can cause
shown below. pneumatoceles (cystic air-filled lesions
in the lungs)
• A is wrong because TMP-SMX prophylaxis is
given when the CD4 count drops to below <
200
• B is wrong because most infections
asymptomatic and do not actually cause
pneumonia
• D is wrong because this is a medication used
for malaria

Photo Credit: CDC/ Dr. Edwin P. Ewing, Jr. [Public domain]

Which of the following is most like true


regarding this patient’s condition?
A. TMP-SMX prophylaxis should be started
when the CD4 count drops to < 500 cells/
mm 3

B. Most infections by the causal organism


result in interstitial pneumonia
C. Cystic air-filled lesions in the lungs may be
seen on imaging
D. An alternative prophylactic medication is
chloroquine
197

PARASITES

Figure 5.4.1 - Parasites overview figure


198

Section 1 - Giardia lamblia

1. “Guard A Lamb”: Giardia lamblia 6. Brown river: causes diarrhea


2. Cysts in pool: cysts found in stool 7. Camper: common in campers and hikers
3. Drinking poo water: fecal-oral transmission 8. Stink lines: foul-smelling diarrhea
4. Parasites in water: cysts develop into trophozoites 9. Man holding stomach: causes abdominal pain
5. Man catching parasites: diagnose with stool 10. Bloated man: causes bloating
examination test (O and P test) which will reveal 11. Protein drink: Giardia lamblia is a protozoan
cysts or trophozoites 12. Metro: treat with metronidazole
199

REVIEW QUESTIONS ?
1. A 23-year-old female presents to the clinic
with abdominal pain for two weeks. She
explains that her symptoms began shortly
after returning from a short camping trip near
a lake. She denies intentionally drinking any
of the lake water, but admits she accidentally
swallowed quite a bit one time while swimming.
The physician orders an ova and parasite
examination (O & P) of the stool. The laboratory
confirms the presence of several pear-shaped,
Photo Credit: Public domain flagellated trophozoites. Based on the likely
Figure 5.4.2 - Giardia lamblia cyst pathogen, which of the following is the patient
most likely to experience at some point during
her infection?
A. A short period of vomiting
B. Blood in the stool
C. Repeated bouts of loose stools
D. Painful headaches

• Correct answer: C
• The patient likely has giardiasis based
on the following:
• Abdominal pain
• Recent camping
• Drinking lake water
• O & P revealing pear-shaped
flagellated trophozoites
• Giardiasis is most likely to cause
Photo Credit: schmidty4112 via Flickr diarrhea
Figure 5.4.3 - Giardia lamblia trophozoite • Choice A is incorrect because vomiting is
much less likely than diarrhea
• Choice B is incorrect because giardiasis does
not cause intestinal bleeding
• Choice D is incorrect because giardiasis is
not known to cause headaches
200

Section 2 - Entamoeba histolytica

1. “Enter My History”: Entamoeba histolytica 9. Ray’s descending on cows: diagnose liver abscesses
2. Red people inside monster: red blood cells seen in with CT scan
trophozoite cytoplasm (erythrophagocytosis) 10. Gems in blood and stool vials: diagnose via antigens
3. Protein shake: protozoa in stool or blood
4. Water supply drops eggs on poop: fecal-oral 11. Cyst on spaceship: diagnose via stool examination
transmission (O and P) which will reveal cysts
5. Monster footprints ulcerating the ground: causes 12. Metro: treat with metronidazole
intestinal ulcers 13. Idol queen: treat with iodoquinol
6. Hot lava river: causes bloody diarrhea 14. Pair of mice: treat with paromomycin
7. Stream reaching liver-spotted cows: trophozoites 15. Dropping Boiling water on monster: prevent
travel up the portal vein to infect the liver infection by boiling water before drinking
8. Melting cow: trophozoites cause liver abscesses
201

Photo Credit: CDC [Public domain]


Figure 5.4.4 - Red blood cells within Entamoeba
cytoplasm (erythrophagocytosis)

Photo Credit: CDC [Public domain]


Figure 5.4.5 - Entamoeba egg in stool sample

REVIEW QUESTIONS ?
1. A 22-year-old female presents to her family • Correct answer: B
physician for a fever and red diarrhea that • The patient likely has an infection with
began one week ago. She recently returned Entamoeba histolytica based on the
from a 3-week humanitarian trip to Kenya. She following:
endorses drinking water that did not undergo • Bloody diarrhea
filtering, heating or any other measures to • Drinking unclean water
ensure water cleanliness. Physical examination • RUQ pain
is significant for right upper quadrant abdominal • O & P micrograph revealing
tenderness. An ova and parasite study of her a protozoan containing RBCs
stool is examined. A micrograph of the test is (erythrophagocytosis)
shown below. Which of the following is true • Entamoeba histolytica causes intestinal
about her infection? ulceration (damage to the entire
mucosa)
• Choice A is incorrect because Entamoeba
histolytica infects the liver, not the biliary
tree
• Choice C is incorrect because antigens can
be found in the stool or the serum
Photo Credit: CDC/Dr. George Healy [Public domain] • Choice D is incorrect because her RUQ pain
may be caused by hepatic abscess, making
A. Her right upper quadrant pain is due to
infection of the biliary tree abdominal CT scan necessary
B. Colonoscopy would reveal areas of damage
to the entire mucosal layer
C. Parasitic antigens will not be found in the
patient’s serum
D. An abdominal CT scan does not have a
diagnostic role this late in her workup
202

Section 3 - Cryptosporidium

1. Crypts: Cryptosporidium 9. Vomiting man: infection can cause vomiting


2. Hardened crypts releasing spores: cysts release 10. River: infection can cause diarrhea
sporozoites 11. Protein drink: protozoa
3. Growing appendages: sporozoites invade and 12. Scientist scooping dirt at river’s edge: diagnose with
replicate within the gastrointestinal epithelium biopsy of intestinal epithelium
4. Infected man entering crypt and saying, “Uh-oh”: 13. River gems: diagnose with stool antigen testing
new oocytes form following replication 14. Exploding acid: diagnose with acid fast stain from
5. Hard coffin flowing down river: new oocysts are stool sample
shed in the stool 15. Nydus worm: treat with nitazoxanide
6. Injured man in stretcher: immunocompromised 16. Pair of mice: can treat with paromomycin
patients demonstrate symptoms 17. Steel grate: prevent infection by filtering water
7. Band-aid on injured man: AIDS patients at risk
8. 100 mph: AIDS patients susceptible when CD4
counts fall below 100
203

REVIEW QUESTIONS ?
1. A 29-year-old man with stabilized and
medicated HIV presents for a routine follow-up
visit. The patient (patient A) endorses feeling in
good health and does not have any concerns.
Blood tests were ordered, revealing a CD4+
T-cell count of 1,100 cells/mm3, unchanged
from the previous visit. The physician also
treats the patient’s life partner (patient B) who
also has AIDS. Last week, the physician treated
patient B for a Cryptosporidium infection.
Patient A and patient B both drink from the
same water source. Regarding patient A, which
of the following is most likely true?
A. He is likely to experience diarrhea soon
B. Prophylactic nitazoxanide is warranted
Photo Credit: CDC [Public domain] C. Unprotected intercourse will increase
Figure 5.4.6 - Cryptosporidium oocysts with acid- Cryptosporidium risk
fast staining D. He may have ingested oocysts from the
same water source

• Correct answer: D
• Patient A likely drinks from the same
water source as patient B, making
ingestion of oocysts possible
• Choice A is incorrect because patient A is
not immunocompromised
• Choice B is incorrect because nitazoxanide
is not warranted in immunocompetent
patients
• Choice C is incorrect because
Cryptosporidium is not sexually transmitted
204

Section 4 - Toxoplasma gondii

1. Toxic symbol: Toxoplasma gondii 11. 100 dollar bill: ring-enhancing lesions occur when
2. Feline stepping in feces: transmission via cat feces CD4+ cells are < 100.
3. Examining fresh blood: diagnosed using serology 12. Pulling on collar: causes pharyngitis in
4. Hero eating cat’s encysted meat: transmitted via immunocompetent patients
consumption of cysts in meat 13. Heat lamps: causes fever in immunocompetent
5. Torch: TORCHES infection patients
6. Crossing force field: can cross the placenta 14. Tired from running: causes fatigue in
7. Flames reflecting off of glasses: causes immunocompetent patients
chorioretinitis 15. Arrows that look the same: no heterophile
8. Milk on feline’s head: can cause intracranial antibodies
calcifications 16. Broken spotlight: monospot negative
9. Blocked hose: can cause hydrocephalus 17. Pirate meth: treat with pyrimethamine
10. Ringed hand on brain of patient with band-aid: 18. Sulfur dice: treat with sulfadiazine
causes ring-enhancing lesions in AIDS patients
205

REVIEW QUESTIONS ?
1. A 24-year-old G2P1 female presents at 17-weeks
gestation with a fever, sore throat and fatigue.
She describes her fatigue as different than
the fatigue she experienced with her previous
pregnancy. She feels healthy otherwise and
does not have a history of HIV exposure. Blood
work is drawn and sent to the laboratory for
analysis. Test results reveal antibodies produced
against a parasite often spread by cysts found in
cat feces. Which of the following is true about
Photo Credit: Aimun AB Jamjoo, Abrar R. Waliuddin, and Abdulhakim B. her infection?
Jamjoom [CC BY 2.5 (https://creativecommons.org/licenses/by/2.5/)]
A. Her blood would agglutinate if exposed to
sheep serum
B. The parasite may cause retinal inflammation
of the fetus
C. She is at risk for the development of brain
lesions
D. Sulfadiazine and pyrimethamine should be
withheld

• Correct answer: B
• The patient likely has a Toxoplasma
gondii infection based on the following:
• Parasitic cysts often spread by cat
feces
• Fever, pharyngitis, fatigue
• Toxoplasma gondii can cause fetal
chorioretinitis
• Choice A is incorrect. Although
toxoplasmosis causes symptoms similar
to mononucleosis (EBV), it would be
heterophile (monospot) negative
• Choice C is incorrect because ring-enhancing
brain lesions only occur in AIDS patients
when the CD4+ T-cell count falls below 100
cells/mm3
• Choice D is incorrect because sulfadiazine
and pyrimethamine should be administered
to prevent congenital toxoplasmosis
206

Section 5 - Naegleria fowleri

1. Niagara falls: Naegleria fowleri 6. Zombie-like victims: causes neck stiffness and
2. Protein shake shop: protozoa vomiting
3. Amoeba: exists in amoeba form only 7. Dead victims: CNS infection is lethal
4. Amoeba attacking nose: amoebas travel through the 8. Amphibians on dead bodies: Amphotericin B rarely
cribriform plate to infect the CNS prevents death
5. Amoeba attacking head: causes
meningoencephalitis
207

REVIEW QUESTIONS ?
1. A 12-year-old boy presents to the physician
due to recent neck pain. His mother is present
and states that soon after their recent family
vacation he began acting strange. Today his
temperature is 38.9°C (102.1°F). The physician
appreciates decreased range of motion of the
cervical spine. A lumbar puncture is performed
and reveals several amoebas. Which of the
following is most likely true of the patient’s
condition?
A. The patient was swimming in the ocean
during the vacation
B. The infectious organism is caused by a
trematode
C. Cysts will not be found in the cerebrospinal
fluid
D. Early administration of amphotericin B will
likely prevent death

• Correct answer: C
• The patient likely has Naegleria-related
meningoencephalitis based on the
following:
• Pain and decreased range of motion
in the neck
• Strange behavior
• Amoebas present in CSF
• Naegleria fowleri exists only in the
amoeba form and would not produce
cysts
• Choice A is incorrect because Naegleria
fowleri exists in freshwater only, not
saltwater as found in the ocean
• Choice B is incorrect because Naegleria
fowleri is a protozoa, not a trematode
• Choice D is incorrect because amphotericin
B is unlikely to prevent death, although it
should still be administered
208

Section 6 - Trypanosoma brucei, gambiense, and rhodesiense

1. African theme: African trypanosomiasis 10. Net with knots: causes enlarged lymph nodes
2. Road: Trypanosoma rhodesiense 11. Comb on knocked out bad guy: leads to coma
3. Gambling guards: Trypanosoma gambiense 12. Pit with bones: leads to death
4. Bruce Lee: Trypanosoma brucei 13. Guards with hats: CNS penetration
5. Teeny flies: transmitted through tsetse flies 14. Tripping goats: trypomastigotes
6. Painful wounds on victims: painful and red ulcers at 15. Goat head in pool of blood: diagnose by visualizing
the site of bites trypomastigotes on peripheral blood smear
7. Heat lamp with varied assortment of gems: 16. Syrup man: treat with suramin
antigenic variation causes relapsing fevers for 17. Mellow yellow cans rolling: treat with melarsoprol
months 18. Protein drink: trypanosomes are protozoa
8. Bad guy holding head: causes headaches
9. Garbled radio sounds for slurred speech: causes
slurred speech
209

REVIEW QUESTIONS ?
1. A 15-year-old boy presents to a humanitarian
outreach clinic in South Africa. His mother is
speaking for him since the patient is poorly
responsive. The mother states that she saw
three red unexplained wounds on his arms
one week ago and did not think much of it at
the time. He has had a fever for three days
and recently complained of being really tired.
This morning he was barely arousable. On
physical exam, the patient is somnolent. The
three red wounds are ulcerated and presumed
Photo Credit: CDC/Dr. Myron G. Schultz [Public domain]
to be bite marks. The physician is concerned
Figure 5.4.7 - Trypomastigotes on peripheral blood about a protozoan parasite penetrating the
smear patient’s central nervous system. The patient is
admitted to a nearby hospital and administered
melarsoprol. Which of the following is true
regarding the most likely causal organism?
A. Diagnosis is confirmed by the presence of
cysts on a blood smear
B. Fever cessation indicates resolution of the
disease
C. Mosquito eradication will not prevent
disease transmission
D. The red wounds indicative of bite mark
were likely painless

• Correct answer: C
• The patient likely has African sleeping
sickness (African trypanosomiasis)
based on the following:
• Lives in Africa
• Wounds consistent with “bug” bites
• Somnolence
• Treatment with melarsoprol
• African trypanosomiasis is transmitted
by Tsetse flies, not mosquitoes
• Choice A is incorrect because a blood smear
would reveal trypomastigotes
• Choice B is incorrect because fevers can
return due to high degree of antigenic
variation of the parasite
• Choice D is incorrect because tsetse fly bites
cause painful ulcers
210

Section 7 - Malaria

1. Mole area: Malaria (infection from Plasmodium 18. Damaged lung test machine: P. falciparum-related
spp.) clogging of vessels causes lung damage.
2. Protein shakes: protozoa 19. Startled man on EEG: P. falciparum-related clogging
3. Five pointed star on AXE can: Plasmodium vivax of vessels can damage the brain resulting in seizures
4. Oval opening on sleeveless shirt: Plasmodium ovale and coma
5. Clorox cleaner: treat P. vivax and ovale with 20. Pregnant woman spills sugar: P. falciparum can
chloroquine cause severe hypoglycemia in pregnant women
6. Hypnotist with Dalmation: P. vivax and ovale have 21. Clorox cleaner helping out: treat P. falciparum
dormant hypnozoites in the liver infections with chloroquine
7. Clorox cleaner knocked down by Dalmation: 22. Helpful artists: treat serious P. falciparum infections
chloroquine cannot eliminate the liver hypnozoites with artemisinins
of P. vivax and P. ovale 23. Queen with a “9” on her crown: treat serious P.
8. Prom queen: treat hypnozoites with primaquine falciparum infections with quinine (quinidine)
9. Sweaty guy: malaria can cause sweating 24. Clorox cleaners all over scene: chloroquine
10. Heat lamps: malaria can cause fever monotherapy is the treatment of choice unless the
11. Shaking weight lifter: malaria can cause shaking patient has a serious P. falciparum infection, a P.
chills vivax/ovale infection, or has a chloroquine-resistant
12. Spleen shaped welt: malaria can cause strain
splenomegaly 25. Attacking Iguanas and spilled ice cream: administer
13. Popped red balloon: malaria can cause anemia proguanil and atovaquone for chloroquine-resistant
14. False pair: Plasmodium falciparum strains
15. 1 out of 3 red balloons popped: P. falciparum infects 26. Meatloaf stand: mefloquine may be administered
1/3 of the patients red blood cells for chloroquine-resistant strains
16. Clogged vent: red blood cells killed by P. falciparum 27. Mosquitoes: malaria is transmitted by Anopheles
can clog small vessels mosquitoes
17. Spilled kidney beans: P. falciparum-related clogging 28. Knocked over trough with red Gatorade: diagnose
of vessels causes kidney damage by visualizing ring-form trophozoites within red
blood cells
211

29. Two dogs: red blood cells infected by P. vivax/ovale 31. Irregular and explosive behavior: red blood
spp. burst every 2 days (48 hours) cells infected by P. falciparum have an irregular
30. “3”-shaped couch in fancy ‘Mole Ariae”: red blood unpredictable bursting pattern
cells infected by P. malariae species burst every 3
days (72 hours)

Figure 5.4.8 - Malaria Life Cycle


212

I. Red blood cells will burst and release merozoites


into the bloodstream → immunologic response
(fever and chills)
A. Different species burst at certain intervals
1. P. vivax/ovale burst every 48 hours
2. P. malariae burst every 72 hours
3. P. falciparum burst irregularly

Photo Credit: MichaelZahniser via Wikimedia Commons [Public domain]


Figure 5.4.9 - Plasmodium trophozoite ring form
within red blood cells
213

REVIEW QUESTIONS ?
1. An 18-year-old otherwise healthy male • Correct answer: C
complains of recurring episodes of chills and • The patient most likely has an infection
excessive sweating. Since his return to the with Plasmodium vivax or Plasmodium
United States from Papua New Guinea, he ovale based on the following:
describes reliably experiencing a fever, chills and • Fever, chills, and diaphoresis in 48
diaphoresis every 48 hours. On physical exam, hour intervals
his abdomen is soft with mild organomegaly • Blood smear revealing trophozoite
in the left upper quadrant. A peripheral blood ring form
smear is ordered. • Splenomegaly
• Chloroquine will eradicate the
erythrocytic parasites, not the liver
hypnozoites in the exo-erythrocytic
forms. When the dormant hypnozoites
become active in the bloodstream,
symptoms will return
• Choice A is incorrect because chloroquine
will not eradicate the exo-erythrocytic life
forms
• Choice B is incorrect because severe
hypoglycemia occurs in pregnant individuals
• Choice D is incorrect because chloroquine
will not worsen these symptoms

Photo Credit: CDC [Public domain]

Chloroquine is administered. Based on the


most likely Plasmodium species, which of
the following is most likely to occur following
treatment?
A. Eradication of all exo-erythrocytic
Plasmodium forms
B. Multiple bouts of severe hypoglycemia prior
to recovery
C. Resolution followed by return of symptoms
months later
D. Worsening fever and chills for several
months
214

Section 8 - Babesia

1. Babies: Babesia 11. Sleeping baby: can cause fatigue


2. Tick toy and Tic Tacs: transmitted through ticks 12. Heat lamps: can cause fever and flu-like symptoms
3. Dust spores: tick saliva carries sporozoites 13. Cleaning lady: can be treated with clindamycin
4. Map of NE United States: ticks with Babesia live in 14. Queen’s Crown with number nine: can be treated
the NE United States with quinine
5. Spores blasting baby into trough: sporozoites 15. Cleaning lady with crown: administer clindamycin
become trophozoites in red blood cells and quinine together
6. Spilled red paint: causes anemia 16. Toe with an ice cream cone on it: can treat with
7. Baby finger painting on wall: diagnosed on atovaquone
peripheral blood smear 17. Zip line: can treat with azithromycin
8. Gold cross: blood smear reveals “Maltese cross” 18. Zip line baby and Ice cream baby: administer
(intraerythrocytic ring form) atovaquone and azithromycin
9. Baby dangling on chains: dianose with polymerase 19. Spleen shaped spot: the spleen is important in
chain reaction (PCR) clearing infection (worse outcome in asplenic
10. Protein drink: Babesia is protozoan patients)
215

REVIEW QUESTIONS ?
1. A 34-year-old asplenic patient presents to
the physician due to malaise, progressive
fatigue, and shortness of breath. He endorses
recent hiking through a forested area in
Massachuchets. His temperature is 39.0°C
(102.2°F). The physician suspects an infection
spread through an arachnid endemic to the
region. After thorough skin inspection, the
physician finds an Ixodes tick on the patient’s
back. Which of the following is most likely true
regarding the patient’s illness?
A. Trophozoites were transmitted from the
Photo Credit: Riccardoariotti [CC BY 3.0 (https://creativecommons.org/
tick’s saliva into the blood
licenses/by/3.0)] B. A complete blood count would reveal a
Figure 5.4.10 - Babesia trophozoite ring form normal hematocrit level
(“Maltese cross”) within red blood cells C. Erythromycin should be considered as part
of the treatment
D. Peripheral blood smear would reveal
intraerythrocytic ring forms

• Correct answer: D
• The patient likely has Babesia based on
the following:
• Tick bite
• Malaise and fever
• Symptoms of anemia (shortness of
breath and fatigue)
• Asplenic
• Intraerythrocytic ring forms (“Maltese
cross”) would be visualized on
peripheral blood smear
• Choice A is incorrect because ticks transmit
sporozoites
• Choice B is incorrect because the patient is
likely anemic
• Choice C is incorrect because azithromycin,
not erythromycin, can treat Babesia
216

Section 9 - Trypanosoma cruzi

1. Cruise line: Trypanosoma cruzi 10. Goat trapped in net: can cause lymphadenopathy
2. Red bugs: transmission via reduviid bugs 11. Sick girl: can cause malaise and flu-like symptoms
3. Couple kissing: reduviid bugs are also known as 12. Chugging contest: causes Chagas disease
“kissing” bugs 13. Protein drink: protozoa
4. Man in protective clothing: prevent bites using long- 14. Weighed down car with popped tire: Chagas disease
sleeved clothing causes dilated cardiomyopathy
5. Insect repellent: prevent bites using insect repellent 15. Vent on car: Chagas disease causes ventricular
6. Tripping goats: diagnose with trypomastigotes on tachycardia
peripheral blood smear 16. Brown cookie tube: Chagas causes megacolon
7. Poop in red paint: reduviid bugs leave feces in bite 17. Giant frosting dispenser: Chagas disease causes
wounds achalasia
8. Man in hat covered with paint: infection can cause 18. Knife, fur, and ox head: treat with nifurtimox
meningoencephalitis 19. Bean knitting: treat with benznidazole
9. Heat lamps: can cause fever

Photo Credit: Farnoosh Farrokhi, Michael F. Vaezi. [CC BY 2.0 (https://


creativecommons.org/licenses/by/2.0)]
Figure 5.4.11 - Barium esophagram showing
achalasia
217

REVIEW QUESTIONS ?
1. A 14-year-old girl presents to the clinic with • Correct answer: B
a fever after returning from a trip to South • The patient likely has acute Chagas
America. Although she remembers getting disease based on the following:
bitten by some bugs during her visit, she • Fever
did not think much of them because the • Painless bites
bites were painless. Other members of her • Travel to South America
family in the United States have not felt sick. • Trypomastigotes on blood smear
Physical examination reveals a nontender • Patients should be given nifurtimox or
and nondistended abdomen. A peripheral benznidazole during the acute phase
blood smear is ordered and trypomastigotes which may prevent meningitis or
are visualized as shown below. Which of the lymphadenopathy
following is true regarding the acute stage of • Choice A is incorrect because achalasia is
her infection? caused by chronic Chagas disease
• Choice C is incorrect because Trypanosoma
cruzi is only transmitted by the reduviid bug
• Choice D is incorrect because
lymphadenopathy is possible during the
acute phase
• Choice E is incorrect because melarsoprol
treats African trypanosomiasis, not Chagas
disease

Photo Credit: Deautsch: PD-DPD Image Library [Public domain]

A. Swallowing may become difficult


B. Nifurtimox may prevent meningeal
inflammation
C. Treat members of her household to prevent
transmission
D. Lymphadenopathy is unlikely
E. Melarsoprol may prevent somnolence and
coma
218

Section 10 - Leishmania donovani

1. Leashed maniac: Leishmania donovani 9. Assorted deflated balloons on frying pan: causes
2. Sand person flying through the air: transmitted pancytopenia
through sandfly bites 10. Skeleton: severe pancytopenia among patients with
3. Man being bitten by sand person: bites cause visceral leishmaniasis has a mortality rate of 90%
painful ulcers 11. Protein drink: protozoa
4. Skin wounds: cutaneous leishmaniasis 12. Masked goat traipsing through blood: amastigotes
5. People impaled through viscera: visceral seen on peripheral blood smear
leishmaniasis 13. Glue causing stye in the eye: treat with
6. Spikes and heat lamps: causes spiking fevers stibogluconate
7. Ballooning spleens: causes splenomegaly 14. Amphibians: treat with amphotericin B
8. Liver-shaped spot on bloating dog: causes
hepatomegaly
219

REVIEW QUESTIONS ?
1. A 23-year-old male presents with a red sore on
his left calf that he first noticed 3 days ago. Last
week, he returned from a trip to Ethiopia and
recalls being bitten by a fly during his stay. He
denies fevers or abdominal pain. An abdominal
exam reveals no palpable organomegaly. The
physician orders a peripheral blood smear which
Photo Credit: Abanima at the Arabic language Wikipedia [CC BY-SA 3.0 reports the presence of multiple amastigotes
(http://creativecommons.org/licenses/by-sa/3.0/)]
found in macrophages. Based on these findings,
Figure 5.4.12 - Cutaneous leishmaniasis
which of the following insects most likely bit the
patient?
A. Sandfly
B. Deer fly
C. Mango fly
D. Tsetse fly

• Correct answer: A
• The patient likely has cutaneous
leishmaniasis based on the following:
• Red sore
• Fly bite
• Trip to Ethiopia
• Amastigotes on blood smear
• Leishmania donovani is transmitted by
the sandfly
• Choice B and C are incorrect because these
transmit Loa Loa
• Choice D is incorrect because the tsetse fly
transmits Trypanosoma brucei, gambiense,
and rhodesiense
220

Section 11 - Trichomonas vaginalis

1. Moaning tricycle boy: Trichomonas vaginalis 6. Spilled frothy alcohol: yellow, frothy vaginal
2. Barrel of alkaline bleach: the pH of the vaginal discharge
discharge pH > 4.5 7. Microscope in fluid: diagnosed on saline microscopy
3. Trough: has motile trophozoites 8. Couple on Metro: treat both partners with
4. Strawberries: causes cervicitis resulting in a metronidazole
“strawberry red” cervix 9. Scarlet letter: is a sexually transmitted infection
5. Protein drink: protozoa
221

REVIEW QUESTIONS ?
1. A 30-year-old woman reports abnormal vaginal
discharge for the past 5 days. She states that
the discharge is foamy and greenish-yellow in
color. After obtaining a more detailed history,
she states that she recently became sexually
active with a new partner. The physician
performs a speculum exam and appreciates an
erythematous cervix. A sample of the vaginal
discharge is then obtained and promptly
examined under the microscope. Microscopy
reveals several protozoan organisms with
Photo Credit: isis325 via Flickr noticeable flagella. Based on the information
Figure 5.4.13 - Flagellated Gardnerella trophozoites provided, which of the following statements is
most likely true?
A. She obtained the infection from her partner
B. A pH of 3.7 would rule out T. vaginalis
C. The pathogen is part of the normal vaginal
flora
D. Clue cells will indicate the likely cause of her
infection

• Correct answer: A
• Trichomonas vaginalis is the most likely
organism based on the following:
• Foamy, greenish-yellow discharge
• Erythematous cervix
Photo Credit: CDC [Public domain] • New sexual partner
Figure 5.4.14 - Gardnerella vaginalis on saline • Flagellated protozoa on saline
microscopy microscopy
• Trichomonas vaginalis is sexually
transmitted
• Choice B is incorrect because evidence of
flagellated protozoa makes Trichomonas
vaginalis most likely, regardless of an
inconsistent vaginal pH
• Choice C is incorrect because Trichomonas
vaginalis is not part of the normal vaginal
flora
• Choice D is incorrect because clue cells
occur with Gardnerella vaginalis infections
222

Section 12 - Enterobius vermicularis (pinworm)

1. Pinball machine: pinworm 6. Tape removed by child: diagnosed by viewing eggs


2. Vermin: Enterobius vermicularis (pinworm) under the microscope following a tape test
3. Merry-go-round: roundworm (nematode) 7. Pyro kid with Pam spray: treat with pyrantel
4. Wedgie: causes anal pruritus pamoate
5. Poop on face: fecal-oral transmission 8. Bent phone: treat with bendazoles
223

REVIEW QUESTIONS ?
1. A 5-year-old boy is brought to the physician
by his concerned mother who states that he
is constantly scratching his butt. The mother
first noticed this habit several weeks ago after
her son began attending kindergarten. Physical
examination reveals erythema surrounding the
perianal region. An adhesive tape is placed in
the perianal region and transferred to a glass
slide. The slide is viewed under light microscopy
and reveals the presence of eggs. This patient’s
Photo Credit: CDC [Public domain] condition is best treated with which of the
Figure 5.4.15 - Pinworm egg following medications?
A. Ivermectin
B. Diethylcarbamazine
C. Pyrantel pamoate
D. Praziquantel
E. Doxycycline

• Correct answer: C
• This patient has a pinworm infection
• The boy has been constantly scratching
his butt
• An adhesive tape placed in the perianal
region revealed the presence of eggs
when viewed under light microscopy
• The treatment for this parasitic infection
is pyrantel pamoate (can also be treated
with mebendazole)
• The other answer choices (A, B, D, and E)
are antiparasitic medications but none of
them are used to treat pinworm
224

Section 13 - Ascaris lumbricoides (giant roundworm)

1. Asterisks: Ascaris lumbricoides (giant roundworm) 7. Perforated hose: causes intestinal perforation
2. Giant man: Ascaris lumbricoides (giant roundworm) 8. Man sprayed in the mouth: migrates to the
3. Merry-go-round: roundworm (nematode) mouth and nose before descending back into the
4. Man stepping in poop: fecal-oral transmission gastrointestinal tract
5. Spit and Loafer shoes: causes Löffler syndrome 9. Banana trees: may cause biliary obstruction
6. Man working on valve of pipe: may cause 10. Colored rocks: diagnosed by detecting knobby-
obstruction at the ileocecal valve coated oval eggs in the feces via microscopy
11. Bent phone: treat with bendazoles

Figure 5.4.16 - Life cycle of parasites that cause Löffler syndrome


225

Photo Credit: CDC [Public domain]


Figure 5.4.17 - Ascaris lumbricoides egg

REVIEW QUESTIONS ?
1. A 6-year-old boy is brought to the physician • Correct answer: B
by his mother because of a 1-month history of • This patient has an Ascaris lumbricoides
intermittent right upper quadrant abdominal infection
pain. Upon further inquiry she states that a • 1-month history of intermittent RUQ
month or so ago he developed a cough which pain → biliary problems
has since resolved. Physical examination reveals • A cough a month or so ago → Löffler
mild discomfort in the right upper quadrant of syndrome
the abdomen. A stool culture is shown below. • Stool culture shows a parasitic
This patient is at risk of developing which of the egg  (seen in the image)
following? • Ascaris lumbricoides may cause
obstruction at the ileocecal valve which
is in close proximity to the appendix →
appendicitis
• A is incorrect because this is suggestive of
Toxocara canis which does not cause Löffler
syndrome and isn’t typically associated with
biliary colic
• C is incorrect because this is suggestive
of Toxocara canis, Onchocerca volvulus,
and Loa loa but these aren’t associated
with biliary colic and do not cause Löffler
syndrome
Photo Credit: CDC/ Dr. Mae Melvin [Public domain] • D is incorrect because seizures are
A. Myocarditis suggestive of Toxocara canis or Taenia
B. Appendicitis solium, but these do not causes biliary colic
C. Visual impairment or Löffler syndrome
D. Seizures
226

Section 14 - Strongyloides stercoralis (threadworm)

1. Strong contestants: Strongyloides stercoralis 6. Confetti on skin: initiates the infection via
(threadworm) penetration of the skin
2. Threads on sports bra: Strongyloides stercoralis 7. Confetti over hose and pool: can penetrate the
(threadworm) intestinal wall and enter the bloodstream
3. Merry-go-round: roundworm (nematode) 8. Loafer shoes: causes Löffler syndrome
4. Rabbi: diagnosed with microscopy (rhabditiform 9. Bent phone: treat with bendazoles
larvae in the feces) 10. Ivory tusk: treat with ivermectin
5. Confetti popper: autoinfection
227

Photo Credit: CDC [Public domain]


Figure 5.4.18 - Strongyloides stercoralis larva

REVIEW QUESTIONS ?
1. A 31-year-old female presents to the physician • Strongyloides is unique because the
due to a chronic cough. She states that she first eggs released from the mature helminth
noticed the cough approximately one year ago hatch within the intestines
after traveling to Thailand. Physical examination • Within the intestines they can penetrate
reveals a high pitched sound over both lung the gastrointestinal wall and migrate
fields best heard during expiration. A complete into the bloodstream causing the cycle
blood cell count reveals elevated levels of to repeat itself
eosinophils but is otherwise normal. Stool • The unique life cycle of Strongyloides
examination reveals an absence of parasitic means it can cause repeated bouts of
eggs. This patient’s condition can be best Löffler syndrome (eg, chronic cough)
characterized by which of the following? • The eggs hatch inside of the host
intestines so eggs will not be detected
A. Disruption of granuloma formation
in the stool (only larvae will be)
B. Hyperplasia and hypertrophy of respiratory
goblet cells • A is a reference to secondary TB but this
C. Penetration of the alveoli by a parasite typically presents with weight loss, fevers,
D. Noncaseating granulomas night sweats, and hemoptysis so this is
unlikely
• Correct answer: C • B is a reference to pathological findings
• This patient has a Strongyloides stercoralis of COPD which is not associated with
infection resulting in Löffler syndrome eosinophilia
• Developed a cough one year ago after • D is a reference to pathological findings
traveling to Thailand of sarcoidosis which can cause pulmonary
• Wheezing (high pitched sound over findings, but not typically associated with
both lung fields) eosinophilia
• Eosinophilia
• Strongyloides penetrates the skin,
enters the bloodstream, and then
penetrates the alveoli causing
pulmonary symptoms
• The organism ascends the trachea and
descends into the GI tract where the
mature parasite remains
228

Section 15 - Ancylostoma duodenale and Necator americanus (hookworms)

1. Ancestry tree: Ancylostoma duodenale 7. Itchy snake tattoo: cause a pruritic maculopapular
2. Nectarine tree: Necator americanus rash (often described as serpiginous)
3. Fishing hook: hookworms 8. Loafer shoes: cause Löffler syndrome
4. Merry-go-round: roundworm (nematode) 9. Pyro kid with Pam spray: treat with pyrantel
5. Man stepping on nectarine: transmitted via larvae pamoate
penetration of the skin 10. Bent phone: treat with bendazoles
6. Deflated balloon: cause anemia by sucking blood
from the intestinal wall
229

REVIEW QUESTIONS ?
1. A 14-year-old boy is brought to the physician
due to a persistent cough that began yesterday.
The family recently returned from a trip to the
beaches of Oregon where the patient walked
around barefoot. A complete blood cell count
reveals elevated levels of eosinophils but is
otherwise normal. The physician suspects
a parasitic infection and orders a stool
preparation but no organisms are seen. What
finding will most likely be observed in this
Photo Credit: CDC [Public domain] patient?
Figure 5.4.19 - Hookworm egg A. Conjunctival pallor
B. Diarrhea
C. A chest x-ray with round opacities seen on
both lung fields
D. Hookworm larvae in the gastrointestinal
tract

• Correct answer: C
• This patient has a hookworm infection
resulting in Löffler syndrome
• A cough after walking around barefoot
on the beaches of Oregon
• Eosinophilia
• The physician suspects a parasitic
Photo Credit: CDC [Public domain] infection
Figure 5.4.20 - Cutaneous hookworm lesion • Hookworms penetrate the skin, migrate
to the bloodstream, and then penetrate
the alveoli → Löffler syndrome → round
opacities seen on both lung fields
• The lifestyle of hookworms is unique
in that Löffler syndrome is a transient
acute condition (the parasite has not
yet descended into the gastrointestinal
tract → absence of GI symptoms and
negative stool sample)
• A, B, and D can be caused by hookworms,
but this patient’s cough only started
yesterday and his stool sample is negative
• The pathogen hasn’t yet descended into the
gastrointestinal tract so anemia, diarrhea,
and larvae in the gastrointestinal tract
would be unlikely
230

Section 16 - Trichinella spiralis

1. Tricycle on spiral-shaped pathway: Trichinella 6. Bricks: encysted larvae infect striated muscle tissue
spiralis 7. Fire by bricks: causes muscle inflammation and
2. Merry-go-round: roundworm (nematode) myalgias
3. Peter’s Pork: transmitted through undercooked 8. Heat lamps: may cause fever
meat (especially pork) 9. Toy minion: may cause periorbital edema
4. Customer stepping on poop: also exhibits fecal-oral 10. Vomiting boy: may cause nausea and vomiting
transmission 11. Bent phone: treat with bendazoles
5. Bloody stream: disseminates into the bloodstream
231

REVIEW QUESTIONS ?
1. A 23-year-old male presents to the physician
due to calf pain that he first noticed one week
ago. Yesterday the pain was so severe that
he was unable to walk. He denies any recent
change in activity. Physical examination is
significant for periorbital edema and muscle
tenderness over the posterior aspect of the
left leg. A muscle biopsy is obtained and shown
below. This patient most likely developed the
infection after:

Photo Credit: TheMrGrove [CC BY-SA 4.0 (https://creativecommons.org/


licenses/by-sa/4.0)]
Figure 5.4.21 - Striated muscle tissue infected with
Trichinella spiralis

Photo Credit: Froggerlaura at English Wikipedia [modified]

A. Consuming undercooked pork


B. Larvae penetration of the skin
C. Exposure to the ixodes deer tick
D. Surfing in a tropical area

• Correct answer: A
• This patient has a Trichinella spiralis
infection
• Calf pain, periorbital edema and a
muscle biopsy shows a spiral-shaped
organism within muscle tissue
• This organism is transmitted via
consumption of undercooked pork
• The organism infects the muscle tissue
where encysted larvae may be seen (as
shown in the image)
• B is describing Strongyloides stercoralis but
this infects the lungs and gastrointestinal
tract so myalgias and an organism seen on a
muscle biopsy would be unlikely
• C is referencing several organisms such
Photo Credit: Klaus D.Peter, Gummersbach, Germany [CC BY 3.0 de (https://
creativecommons.org/licenses/by/3.0/de/deed.en)] as Borrelia burgdorferi, Anaplasma, or
Figure 5.4.22 - Periorbital edema Ehrlichia, but these are not spiral shaped
organisms and do not directly infect muscle
tissue
• D is a reference to Leptospira interrogans
which is spiral shaped, but a muscle biopsy
would not reveal spiral shaped larvae within
muscle tissue
232

Section 17 - Trichuris trichiura (whipworm)

1. Whip: Trichuris trichiura (whipworm) 6. Deflated red balloon: may cause anemia
2. Merry-go-round: roundworm (nematode) 7. Pouch on rear end: may cause rectal prolapse in
3. Poop splashed on mouth: fecal-oral transmission children
4. Loose stool: may cause loose stools 8. Bent phone: treat with bendazoles
5. Happy girl: often asymptomatic
233

REVIEW QUESTIONS ?
1. A resident physician is in Honduras on a
humanitarian trip when an 8-year-old girl is
brought to the clinic due to several months
of fatigue. Physical examination reveals
conjunctival pallor. Laboratory analysis of the
stool reveals the presence of eggs belonging
to a roundworm. After thorough evaluation
the resident physician explains that without
treatment, the patient is at risk of developing
rectal prolapse in the setting of a heavy
Photo Credit: Unknown photographer/CDC [Public domain] infection. This patient’s condition is best treated
Figure 5.4.23 - Whipworm egg with which of the following?
A. Watchful waiting
B. Mebendazole
C. Ivermectin
D. Diethylcarbamazine

• Correct answer: B
• This girl has an infection caused by Trichuris
trichiura
• Anemia (fatigue and conjunctival pallor)
• Laboratory analysis has confirmed that
the anemia is due to a roundworm
• The resident concludes that this
infection increases the risk of
developing rectal prolapse
• Trichuris trichiura should be treated
with mebendazole
• A is wrong because without treatment this
patient is at risk of developing ongoing
anemia, diarrhea, and rectal prolapse
• C is wrong because while this may have
some efficacy against whipworm, it has not
been shown to be as effective as bendazoles
• D is wrong because diethylcarbamazine
is used to treat Loa loa and Wuchereria
bancrofti
234

Section 18 - Toxocara canis

1. Toxic cloud over cans: Toxocara canis 7. Juice on sunglasses: affects the eyes resulting in
2. Round gears: roundworm (nematode) visual impairment and blindness
3. Brown mess: fecal-oral transmission 8. Muscular guy in car: myocarditis
4. Brown juice through sieve turns red: larvae migrate 9. Sleeping man with hat: CNS penetration resulting in
through the intestinal wall into the bloodstream coma
5. Dalmatian dog: transmitted through dog feces 10. Alarm Clock: CNS penetration resulting in seizures
6. Liver shaped spot: affects the liver 11. Bent phone: treat with bendazoles
235

REVIEW QUESTIONS ?
1. A 4-year-old boy is brought to the physician
by his mother due to recent changes in his
skin color. She states that several weeks
ago they started going to a new playground
contaminated with dog feces, and is worried
her son may have become ill after accidental
ingestion of the feces. His temperature is 38.7°C
(101.6°F). Physical examination reveals jaundice
of the skin and hepatomegaly. Laboratory
analysis of the patient’s blood is significant for
transaminitis and elevated levels of eosinophils.
An enzyme-linked immunosorbent assay test
detects IgG antibodies to antigens secreted by a
roundworm. This patient’s condition:
A. Is transmitted through larvae penetration of
the skin
B. Should be treated with ivermectin
C. Can be best explained by autoinfection
D. May cause myocarditis

• Correct answer: D
• This boy has hepatitis caused by Toxocara
canis
• He likely ingested dog feces (risk factor
for Toxocara)
• He has a fever (38.7°C), jaundice, and
hepatomegaly → hepatitis
• An ELISA test detected the presence of a
roundworm
• Toxocara canis also causes myocarditis
• A , B, and C, are all suggestive of
Strongyloides stercoralis which is
transmitted when larvae penetrate the skin
236

Section 19 - Onchocerca volvulus

1. Crowd yelling “encore”: Onchocerca volvulus 6. Girl sneezing: microfilariae may cause an allergic
2. Merry-go-round: roundworm (nematode) reaction
3. Fly suit: transmitted through the female blackfly 7. Sunglasses by the river: may cause river blindness
4. Bumps on the bouncer: causes subcutaneous 8. Ivory tusks: treat with ivermectin
nodules
5. Loose string: loss of elastic fibers resulting in skin
atrophy
237

REVIEW QUESTIONS ?
1. A resident physician is in Kenya on a
humanitarian trip when a 37-year-old male is
brought to the clinic due to blurry vision which
began several months ago. The patient also
states that he has had small hard bumps on his
skin which he noticed several years ago. Physical
examination reveals subcutaneous nodules
prevalent over the pelvic girdle. Slit-lamp
examination reveals microfilariae in the anterior
chamber of the eye. After thorough evaluation,
the resident informs the team that this patient’s
condition was likely transmitted through the
female blackfly. This patient’s condition is best
Photo Credit: John Curtis’s British Entomology 1824–1840 [Public domain]
treated with which of the following?
Figure 5.4.24 - Blackfly
A. Praziquantel
B. Mebendazole
C. Ivermectin
D. Diethylcarbamazine

• Correct answer: C
• This patient has an infection caused by
Onchocerca volvulus
• Blurry vision, subcutaneous nodules,
and a slit-lamp exam has revealed
microfilariae in the eye
• Transmitted by the female blackfly
• Loa loa can present similarly but is not
transmitted by the female blackfly
• A and B are wrong because these are not
first-line treatments for either Onchocerca
volvulus or Loa loa
• D is wrong because this is used to treat Loa
loa but not Onchocerca volvulus
238

Section 20 - Loa loa and Wuchereria bancrofti

1. Lara Croft: Wuchereria bancrofti 8. Elephant: Wuchereria bancrofti causes lymphatic


2. Luau: Loa loa filariasis (elephantiasis)
3. Merry-go-round tattoo: roundworms (nematodes) 9. Beaded net: Wuchereria bancrofti invades the
4. Horse, deer, and mango: Loa loa is transmitted by lymphatic system resulting in inflammation and
the deer fly, horse fly, and mango fly lymphedema
5. Swollen worms: Loa loa causes swelling of the skin 10. One arrow in quiver: symptoms begin approximately
6. Tribe member inspecting worms: Loa loa may 9 months to 1 year following infection
migrate to the conjunctiva resulting in transient 11. Pistol with magazines of ammo: Loa loa
ocular edema and inflammation and Wuchereria bancrofti are treated with
7. Mosquitoes: Wuchereria bancrofti is transmitted by diethylcarbamazine
female mosquitoes
239

REVIEW QUESTIONS ?
1. A resident physician is in Uganda on a
humanitarian trip when a 47-year-old male
is brought to the clinic due to eye pain which
began several days ago. The patient also states
that he has experienced several episodes of
pain and pruritus on the face over the past
several weeks. Physical examination reveals
subcutaneous edema prevalent over the cheeks.
Slit-lamp examination reveals conjunctivitis.
After thorough evaluation, the resident informs
the team that this patient’s condition was likely
transmitted by a:
A. Female mosquito
B. Tsetse fly
C. Deer fly
D. Sandfly

Photo Credit: Courtesy of the US Department of Health and Human Services • Correct answer: C
Figure 5.4.25 - Elephantiasis • This patient has an infection caused by Loa
loa
• Conjunctivitis and subcutaneous edema
in the face are unique to this parasite
• Loa loa is transmitted by the deer fly,
horse fly, and mango fly
• A  is a reference to Wuchereria bancrofti,
but this causes elephantiasis
• B is a reference to Trypanosoma brucei, but
this causes somnolence and coma
• D is a reference to Leishmania donovani,
but this causes spiking fevers, skin ulcers,
hepatomegaly, and pancytopenia
240

Section 21 - Taenia solium

1. Tanning salon: Taenia solium 9. Boil on head: cysticerci in the brain causes
2. Eating pig meat with cysts: transmission of encysted neurocysticercosis
larvae in undercooked pork 10. Stepped on hose: neurocysticercosis can cause
3. Taped worm: tapeworm obstructive hydrocephalus
4. Worms in mud: ingestion of encysted larvae leads to 11. Malfunctioning tanning bed: neurocysticercosis can
intestinal tapeworm (diarrhea) cause seizures
5. Pretzel: treat intestinal tapeworm with praziquantel 12. Beam of light on woman’s head: diagnose
6. Health inspector examining egg from mud: diagnose neurocysticercosis with a CT scan
intestinal tapeworm by identifying eggs in stool 13. Receptionist eating pretzel: treat cysticercosis with
7. Eggs in poo: ingestion of eggs from human feces praziquantel
8. Woman with boils: ingestion of eggs causes 14. Harvey Porter and Albus: treat neurocysticercosis
cysticercosis with albendazole
241

Figure 5.4.26 - Taenia solium life cycle

Photo Credit: Pjgalindo [CC BY-SA 3.0 (https://creativecommons.org/licenses/


by-sa/3.0/)]
Figure 5.4.27 - Taenia solium on microscopy

Photo Credit: CDC [Public domain]


Figure 5.4.28 - Taenia solium egg in stool sample
242

REVIEW QUESTIONS ?
1. A 45-year-old butcher presents to the clinic
with diarrhea and abdominal pain for several
weeks. He indicates that he routinely eats
scraps of uncooked pork while he is butchering.
The physician notes that the patient may have
acquired a tapeworm infection from the pork.
Assuming the physician is correct, which of the
following statements is true?
A. The infection may eventually cause seizures
B. Diagnosis will be confirmed with eggs found
in the stool
C. Albendazole should be part of the
treatment regimen
D. The patient ingested T solium eggs to
acquire the infection

• Correct answer: B
• The patient likely has intestinal
Photo Credit: Rjgalindo [CC BY-SA 3.0 (https://creativecommons.org/licenses/
tapeworm based on the following:
by-sa/3.0/)] • Diarrhea and abdominal pain
Figure 5.4.29 - Neurocysticercosis on imaging • Regular ingestion of undercooked
pork
• Intestinal tapeworm is diagnosed by
identifying eggs in stool
• Choice A is incorrect seizures occur in
neurocysticercosis which is acquired by
eating Taenia eggs in feces
• Choice C is incorrect because albendazole is
used to treat neurocysticercosis
• Choice D is incorrect because intestinal
tapeworm is acquired through ingestion of
cysts in undercooked pork
243

Section 22 - Diphyllobothrium latum

1. “Dive For A Bath” sign: Diphyllobothrium latum 4. Moon: B12 deficiency causes megaloblastic anemia
2. Taped worms: tapeworm 5. Dead fish: transmitted through ingestion of
3. Kobe Bryant low in the pit: infection causes B12 undercooked fish
deficiency 6. Pretzel: praziquantel

REVIEW QUESTIONS ?
1. An 8-year-old girl presents to the physician • Correct answer: D
due to a 2-month history of worsening fatigue. • The patient likely has Diphyllobothrium
Physical examination reveals conjunctival pallor. latum based on the following:
Blood tests are performed which reveal a • Macrocytic anemia (high MCV and
hemoglobin of 9.4 g/dL and a mean corpuscular low Hb)
volume of 108 fL. After thorough evaluation the • Symptoms of anemia (fatigue and
physician prescribes praziquantel. Based on the conjunctival pallor)
causal organism, which of the following is a true • Treatment with an antiparasitic
statement? (praziquantel)
• Diphyllobothrium latum is transmitted
A. Transmission occurred through uncooked
through ingestion of larvae in
pork
B. Cobalamin levels are likely elevated undercooked fish
C. Anaphylaxis may result from cyst rupture • Choice A is incorrect because undercooked
D. Transmission occurred through the pork transmits other parasites (eg, taenia
ingestion of larvae solium)
• Choice B is incorrect because
Diphyllobothrium latum causes low B12
(cobalamin)
• Choice C is incorrect because anaphylaxis
occurs with Echinococcus cyst rupture
244

Section 23 - Echinococcus granulosus

1. Crazy cock lady: Echinococcus granulosus 7. Big bean bag: hydatid cysts cause hepatomegaly
2. Worms taped to table: tapeworm 8. Bean bag contents landing on girl: cyst rupture can
3. Chicken eggs in dog poop: transmission via egg- cause anaphylaxis
infested dog feces 9. Harvey Porter and Albus: treat with albendazole
4. Sheep in dog food bowl: sheep are intermediate 10. Alcohol bottle: inject cyst with alcohol prior to
hosts surgery
5. Baby chick in poop track shaped like intestines: eggs 11. Scalpel in first aid box: surgically remove the hydatid
hatch into larvae within the intestines cyst
6. Bulging tears in bean bag: hydatid cysts form in the
liver

Figure 5.4.30 - Echinococcus granulosus life cycle


245

REVIEW QUESTIONS ?
1. A 45-year-old female presents to the physician
with right upper quadrant pain. Physical
examination reveals hepatomegaly. A CT
scan is performed which reveals a liver mass
surrounded by a white ring of calcification. The
mass is not consistent with an abscess. Plans are
made to surgically remove the mass following
an albendazole regimen. How did the patient
most likely acquire the infection causing the
mass?
Photo Credit: Ganimedes [CC BY-SA 4.0 (https://creativecommons.org/ A. Larvae in dog feces
licenses/by-sa/4.0)] from Wikimedia Commons
B. Eggs in dog feces
Figure 5.4.31 - Echinococcus granulosus tapeworm C. Larvae in sheep feces
on microscopy D. Eggs in sheep feces

• Correct answer: B
• The patient most likely has hydatid cysts
from Echinococcus granulosus based on
the following:
• RUQ pain
• Hepatomegaly
• CT scan revealing “eggshell
calcification”
• Pre-surgical albendazole regimen
• Echinococcus granulosus is acquired
through ingestion of eggs found in dog
feces
• Choice A is incorrect because eggs hatch
into larvae in the intestines after ingestion
Photo Credit: Wenya Liu, Eric Delabrousse, Oleg Blagosklonov, Jing Wang, • Choices C and D are incorrect because
Hongchun Zeng, Yi Jiang, Jian Wang, Yongde Qin, Dominique Angèle Vuitton
and Hao Wen [CC BY 4.0 (https://creativecommons.org/licenses/by/4.0)], via
humans do not acquire infection through
Wikimedia Commons, adjustments made sheep feces
Figure 5.4.32 - Liver hydatid cyst on MRI

Photo Credit: By Yale Rosen via Flickr, adjustments made


Figure 5.4.33 - Liver hydatid cyst (gross)
246

Section 24 - Schistosoma haematobium, mansoni, and japanicom

1. Twin sisters: Schistosoma 14. Japanese embassy: S. mansoni and japanicom


2. Tree of toads: trematodes 15. Spleen shaped welt: S. mansoni and japanicom can
3. Eggs with spines on side: eggs of S. mansoni and cause splenomegaly
japanicom have lateral spines 16. Muddy water: S. mansoni and japanicom can cause
4. Smashed eggs: eggs of S. haematobium have diarrhea
terminal spines 17. Ulcer shaped chunk of dirt: S. mansoni and
5. Eggs: Schistosoma eggs have spines japanicom can cause intestinal ulceration leading to
6. Hatching eggs: eggs only hatch in freshwater iron deficiency anemia
7. Larvae from eggs: eggs release immature larvae 18. Shattered port-hole: S. mansoni and japanicom can
8. Glowing snail: immature larvae must infect cause portal hypertension
freshwater snail (intermediate host) 19. Fat dalmatian dog falling from boat: S. mansoni and
9. Glowing toxic circles: snails release mature larvae japanicom can cause hepatomegaly
called cercariae 20. Gnarled fibrous branches: S. mansoni and
10. Woman swimming in pond: cercariae infect humans japanicom can cause liver cirrhosis
through skin (e.g. open wounds) 21. Steam from pressure cooker: all species can cause
11. Laser pointing to crotch: can cause hematuria pulmonary hypertension
12. Hematuria sounds like haematobium: Schistosoma 22. Pretzels: treat all forms of schistosomiasis with
haematobium praziquantel
13. Hope ribbon: chronic bladder infection can cause
squamous cell carcinoma of the bladder
247

REVIEW QUESTIONS ?
1. A 65-year-old Middle-eastern female presents
to the clinician due to worsening fatigue and
dark red diarrhea that began 5 years ago. She
emigrated to the United States 1 year ago.
Physical examination reveals a palpable spleen
and liver enlargement. A stool sample was sent
to the pathologist and the results are shown
below.

Figure 5.4.34 - Schistosoma life cycle


Photo Credit: CDC [Public domain]

2. Which of the following is true regarding this


pathogen?
A. It infects the vasculature of the bladder
B. Eggs must hatch in saltwater
C. It can infiltrate lung alveoli causing cough
D. It can cause a decreased MCV and Hb

• Correct answer: D
• The patient likely has an infection with
S. mansoni or S. japanicom based on the
Photo Credit: Wikimedia Commons [Public domain]
following:
Figure 5.4.35 - Lateral spine on Schistosoma egg
• Parasitic egg with a lateral spine
(mansoni or japanicom)
• Hepatosplenomegaly
• Bloody diarrhea
• Fatigue (symptom of anemia)
• Schistosoma mansoni or japanicom can
ulcerate the intestines causing chronic
blood loss, leading to iron deficiency
anemia (low MCV and low Hb)
• Choice A is incorrect because Schistosoma
haematobium infects bladder vasculature
• Choice B is incorrect because Schistosoma
eggs must hatch in freshwater
• Choice C is incorrect because Schistosoma
Photo Credit: CDC [Public domain]
spp. damage pulmonary vasculature, not
Figure 5.4.36 - Lateral spine on Schistosoma alveoli
haematobium egg
248

Section 25 - Clonorchis sinensis

1. Clones of orcs: Clonorchis sinensis 4. Goal stones: can cause pigmented gallstones
2. Tree with toads: trematodes 5. Workers collecting bananas with a pole: can cause
3. Orc eating raw fish: transmitted through cholangiocarcinoma
undercooked fish 6. Pretzels: treat with praziquantel
249

REVIEW QUESTIONS ?
1. A 64-year-old male with a 2-month history of • Correct answer: A
recurrent right upper quadrant abdominal pain • Biliary infection with Clonorchis sinensis
presents to a gastroenterologist for evaluation. is most likely based on the following:
The patient states the pain is worse after he • Biliary colic (transient RUQ pain)
eats and then recedes an hour later. Endoscopic • ERCP revealing pigmented
retrograde cholangiopancreatography (ERCP) gallstones
is recommended to evaluate the biliary tract. • Clonorchis sinensis is transmitted
During the procedure, the physician identifies through fish and treated with
dark stones and removes them. A picture of praziquantel
the procedure is shown below. Additional • Choice B is incorrect because albendazole
evaluation suggests the patient’s pain was due does not treat Clonorchis sinensis
to a helminth infection. Which of the following • Choice C is incorrect because Clonorchis
is true regarding the infectious organism? sinensis is not transmitted via pork
• Choice D is incorrect because Clonorchis
sinensis is not transmitted through pork or
treated with albendazole

Photo Credit: Samir (The Scope) at English Wikipedia [CC BY 2.5


(https://creativecommons.org/licenses/by/2.5)]

A. Transmitted via fish; treated with


praziquantel
B. Transmitted via fish; treated with
albendazole
C. Transmitted via pork; treated with
praziquantel
D. Transmitted via pork; treated with
albendazole
250

Section 26 - Sarcoptes scabiei

1. Arm scab: Sarcoptes scabiei (scabies) 6. Baby with snake: creates serpiginous burrow lines
2. Swarming children: common in children 7. Layer of corn: burrow to the stratum corneum
3. Red marker on children’s hands: infects interwebbed 8. Daughter with a perm: treat with permethrin
spaces 9. Smoke surrounding everyone: all household
4. Markered skin on dad: transmitted through skin-to- members should be treated
skin contact 10. Laundry: wash all bedding and clothing
5. Dad scratching hands: causes pruritus
251

REVIEW QUESTIONS ?
1. A 35-year-old male presents to his family
physician with itching on his hands for 2 weeks.
The patient states that several of his children
have experienced similar symptoms recently.
Physical examination reveals several red
papules on the patient’s hand as shown below.
The physician writes a prescription for topical
permethrin. Which of the following should also
be considered as part of the management of
this patient’s condition?

Photo Credit: CDC [Public domain]


Figure 5.4.37 - Scabies-related erythematous
papules

Photo Credit: Wiki [Public domain]

A. Topical permethrin only for the


symptomatic children
B. Wash linens of only the symptomatic
children
C. Wash linens of each household member
D. Topical permethrin for entire household of
only symptomatic contacts

• Correct answer: C
• Scabies infection is most likely based on
the following:
• Pruritus
• Household members with similar
symptoms
• Papules on physical exam
• All household members should receive
topical permethrin and have linens
washed
• Choices A, B and D are incorrect because
these treatments exclude household
members that may have scabies
252

Section 27 - Pediculus corporis, pubis and capitis

1. Sign reading, “Teacher Pedicure Night”: Pediculus 8. Recon child suspended by armpits and waist:
species Pediculus corporis infests the axilla and waist
2. Alien attacking the teacher’s head: Pediculus capitis 9. Alien attack the teachers vinyl records: lice transmit
infests scalp hair Borrelia recurrentis
3. Particles from the hatched egg making the teacher’s 10. Alien prowling: lice transmit Rickettsia prowazekii
skin itchy: lice cause pruritus 11. Teacher yelling “Cortana” into her smartphone: lice
4. Alien drinking spilled red nail polish: lice drink transmit Bartonella quintana
human blood 12. Girl with perm punching an alien: treat with
5. Children coming to defend the school: school permethrin
attendance should not be disrupted 13. Boys attacking alien eggs with school museum tusks:
6. Alien eggs stabbed with knitting needles: lice eggs treat with ivermectin
are called nits 14. Confused principal ready for a marathon: treat with
7. Child looking at pubic hairs on maturation poster: malathion
Pediculus pubis infests pubic hair
253

REVIEW QUESTIONS ?
1. A 13-year-old boy is brought to the physician
by his mother due to persistent dandruff. She
states that he has been constantly itching
his scalp for the past two weeks. Physical
examination reveals several small white
particles that are cemented securely to the
hair shaft. The physician also appreciates
several areas of excoriations on the scalp.
Which of the following is the most appropriate
Photo Credit: KostaMumcuoglu at the English Wikipedia [CC BY-SA 3.0
(https://creativecommons.org/licenses/by-sa/3.0/)] recommendation for the patient?
Figure 5.4.38 - Pediculus capitis louse
A. Oral ivermectin
B. Exclusion from school activities until the
infection has been eradicated
C. Topical permethrin applied to the scalp
D. Topical malathion applied to the scalp and
pubic regions

• Correct answer: C
• Head lice with Pediculus capitis is most
Photo Credit: Gilles San Martin via Flickr likely based on the following:
Figure 5.4.39 - Pediculus nit on hair shaft • Pruritus with scalp excoriations
• Nits
• Head lice may be treated with topical
permethrin
• Choice A is incorrect because topical
ivermectin, not oral ivermectin, can treat
lice
• Choice B is incorrect because school
attendance should not be disrupted
• Choice D is incorrect because pubic lice
should not be suspected
Photo Credit: Ed Uthman from Houston, TX, USA [CC BY 2.0 (https://
creativecommons.org/licenses/by/2.0)], via Wikimedia Commons
Figure 5.4.40 - Pediculus pubis louse

Photo Credit: Janice Harney Carr, Center for Disease Control [Public domain],
via Wikimedia Commons
Figure 5.4.41 - Pediculus corporis louse
254

VIRUSES

Figure 5.4.42 - Virus overview figure


255

Section 1 - Herpes simplex virus-1 (HSV-1) and herpes simplex virus-2 (HSV-2)

1. Hercules: Herpes simplex virus (HSV-1 & HSV-2) 15. Temple on fire: HSV-1 can cause temporal lobe
2. Dark colors: DNA viruses encephalitis
3. Two snakes moving out of temple: double stranded 16. “S” on clothing: HSV-2 is sexually transmitted
4. Straight path to temple: linear 17. Torch: a TORCHES infection and can cause neonatal
5. Mist: HSV-1 is transmitted through respiratory herpes
droplets 18. Flames over crotch: HSV-2 can cause painful
6. Three gems on belt: HSV-1 remains latent in vesicular genital lesions
trigeminal ganglion 19. Green beads: HSV-2 can cause inguinal
7. Basset hound: HSV-1 may precipitate behçet lymphadenopathy
syndrome 20. Hat on fire: HSV-2 is more commonly associated
8. Ginger in satchel: HSV-1 can cause gingivostomatitis with meningitis
9. Carrots in satchel: HSV-1 can cause 21. Sack of weapons: HSV-2 remains dormant in sacral
keratoconjunctivitis neural ganglia
10. Hercules’ bloody finger: HSV-1 can cause herpetic 22. 3 chains: diagnosed with PCR
whitlow 23. Zeus with giant hammer: a Tzanck smear may reveal
11. Blood on snake lips: HSV-1 can cause herpes labialis multinucleated giant cells
12. Red bite marks on arm: HSV-1 can cause erythema 24. Cow in cyclone: Cowdry A inclusions may be seen
multiforme 25. Cyclone: treat with acyclovir
13. Red, bloody eyes: HSV-1 can cause retinitis 26. Violet-colored cyclone: treat with valacyclovir
14. Craters: HSV-1 can cause ulcerated esophagitis 27. Fan cyclone: treat with famciclovir
256

Photo Credit: Marco Mayer [CC BY-SA 4.0 (https://creativecommons.org/


licenses/by-sa/4.0)]
Figure 5.4.45 - Viral keratoconjunctivitis

Photo Credit: Henry Vandyke Carter [Public domain]


Figure 5.4.43 - Trigeminal ganglion

Photo Credit: James Heilman, MD [CC BY-SA 3.0 (https://creativecommons.


org/licenses/by-sa/3.0)]

Photo Credit: James Heilman, MD [CC BY-SA 3.0 (https://creativecommons. Figure 5.4.46 - Herpetic whitlow
org/licenses/by-sa/3.0)]
Figure 5.4.44 - Gingivostomatitis
257

Photo Credit: Courtesy of the U.S. Department of Health and Human Services
and Dr. Hermann
Figure 5.4.47 - Herpes labialis

Photo Credit: Donald E. Mansell, MD [CC BY-SA 3.0 (https://creativecommons.


org/licenses/by-sa/3.0)]
Figure 5.4.49 - HSV-related esophageal ulcers

Photo Credit: Grook Da Oger [CC BY-SA 3.0 (https://creativecommons.org/


licenses/by-sa/3.0)]
Figure 5.4.48 - Erythema multiforme
258

Photo Credit: SOA-AIDS Amsterdam [CC BY-SA 3.0 (https://creativecommons.


org/licenses/by-sa/3.0/)]
Figure 5.4.52 - Vesicular and ulcerative herpes
lesions

Photo Credit: Dr. Laughlin Dawes [CC BY 3.0 (https://creativecommons.org/


licenses/by/3.0)]
Figure 5.4.50 - MRI of temporal lobe encephalitis

Photo Credit: National Institute of Allergy and Infectious Diseases (NIAID)


[Public domain]
Figure 5.4.53 - Multinucleated giant cells on Tzanck
smear

Photo Credit: Kapitainekavern [CC BY-SA 4.0 (https://creativecommons.org/


licenses/by-sa/4.0)]
Figure 5.4.51 - Neonatal herpes

Photo Credit: Shaheen E. Lakhan and Lindsey Harle [CC BY 2.0 (https://
creativecommons.org/licenses/by/2.0)]
Figure 5.4.54 - Cowdry bodies in hepatocytes
259

REVIEW QUESTIONS ?
1. A 14-year-old boy is brought to the emergency
department by his mother due to confusion
that began earlier today. His mother states
that he became lost at school even though
he has been at the same school for years. On
physical examination, he is disoriented to time,
place, and person. An MRI of the brain reveals
necrosis and hemorrhage of the temporal lobes
bilaterally. Which of the following is most likely
responsible for this patient’s condition?
A. DNA virus, double-stranded, linear, naked
B. DNA virus, single-stranded, linear, naked
C. DNA virus, double-stranded, linear,
enveloped
D. RNA virus, single-stranded, linear, naked,
positive-sense
E. RNA virus, single-stranded, linear,
enveloped, positive-sense

• Correct answer: C
• This patient has HSV-1 temporal lobe
encephalitis
• Confusion
• MRI that has revealed bilateral necrosis
and hemorrhage of the temporal lobes
• HSV is a DNA virus, double-stranded,
linear, enveloped
• A is describing Adenovirus which is not
associated with temporal lobe encephalitis
(it’s associated with rhinorrhea, pharyngitis,
and conjunctivitis)
• B is describing Parvovirus B19  which is a
single stranded DNA virus
• D is describing Picornaviruses  such as
Coxsackievirus which can cause meningitis
but not temporal lobe encephalitis
• E is describing Flaviviruses which can cause
encephalitis but are unlikely to cause
bilateral temporal lobe damage
260

Section 2 - Varicella-zoster virus (VZV)

1. Zoo: Varicella zoster virus (VZV) 15. Girl coughing because of dirt: can cause pneumonia
2. Dark colors: DNA virus 16. Husband pouring water on wife’s head: can cause
3. Fighting animals with parallel antlers: double encephalitis
stranded 17. Torches: a TORCHES infection
4. Line of people leaving zoo: linear 18. Pregnant woman and little boy by torches:
5. Fog entering zoo: transmitted through respiratory congenital VZV
secretions 19. Scar on little boy’s cheek: congenital VZV can cause
6. Chickens: causes chickenpox (pruritic vesicular rash) dermatomal scarring
7. Tree trunk: rash starts on the trunk 20. Boy wearing sunglasses: congenital VZV can cause
8. Zeus action figure: a Tzanck smear will reveal blindness
multinucleated giant cells 21. Boy holding stuffed bear with no arms or legs:
9. Tree root: remains latent in ​the d
​ orsal root ganglia congenital VZV can cause limb hypoplasia
10. Three gems on Zeus’ belt: remains latent in the 22. Live animals sign: live vaccine
trigeminal ganglion 23. BANDAID, syringe, and “Max Occupancy 200” sign:
11. Monkey throwing shingles from roof: can cause the vaccine may be given to HIV patients when the
shingles CD4+ count is greater than 200
12. Woman hit with shingle in abdomen: presents as a 24. Old man under pavilion: the vaccine is
painful rash within a dermatomal distribution recommended for those > 60 years old
13. Man hit with shingle in eye: may cause herpes 25. Cyclone fan: treat with acyclovir
zoster ophthalmicus 26. Violet cyclone fan: treat with valacyclovir
14. Man hit with shingle in the foot: may cause
postherpetic neuralgia
261

REVIEW QUESTIONS ?
1. A 28-year-old male comes to the office due to
a rash that he first noticed yesterday. Physical
examination reveals a vesicular rash in a
dermatomal distribution on the patient’s trunk.
A viral infection is suspected. Which of the
following is an appropriate recommendation for
this patient’s condition?
A. Ganciclovir
B. Avoid contact with pregnant women
C. Supportive care
D. Prednisone

• Correct answer: B
• This patient has shingles caused by VZV
• Vesicular rash in a dermatomal
distribution
• Viral infection was suspected
Photo Credit: Mad Max [Public domain]
• VZV can be particularly devastating for
Figure 5.4.55 - Chickenpox pregnant women by harming the fetus
and causing congenital varicella →
patients who have an active infection
should avoid contact with pregnant
women
• A is incorrect because this is the treatment
for CMV
• C is incorrect because supportive care would
not be advised as this patient is presenting
early in the course of the infection and
Photo Credit: Elizgoiri [CC BY-SA 3.0 (https://creativecommons.org/licenses/ would likely benefit from treatment with
by-sa/3.0)]
acyclovir or valacyclovir
Figure 5.4.56 - Shingles • D is incorrect because glucocorticoids have
shown no clinical benefit in patients with
shingles

Photo Credit: Burntfingers [CC BY-SA 4.0 (https://creativecommons.org/


licenses/by-sa/4.0)]
Figure 5.4.57 - Herpes zoster ophthalmicus
262

Section 3 - Epstein-Barr Virus (EBV)

1. Elves Blue Village: Epstein-Barr virus 13. “No Contact Sports” sign: avoid contact sports for
2. Dark colors: DNA virus 3-4 weeks due to rupture of enlarged spleen
3. Female elf kissing booth: spread via saliva and 14. Bow shooting arrows at the stars: heterophile
kissing antibodies
4. Elf falling asleep after kiss: causes fatigue 15. Light spotting arrows in the sky: monospot positive
5. Elf pulling on collar: causes pharyngitis 16. Arrow through nose of the villain: can cause
6. Elf sticking out tongue with hairs: can cause oral nasopharyngeal B-cell carcinoma
hairy leukoplakia 17. Net with knots trapping elves with pink cancer
7. Hair curls behind ear and on neck: posterior ribbons: can cause lymphoma
auricular and cervical lymphadenopathy 18. Elf holding bucket full of arrows: can cause Burkitt’s
8. Cane with icosahedral gem: icosahedral capsid lymphoma (B-cell lymphoma)
9. BANDAID and “<500 kisses remain” sign: oral hairy 19. Sky full of stars: starry sky appearance on biopsy of
leukoplakia occurs at CD4+ levels of <500 Burkitt’s lymphoma
10. Two ribbons hanging from lookout tower: double 20. Villain shocking brain with electric rings: can cause
stranded DNA virus primary central nervous system lymphoma which
11. Line for kissing booth: linear virus exhibits ring-enhancing lesions on imaging
12. Fighting with T-shaped flails: cytotoxic T cells seen 21. Elf holding up $100 bill: primary central nervous
on peripheral smear system lymphoma occurs in HIV when CD4+ cells are
<100
263

REVIEW QUESTIONS ?
1. A 15-year-old obese girl presents to the clinician
with a fever and a sore throat. Upon further
discussion, the girl states that she has felt tired
for the past several days. She has felt healthy
prior to this illness and does not complain of
any other symptoms. Physical examination does
Photo Credit: Ed Uthman via Flickr not reveal any noticeable organomegaly in the
Figure 5.4.58 - Cytotoxic T-cells on EBV-related left upper quadrant of her abdomen. Her blood
peripheral blood smear is drawn and a lab test confirms the presence of
heterophile antibodies. Which of the following
is most likely true about her condition?
A. The patient had a sexual encounter before
acquiring the infection
B. A double-stranded RNA virus is the likely
pathogen
C. The patient may have a lymphocytic
neoplasm in her brain
D. She should avoid contact sports for at least
3 weeks

• Correct answer: D
• The patient likely has an EBV infection
based on the following findings:
• Fever, sore throat, fatigue
• Positive heterophile (monospot)
test
• Though this patient does not have
evidence of splenomegaly, the risk of
splenomegaly is still high enough that
Photo Credit: Louis M. Staudt by NIH [Public domain]
she should be advised to avoid contact
Figure 5.4.59 - Starry sky appearance on biopsy of sports for at least 3 weeks
Burkitt’s lymphoma • Choice A is incorrect because EBV is not
sexually transmitted
• Choice B is incorrect because EBV is a
double-stranded DNA virus
• Choice C is incorrect because EBV does
not cause primary central nervous system
lymphoma in immunocompetent patients

Photo Credit: Aimun AB Jamjoom, Abrar R. Waliuddin and Abdulhakim B.


Jamjoom [CC BY 2.5 (https://creativecommons.org/licenses/by/2.5)]
Figure 5.4.60 - MRI showing ring-enhancing lesion
264

Section 4 - Cytomegalovirus (CMV)

1. Megatronix: Cytomegalovirus (CMV) 11. Torches on balcony: a TORCHES infection


2. Dark colors: DNA virus 12. Mom eating blueberry muffin: congenital CMV may
3. Two parallel ladders: double stranded cause a “blueberry muffin” rash
4. Straight road with dashed lines down the middle: 13. Lazy Dalmatian dog with liver spot:
linear hepatosplenomegaly
5. Sleeping robot with bow and T-shaped flail near 14. Baby wearing earmuffs: congenital CMV may cause
nuclear power plant: CMV becomes latent in B cells, deafness
T cells, and mononuclear cells 15. Parrot causing milk to spill on baby’s head:
6. Optimum prime scraping lines in building over congenital CMV may cause periventricular
50mph sign: increased risk of linear esophagus calcifications
ulcers in HIV patients when the CD4+ <50 cells/ 16. Baby holding stuffed penguin: congenital CMV may
microL cause microcephaly
7. Laser beams from eyes: increased risk of retinitis in 17. Mother wearing sunglasses: congenital CMV may
HIV patients when the CD4+ <50 cells/microL cause chorioretinitis
8. Gutter going up in flames: increased risk of colitis in 18. Broken spotlight: negative monospot test
HIV patients when the CD4+ <50 cells/microL 19. Halos around laser beam: infected cells may show
9. Exploding fire hydrant pouring water on man’s head: intranuclear and intracytoplasmic inclusions with a
increased risk of encephalitis in HIV patients when surrounding halo (owl’s eyes)
the CD4+ <50 cells/microL 20. Fast car: treat with foscarnet
10. Lung-shaped axe and transplant guy: may cause 21. Garbage can: ganciclovir
pneumonitis in transplant patients 22. Violet garbage can: valganciclovir
265

Photo Credit: CDC/Dr. Edwin P. Ewing, Jr. [Public domain]


Figure 5.4.64 - Owl’s eye appearance on lung biopsy
Photo Credit: https://ww.nei.nih.gov/photo/eyedis/index.asp [Public domain]
Figure 5.4.61 - CMV retinitis
REVIEW QUESTIONS ?
1. A 24-year-old female presents to her physician
for results regarding a monospot test. A week
ago she developed pharyngitis and cervical
lymphadenopathy so a monospot test was
performed and the results were negative. She
is 19 weeks pregnant with her second child
and is concerned about the implications of the
test. The physician suspects that her illness
was caused by a double-stranded, linear, DNA
virus. He counsels her that the unborn child
is at greatest risk for developing which of the
following?
Photo Credit: CDC [Public domain]
Figure 5.4.62 - “Blueberry muffin” rash A. Retinitis pigmentosa
B. Patent ductus arteriosus
C. Vesicular lesions on the scalp
D. Hepatosplenomegaly

• Correct answer: D
• This woman had a CMV infection during
pregnancy
• Pharyngitis
• Cervical lymphadenopathy
• Negative monospot test
• Double-stranded, linear, DNA virus
• 19 weeks pregnant so a CMV infection
during pregnancy puts the unborn child
at risk of developing congenital CMV →
hepatosplenomegaly
• A is incorrect because congenital CMV
can cause chorioretinitis but not retinitis
pigmentosa
• B is a reference to congenital Rubella, but
this is a single stranded RNA virus
Photo Credit: Adriano R. Tonelli1, Kalyan Kosun1, Sainan Wei2 and Davoren • C is a reference to congenital HSV, but this
Chick1 [Modified] would present with a painful vesicular rash
Figure 5.4.63 - Periventricular calcifications on CT on the genitals in the mother
imaging
266

Section 5 - Roseola (HHV-6 and HHV-7)

1. Creepy baby doll holding roses: Roseola 6. Line of slobber: linear virus
2. Dark colors: DNA virus 7. Two strands of yarn: double-stranded
3. Char on baby from head to toe: widespread 8. “666” on side of trunk: HHV-6
maculopapular rash 9. Bulletin board with “7 deadly sins” list: HHV-7
4. Oil lamp causing tall flames: high fevers 10. Shaking telephone: seizures
5. Salivating dog knocking over lamp: transmitted
through saliva
267

REVIEW QUESTIONS ?
1. A 13-month-old girl presents to her pediatrician
with a red “rash” all across her body for 1 day.
Her temperature in the clinic is 37.9 C and
the patient appears mildly uncomfortable.
Upon further history, the father explains the
patient had a temperature of 39.9 C at home
4 days ago. Around this time the patient
also demonstrated a 30 second period of
unresponsiveness, uncontrollable shaking
movements and open eyelids. The physician
suspects a viral infection. Which of the following
is most likely true regarding the virus?
A. It is an RNA virus
B. It is a naked virus
C. The genome is circular
D. It is double-stranded

• Correct answer: D
• The patient likely has a Roseola
infection (HHV-6) based on the
following:
• High fever
• Seizure followed by a rash 3 days
later
• HHV-6 is double-stranded
• Choice A is incorrect because HHV-6 is a
DNA virus, not an RNA virus
• Choice B is incorrect because HHV-6 is
enveloped, not naked
• Choice C is incorrect because HHV-6 is has a
linear genome, not circular
268

Section 6 - Kaposi sarcoma (HHV-8)

1. Dark colors: DNA virus 5. Cancer ribbons near dead camels: Kaposi sarcoma
2. Bones show that sirens ate the sailors: HHV-8 6. Stretcher: immunocompromised patients vulnerable
3. Sailors lined up on boat: linear virus to Kaposi sarcoma
4. Red and purple lesions on evil sirens: dark purple 7. Two lines of dirt caused by stretcher: double
skin plaques formed by endothelial neoplasia stranded
269

REVIEW QUESTIONS ?
1. A 35-year-old male presents with dark purple
plaques all over his hands and arms. He states
the rash started a few months ago on his hands
and has continued to spread up his arms. The
patient endorses a 15-year history of having
regular, unprotected sexual intercourse with
prostitutes. A lymphocyte CD4+ count reveals
a level of 350 cells/mm3. Biopsy of a skin
lesion reveals numerous lymphocytes. Gram
  staining and blood cultures do not grow any
Photo Credit: National Cancer Institute, AV-8500-3620 [Public domain] organisms. Which of the following is most
Figure 5.4.65 - Skin lesions in Kaposi’s sarcoma likely true regarding the cause of this patient’s
presentation?
A. The purple plaques are caused by a single-
stranded virus
B. Healthy patients can acquire the infection
with similar symptoms through direct
contact with the skin plaques
C. A gram negative organism caused the skin
findings
D. He would likely be asymptomatic if his CD4+
count was 1,200 cells/mm3

• Correct answer: D
• Kaposi sarcoma is likely based on the
following:
• Purple plaques that have spread
• CD4+ count of 350 cells/mm 3

• HHV-8 does not cause symptoms


in patients with adequate immune
systems (eg, patients with CD4+ counts
> 1,000 cells/mm 3

• Choice A is incorrect because HHV-8 is


double-stranded
• Choice B is incorrect because skin plaques
will not infect immunocompetent patients
• Choice C is incorrect because a sample
would likely have shown Bartonella
hensalae, a gram negative organism known
to cause bacillary angiomatosis in AIDS
patients
270

Section 7 - Hepatitis B Virus (HBV)

1. Cows with liver spots being attacked by killer bees: vaccination


Hepatitis B virus 15. Woman wearing scarlet letter: transmission through
2. Hip pads: hepadnavirus sexual intercourse
3. Arrows shot into the beehive surface: Anti-HBs 16. Pregnant woman: transplacental transmission
antibodies confer immunity 17. Woman with IV drip: transmission through IV drug
4. Surface antigens injected through needle: the use
vaccine contains HBs antigens 18. Long branch with bird’s nest full of eggs: long
5. Partially double-file circle of men: circular and incubation period
partially double-stranded genome 19. Cow with liver spot stuck in gnarled tree branches:
6. Toxic green glowing goop on T-shaped flail: cytotoxic chronic infection can cause liver cirrhosis
T-cells cause liver damage 20. Pink ribbon around cow: cirrhosis can lead to
7. Shards of glass on the ground: liver biopsy shows hepatocellular carcinoma
ground glass appearance 21. Pink ribbon around cow not stuck in the tree:
8. Icosahedral-shaped core: icosahedral capsid chronic hepatitis B can lead to hepatocellular
9. Bees actively attacking man stealing gems at the carcinoma even in the absence of cirrhosis
core of the beehive: HBcAg indicates active infection 22. Bee unable to lift cow out of stable: chronic and
10. Old man among gems: Anti-HBc indicates old, stable infections possible
resolved infection 23. Dark, dead cow: fulminant liver failure with necrosis
11. Mountain of gems at core with “e” shaped bee is possible during acute phase
larvae: HBeAg indicates high viral replication and 24. Cows with scratches and scrapes: acute hepatitis
infectivity can cause rashes and arthralgia
12. Archer attacking “e” shaped larvae: Anti-HBe 25. Oil lamp hanging from tree branch: acute hepatitis
antibodies indicate low replication and infectivity can cause fever
13. Man stealing gems getting actively attacked by a 26. Cow drenched in honey: acute hepatitis can cause
bee: HBsAg indicates active infection jaundice
14. Marksman pinning bee to surface of hive near gems: 27. Unharmed cow and happy boy: acute hepatitis can
Anti-HBs antibodies indicate resolved infection or be asymptomatic
271

28. Lots of cows with rash and arthralgia: acute 32. Kidney beans and arrows blocking rain
hepatitis with resolution is the most common gutter: associated with membranous
outcome glomerulonephropathy
29. Unharmed cow being carried by bee: carrier state is 33. Frayed, growing rain gutter: associated with
common membranoproliferative glomerulonephropathy
30. Red ropes around man’s abdomen: associated with 34. Water well with one rope becoming two ropes:
polyarteritis nodosa hepatitis B has reverse transcriptase
31. Balloons of various colors popped while still inside 35. Sheep dogs rescuing cows: hepatitis D relies on
net: associated with aplastic anemia hepatitis B virus to infect host cells

Figure 5.4.66 - HBV structure

Serologic Marker Description and Role


• Surface glycoprotein
HBsAg • Present in acute infection
• If present > 6 months → infection is considered chronic
• HBV polypeptide between core and surface
• Present in acute infection as well as early chronic patients
HBeAg
• Indicates high viral replication
• Indicates patient is highly contagious
• Present in acute infection
Anti-HBc IgM
• Present in window phase
• Indicates immunity
Anti-HBs • Present in recovered patients
• Present in vaccinated patients
• Indicates low viral replication
Anti-HBe • Indicates patient is not very contagious
• Present in recovered patients as well as late chronic patients

Anti-HBc IgG • Present after ~6 months in both chronic or fully recovered patients

Table 5.4.1 - Descriptions of serologic markers in Hepatitis B


272

Figure 5.4.67 - Serological markers in acute hepatitis B infection

Figure 5.4.68 - Serological markers in chronic hepatitis B infection


273

REVIEW QUESTIONS ?
1. A 35-year-old male presents with dark purple
plaques all over his hands and arms. He states
the rash started a few months ago on his hands
and has continued to spread up his arms. The
patient endorses a 15-year history of having
regular, unprotected sexual intercourse with
prostitutes. A lymphocyte CD4+ count reveals
a level of 350 cells/mm3. Biopsy of a skin
lesion reveals numerous lymphocytes. Gram
staining and blood cultures do not grow any
Photo Credit: Nephron [CC BY-SA 3.0 (https://creativecommons.org/licenses/ organisms. Which of the following is most
by-sa/3.0)]
likely true regarding the cause of this patient’s
Figure 5.4.69 - H&E stain showing ground glass presentation?
appearance
A. The purple plaques are caused by a single-
stranded virus
I. Clinical outcomes of Hepatitis B (heading) B. Healthy patients can acquire the infection
A. Acute infection that resolves within 6 months with similar symptoms through direct
(most common) contact with the skin plaques
C. A gram negative organism caused the skin
1. May have arthralgias, rash, fever, jaundice findings
2. May be asymptomatic (anicteric) D. He would likely be asymptomatic if his CD4+
count was 1,200 cells/mm3
B. Fulminant liver failure and hepatic necrosis
C. Chronic infection (stable) • Correct answer: C
• Vaccinated patients should have
D. Chronic infection → cirrhosis antibodies against the HBsAg
E. Chronic infection → cirrhosis → hepatocellular • Anti-HBc antibodies are only present in
carcinoma previously infected patients
• HBeAg is only present in patients during
F. Chronic infection → hepatocellular carcinoma an active infection
• Choice A is incorrect because this patient
should be negative for anti-HBc and HBeAg
• Choice B is incorrect because this patient
should be positive for anti-HBs and negative
for anti-HBc

Figure 5.4.70 - Reverse transcriptase in HBV


274

Section 8 - Smallpox, Cowpox, and Molluscum contagiosum

1. Boxes: pox viruses 8. Cow being milked: Cowpox (“milkmaid blisters”) is a


2. Blue background: DNA virus poxvirus
3. Boxes outside of sandbox: replication in cytoplasm 9. Two parallel slides: double stranded
4. Small boxes: smallpox virus is a poxvirus 10. Brick in sand box: brick-shaped
5. Child burying boxes: smallpox virus has been 11. Kids waiting in line for milk: linear
eradicated 12. Map of the world: largest DNA viruses in the world
6. Sandy slug: molluscum contagiosum is a pox virus 13. Live animals sign: live attenuated vaccine
7. Deep hole in sand next to slug: Molluscum
contagiosum causes umbilicated skin lesions
275

Photo Credit: Gzzz [CC BY-SA 4.0 (https://creativecommons.org/licenses/by-


sa/4.0)], from Wikimedia Commons
Figure 5.4.72 - Umbilicated lesions in Molluscum
contagiosum

Photo Credit: https://en.wikipedia.org/wiki/File:Child_with_Smallpox_


Bangladesh.jpg
Figure 5.4.71 - Smallpox

REVIEW QUESTIONS ?
1. A 4-year-old boy is brought to the physician • Correct answer: C
due to a 3-day history of a rash. The boy’s • This boy has an infection caused by
mother states that she first noticed the rash Molloscum contagiosum
when she picked him up from daycare. Physical • Small papules with central areas of
examination of the left arm reveals clusters of depression (umbilicated lesions)
small papules with central areas of depression. • Molloscum contagiosum is enveloped
Which of the following is most likely true • A is wrong because poxviruses are DNA
regarding the causal organism? viruses
• B is wrong because poxviruses are double
A. It is a RNA virus
stranded
B. It is single stranded
C. It is enveloped • C is wrong because poxviruses replicate in
D. It replicates in the nucleus of infected cells the cytoplasm of infected cells
276

Section 9 - Human papillomavirus

1. Pepsi: Human papillomavirus (HPV) 10. Baby eating ladybugs near rows representing
2. Blue colors: DNA virus serotypes 6 and 11: serotypes 6 and 11 can cause
3. Grape vines wrapped in circle: circular genome laryngeal papillomas
4. Two parallel grape vines: double-stranded 11. Indy 500 tattoo and Band-Aid: HIV patients have an
5. Naked baby: naked virus increased risk of anal and cervical cancer when CD4
6. Cauliflower row with 1 plant, row with 2 plants, row cell count <500
with 6 plants, and row with 2 tall plants: serotypes 12. “7” shaped straw: serotypes 16 and 18 produce E7
1, 2, 6, and 11 are associated with warts (a protein associated with cancer)
7. Car keys, “I voted” sticker, and woman with cancer 13. 6 pack of Pepsi: serotypes 16 and 18 produce E6 (a
hope ribbon near crotch: serotypes 16 and 18 are protein associated with cancer)
associated with cervical cancer 14. Rotten tomatoes with big brown spots near
8. Barefoot guy picking cauliflower in first two rows: rows that represent serotypes 6, 11, 16, and 18:
serotypes 1 and 2 are associated with verrucae on Koilocytes may be seen in patients with an HPV
the hands and feet infection from serotypes 6, 11, 16, or 18
9. Cucumbers next to rows that represent serotypes 15. Syringe shaped sign pointing towards serotypes 6,
6 and 11: serotypes 6 and 11 cause condyloma 11, 16, and 18: the Gardasil vaccine can prevent
acuminatum infection from serotypes 6, 11, 16, and 18
277

Photo Credit: Marionette [CC BY-SA 3.0 (https://creativecommons.org/


livenses/by-sa/3.0)]
Figure 5.4.73 - Plantar warts

Photo Credit: Colorectal at Greek Wikipedia Aπόστολος Σταματιάϭης [CC BY-


SA 3.0 (https://creativecommons.org/licenses/by-sa/3.0)]
Figure 5.4.74 - Anal warts
278

Figure 5.4.75 - HPV carcinogenesis mechanism

Photo Credit: Photomicrograph by Ed Uthman, MD, 20 July 2006 Euthman


20:24, 29 November 2006 (UTC) [Public domain]
Figure 5.4.75 - Koilocytes on Pap smear
279

REVIEW QUESTIONS ?
1. A 5-year-old boy is brought to the physician
due to shortness of breath and wheezing that
began 20 minutes ago. Physical examination
is significant for nasal flaring and subcostal
retractions. Laryngoscopy reveals a benign
laryngeal tumor occluding the airway. The
tumor is excised and a pathological report states
that the histological changes were caused by
a virus. Which of the following is most likely
responsible for this patient’s condition?
A. RNA virus, single-stranded, positive-sense
B. RNA virus, single-stranded, negative-sense
C. DNA virus, double-stranded, non-enveloped
D. DNA virus, double-stranded, enveloped
E. DNA virus, single-stranded, non-enveloped

• Correct answer: C
• This patient has a laryngeal papilloma
caused by HPV resulting in a compromised
airway
• Shortness of breath, wheezing, and a
benign laryngeal tumor occluding the
airway
• The changes were caused by a virus
• HPV is a DNA virus, double-stranded,
and non-enveloped
• A and B are incorrect because most RNA
viruses are single-stranded and either
positive or negative sense, but a laryngeal
papilloma in a child is a unique finding to
HPV which is a DNA virus
• D is incorrect because most DNA viruses are
double-stranded and many are enveloped
such as HSV and poxviruses, but these are
not associated with laryngeal papillomas
• E is incorrect because the only DNA virus
that is single stranded is parvovirus, but is
not associated with laryngeal papillomas
280

Section 10 - Adenovirus

1. Adam: Adenovirus 8. Adam with red eyes: can cause conjunctivitis


2. Blue colors: DNA virus 9. Adam coughing: can cause pneumonia
3. Two serpents slithering up the tree: double 10. Eve wearing handkerchief: can cause pharyngitis
stranded 11. Watery river: can cause watery diarrhea
4. Line of people waiting for seats: linear 12. Raspberry patches: may cause Peyer patch
5. Naked Adam and Eve: naked (non-enveloped) hypertrophy
6. Crowded audience wearing camouflage: outbreaks 13. Telescope: can cause intussusception
in crowded quarters (eg, military recruits) 14. Muscly bouncer guy with car tattoo: can cause
7. Juicy pomegranate full of seeds: hemorrhagic myocarditis
cystitis 15. Live show sign shaped like a syringe: live vaccine
281

REVIEW QUESTIONS ?
1.  A 3-year-old boy is brought to the emergency
department due to sudden onset abdominal
pain. The episodes are severe but intermittent
and occur approximately every 15-20 minutes.
The mother also states that just before coming
to the hospital she saw red stools in the toilet
after her son had a bowel movement. After
appropriate management the family is informed
that the patient’s condition was most likely due
to a viral infection. This causal organism is also
associated with which of the following?
A. A purpuric rash
B. Aplastic anemia
C. Nasopharyngeal carcinoma
D. Hemorrhagic cystitis

• Correct answer: D
• This boy’s presentation is due to
intussusception caused by Adenovirus
• He is 3 years old and intussusception is
most common in young children
• He has had intermittent episodes of
abdominal pain and red stools
• His condition was most likely due to
a viral infection, and intussusception
commonly occurs following a viral
infection (most commonly Adenovirus)
• Adenovirus is also associated with
hemorrhagic cystitis
• A is a reference to Henoch-Schönlein
purpura which is a small-vessel vasculitis
that is associated with intussusception, but
this not a viral infection
• B is a reference to Parvovirus B19 which
can cause aplastic anemia in sickle cell
patients but this is not a typical feature of
adenovirus
• C is a reference to EBV which can cause
pharyngitis and posterior cervical
lymphadenopathy but isn’t typically
associated with intussusception
282

Section 11 - JC Virus and BK Virus

1. Polly the bird: Polyomavirus 9. Juicy pomegranate full of seeds: BK virus causes
2. Burger King and JC Penney signs: BK virus and JC hemorrhagic cystitis
virus are polyomaviruses 10. Sleeping employee: JC virus becomes latent
3. Blue colors: DNA virus 11. Girl with a band-aid on her arm: JC virus is
4. Circular shaped Christmas lights decorating an arch: commonly associated with AIDS patients
circular DNA structure 12. Receipt for $200: reactivation doesn’t occur until
5. Parallel pine decorations: double-stranded DNA CD4+ cell count <200
genome 13. Polka dot veil: JC virus causes progressive multifocal
6. Naked mannequins: naked virus leukoencephalopathy
7. “Try our new kidney bean veggie burger” sign: BK 14. MRI in front of sleeping employee: MRI shows non-
virus targets the kidneys enhancing areas of demyelination
8. Butcher with meat: BK virus infects transplant 15. Nat King Cole CD’s: Natalizumab increases risk of
patients PML
283

REVIEW QUESTIONS ?
1. A 27-year-old female comes to the office due
to a 3-day history of dysuria and hematuria.
One year ago she underwent major surgery for
a left kidney transplant. Physical examination
is unremarkable. Laboratory evaluation reveals
an elevated serum creatinine. A kidney biopsy
reveals intranuclear basophilic viral inclusions
without a surrounding a halo. PCR analysis
of the patient’s plasma confirms that her
Photo Credit: Hellerhoff [CC BY-SA 3.0 (https://creativecommons.org/ symptoms are due to a viral infection. What is
licenses/by-sa/3.0)], from Wikimedia Commons
the most likely diagnosis?
Figure 5.4.77 - Progressive multifocal
leukoencephalopathy (PML) on MRI A. CMV
B. BK virus
C. JC virus
D. Delta virus

• Correct answer: B
• This patient has hemorrhagic cystitis due to
a BK virus infection
• Dysuria and hematuria after a left
kidney transplant → hemorrhagic
cystitis
• BK virus commonly targets the kidneys
and causes hemorrhagic cystitis in
kidney transplant patients
• A is incorrect because the biopsy revealed
intranuclear basophilic viral inclusions
without a surrounding halo (CMV can
result in infection among transplant
patients but a biopsy would have revealed
the characteristic “owl eye” intranuclear
inclusions)
• JC virus is a polyomavirus just like BK virus
but it causes PML in AIDS patients
• D is referring to hepatitis D virus (commonly
associated with hepatitis B coinfections)
284

Section 12 - Parvovirus B19

1. Parfaits: Parvovirus B19 10. Girl wearing lacy shirt: causes a lacy, reticular rash
2. Blue colors: DNA virus on the trunk and extremities
3. Hummingbird (small bird): smallest DNA virus 11. Kid popping balloons with a sickle: can cause
4. Kids lined up to see bird: linear aplastic crises in sickle cell patients
5. Silly string: single stranded 12. Muscle man in car: can cause myocarditis
6. Skinny dippers: naked 13. Basset hound: can precipitate behçet syndrome
7. Steam from hot tub: transmitted through 14. Woman reaching for toy with sore knees: can cause
respiratory droplets arthralgias and arthritis in adults
8. Red balloons in enclosed area: replicates in red 15. Torch: a A TORCHES infection
blood cell precursor cells in bone marrow 16. Pregnant woman in hot tub: pregnant woman can
9. Bully slapping child with plastic hand: causes fifth contract the disease
disease (erythema infectiosum) which presents with
an erythematous rash on the cheeks (“slapped-
cheek”)
285

REVIEW QUESTIONS ?
1. A 9-year-old boy with a history of sickle
cell disease is brought to the emergency
department due to shortness of breath.
Several days ago he developed a fever, malaise,
and myalgias. Physical examination reveals
conjunctival pallor. Laboratory analysis is
significant for a hematocrit of 27% (normal:
40%) and a reticulocyte count of 0.2% (normal:
0.5% - 1.5%). The medical team concludes that
this patient’s condition is due to a pathogen
that most likely replicates in which of the
following cell types?
A. B cell precursors
B. Erythrocyte precursors
C. T cell precursors
D. Macrophage precursors

Photo Credit: Andrew Kerr [Public domain] • Correct answer: B


Figure 5.4.78 - Fifth disease • This boy has an aplastic crisis caused by
Parvovirus B19
• History of sickle cell disease
• Developed a fever, malaise, and
myalgias which was likely due to a viral
infection
• His presentation at the ED has shown
conjunctival pallor → anemia
• Laboratory results showing a decreased
hematocrit and reticulocyte count (the
bone marrow is unable to adequately
produce reticulocytes)
• Parvovirus B19 replicates in erythrocyte
precursors in the bone marrow,
resulting in abnormal erythrocyte
production and a transient aplastic crisis
• Many viruses replicate in lymphocytes such
as HIV but these are not associated with
aplastic crises (A and B)
• Bacteria such as TB replicate in
macrophages (D)
286

Section 13 - Coronavirus and SARS

1. Warm colors: RNA virus 5. Corkscrew bottle opener: helical capsid


2. Rainbow: positive sense 6. SARS shirt: causes severe acute respiratory
3. Sick, cold man: common cold virus syndrome (SARS)
4. Corona beer: coronavirus 7. Man out of breath: respiratory distress
287

REVIEW QUESTIONS ?
1. A 15-year-old girl presents to the physician
with a mild headache and nasal congestion for
2 days. She states that everyone in her family
has had similar symptoms recently. On physical
examination, her throat appears red with visible
mucus drainage. Her temperature is 37.8 C
and she appears mildly uncomfortable. A rapid
streptococcal antigen test is negative and a viral
PCR panel is negative for rhinovirus. Which of
the following statements is true regarding the
most likely causal organism?
A. The pathogen carries its own DNA
polymerase
B. The capsid has an icosahedral shape
C. The pathogen has a lipid bilayer surrounding
its genome
D. The genetic material is double-stranded

• Correct answer: C
• The patient most likely has an infection
with coronavirus because of the
following:
• Pharyngitis, nasal congestion,
headaches, and mucus drainage
• No evidence of S. pyogenes
• No evidence of rhinovirus
• Coronavirus is enveloped
• Choice A is incorrect because negative sense
viruses carry their own RNA polymerase,
not positive sense viruses
• Choice B is incorrect because coronavirus
has an icosahedral capsid
• Choice D is incorrect because coronavirus is
a single-stranded RNA virus
288

Section 14 - Human immunodeficiency virus (HIV)

1. Woman with wounds covered by Band-Aids: Human building: gp41 is associated with viral fusion and
immunodeficiency virus (HIV) entry into host cell
2. Red and orange colors: RNA virus 17. Group 120 soldiers climbing ladder and pulling
3. Pulley: retrovirus (reverse transcriptase) down servant helper with the number “4” on his
4. One red rope splitting into two blue ropes: reverse shirt: gp120 binds to CD4 receptor
transcriptase converts a single strand of RNA into 18. Scarlet letter: sexually transmitted
double stranded DNA 19. T-shaped flail held by a sexy girl wearing “CXR4”:
5. Rainbow: positive sense virus infects CD4 T cells using the CXCR4 coreceptor
6. Line of soldiers: linear 20. Guy playing guitar with CCR5 shirt on top of cage:
7. Mirror: has a diploid genome (two copies of RNA) infects macrophages using the CCR5 coreceptor
8. Captured dwarf eating ice cream cone: cone-shaped 21. CCR band member resisting opposing soldiers: CCR5
capsid coreceptor mutations confer resistance or immunity
9. Gagged dwarf: the gag gene encodes for the capsid to HIV
(p24) 22. Servant suffocating and holding neck: the initial
10. Wizard casting a matrix spell with the number 17: infection may present with pharyngitis
the gag gene encodes for the matrix (p17) 23. Lamp: the initial infection may present with a fever
11. Metal poles near pulley: the pol gene encodes for 24. Green beaded net: the initial infection may present
reverse transcriptase with lymphadenopathy
12. Metal poles near scissors: the pol gene encodes for 25. Sleeping soldier: there is a period of latency
aspartate protease following the initial infection
13. Metal poles near dwarf tying ropes together: the pol 26. "200 feet drop” sign: AIDS is diagnosed when the
gene encodes for integrase CD4+ cell count is <200
14. “Envious” on dress: the env gene encodes for gp160 27. Person trapped in green net, with shocked brain,
which becomes gp120 and gp41 and wearing a cancer ribbon: can cause primary
15. 160 soldiers split into a group of 120 and a group of central nervous system lymphoma (imaging may
41: gp160 is cleaved to form gp120 and gp41 show ring-enhancing lesions)
16. Group of 41 soldiers using battering ram to enter 28. Torches: a TORCHES infection
289

29. Soldier wearing belt made of gems and pointing 33. Pregnant dwarf lady with sitting dove: pregnant
arrow at dwarf holding sundial: diagnosed with a women with HIV should receive zidovudine
combined immunoassay (detects HIV antibodies 34. Two soldiers inhibiting dwarf from using well, one
and the p24 antigen) followed by a differentiation soldier inhibiting dwarf from tying the ropes: the
immunoassay (detects HIV antibodies) initial regimen for HIV typically includes 2 NRTIs and
30. Wheelbarrow full of Band-Aids: a viral load may be an integrase inhibitor
obtained using NAAT 35. Soldier next to 200 sign holding meth crystals:
31. Crossed out baby sign next to immunoassay soldier: TMP-SMX prophylaxis is administered when the CD4
the antibody/antigen immunoassay test should not count drops below 200
be performed on babies (only use viral load) 36. Scarecrow with U.S. map of 50 states shirt:
32. Skull on front of wheelbarrow: a high viral load is macrolide prophylaxis is administered when the CD4
associated with a poor prognosis and may be lethal count drops below 50

Figure 5.4.79 - HIV structure


290

Figure 5.4.80 - HIV replication cycle

Figure 5.4.81 - Time course of HIV infection


291

Figure 5.4.82 - HIV diagnosis

REVIEW QUESTIONS ?
1. A novel drug is being developed to treat HIV. • Correct answer: C
An in vitro experiment is performed by adding • Reasons why C is correct:
HIV on a medium containing the novel drug and • The novel drug makes it so the virus
isolates of cultured human CD4 T lymphocytes. is unable to attach to the cellular
Further analysis reveals that in the presence membrane of CD4 T lymphocytes
of the drug the virus is unable to attach to the • gp120 interacts with the CD4 receptors
cellular membrane. Which of the following is which allows the virus to attach to the
the most likely target of this drug? cell membrane
• A is incorrect because CXCR4 is the
A. CXCR4
coreceptor on CD4 T lymphocytes that
B. CCR5
C. gp120 interacts with gp120 and induces a
D. gp41 conformational change in gp120 (viral entry
into the host cell)
• B is incorrect because CCR5 is similar to
CXCR4 but is present on macrophages
instead of T lymphocytes
• D is incorrect because gp41 is responsible
for fusion and entry (the virus can still bind
to the cell membrane without this receptor
- it just can’t enter the host cell)
292

Section 15 - Human T-lymphotropic virus (HTLV)

1. T-shaped flail in knight’s hand: Human 11. Medicinal vials: associated with IV drug use
T-lymphotropic virus 12. Old guy with cane: HTLV can cause slowly
2. Retrograde pulley: retrovirus (reverse transcriptase) progressive weakness
3. Warm colors: RNA virus 13. West African guy with bottle of milk: adult T-cell
4. Line of people: linear and single stranded lymphoma can cause hypercalcemia
5. No naked people: enveloped 14. Man with skin lesion: adult T-cell lymphoma causes
6. Mirror: two copies of RNA cutaneous lesions
7. Icosahedral-shaped water containers: icosahedral- 15. Lung-shaped axe: adult T-cell lymphoma can cause
shaped capsid pulmonary infiltrates
8. Rainbow: positive sense 16. Well made of bones: adult T-cell lymphoma can
9. Cancer ribbon on t-shaped flail covered in blood: cause lytic bone lesions
causes adult T-cell leukemia-lymphoma
10. Japanese samurai, west African guy, & pirate from
the Caribbean: more prevalent in Japan, West
Africa, and the Caribbean
293

REVIEW QUESTIONS ?
1. A 44-year-old male comes to the office due to a
rash. His medical history is significant for recent
travel to the Caribbean for a business-related
trip. Physical examination reveals a generalized
papular rash and lymphadenopathy. Laboratory
analysis is significant for hypercalcemia and
an elevated white blood cell count. Imaging
reveals lytic bone lesions. The pathogenesis of
this patient’s condition is thought to be due to
constitutive activation of the JAK-STAT pathway
and inactivation of the tumor suppressor
TP53. Which of the following is most likely true
regarding this patient’s condition?
A. It’s caused by a single stranded linear RNA
virus
B. It’s caused by a single stranded linear DNA
virus
C. It’s caused by a single stranded circular RNA
virus
D. It’s caused by a single stranded circular DNA
virus

• Correct answer: A
• This patient has adult t-cell leukemia-
lymphoma caused by HTLV
• Recent travel to the Caribbean
• A rash
• Hypercalcemia
• Lytic bone lesions
• Abnormalities of signaling pathways
that are classically associated with
cancer including the JAK-STAT pathway
and TP53
• HTLV is a single stranded linear RNA
virus
• B is incorrect because the only single
stranded DNA virus is Parvovirus B19 and
this is typically associated with aplastic
crises in sickle cell disease
• C is describing the RNA structure of
Bunyaviruses which are not associated with
adult T-cell leukemia-lymphoma
• D is a red herring because there isn’t a
single stranded circular DNA virus (DNA
viruses are all double stranded except
Parvovirus B19 and the DNA viruses that are
circular are also double stranded)
294

Section 16 - Hepatitis C Virus (HCV)

1. “C” shaped crop circle: Hepatitis C 18. Man eating lead paint: treat with ledipasvir
2. Warm color scheme: RNA virus 19. Burn marks on alien: Hepatitis C infection is
3. Rainbow: positive sense associated with porphyria cutanea tarda
4. Icosahedral lights on spaceship: icosahedral capsid 20. NHL no T.V.: Hepatitis C infections are associated
5. Cow with liver spot stuck in tree branches: chronic with non-hodgkin lymphoma (NHL)
infection leads to liver cirrhosis 21. Antibody-shaped antennas: Hepatitis C is associated
6. Pink cancer ribbon: cirrhosis can progress to with cryoglobulinemia (antibodies clogging small
hepatocellular carcinoma vessels)
7. Bird’s nest on long branch: long incubation period 22. Crying girl: Hepatitis C associated with antibodies
8. Alien with yellow skin: causes jaundice clogging small vessels (cryoglobulinemia)
9. Line in grass: linear virus 23. Red balloons popped by antenna: Hepatitis C is
10. Man carrying oil lamp: causes fever associated with autoimmune hemolytic anemia
11. IV drip: transmission through IV drug use (AIHA)
12. Fat cow with bloody liver spot: liver biopsy 24. Rain gutter damage from antibody-shaped
13. Harpoon pulling out white fur and yellow fat: antennas: Hepatitis C is associated with
steatosis and lymphoid aggregates seen on liver membranous glomerulonephropathy
biopsy 25. Splotchy red paint: Hepatitis C is associated with
14. “No cow tipping from 3-5” sign: lack of 3-5’ lichen planus
exonuclease activity, high antigenic variation, weak 26. Dyed beads hanging from wind chime: Hepatitis C is
immune response and no vaccine associated with diabetes
15. Cow carrying the alien away from the fire: 27. Boy with antibody-shaped antenna smashing
asymptomatic carriers are possible neck: Hepatitis C is associated with autoimmune
16. Dead cow with exposed ribs: treat with Ribavirin hemolytic anemia
17. Old sofa: treat with sofosbuvir
295

REVIEW QUESTIONS ?
1. A 35-year-old man presents to clinic due to
complaints of fever and yellowing of his skin.
He has a 16-year history of IV drug abuse and
regular unprotected sex with prostitutes. The
physician orders blood tests which suggest
an active viral infection. Additionally, AST and
ALT levels were slightly elevated. Physical
Photo Credit: BruceBlaus (Adert traduzione) [CC BY-SA 4.0 (https:// examination reveals slight yellowing of the
creativecommons.org/licenses/by-sa/4.0)], adjustments made
sclera in both eyes. To treat the infection, the
physician prescribes ribavirin and ledipasvir.
Based on the likely virus, which of the following
statements is true?
A. The virus was most likely obtained through
unprotected intercourse
B. This infection confers a higher likelihood of
developing lichen planus
C. There is no vaccine because the virus lacks
an envelope
D. The liver is likely unaffected at this stage in
the patient’s illness
Photo Credit: Nephron [CC BY-SA 3.0 (https://creativecommons.org/licenses/
by-sa/3.0)] E. The virus could only have been transmitted
Figure 5.4.83 - Fat and lymphoid aggregates seen from a symptomatic carrier
• Correct answer: B
on HCV liver biopsy
• The patient likely has HCV based on the
following:
• Fever, jaundice, and scleral icterus
• Elevated AST and ALT
• History of IVDU
• Treatment with ribavirin and
ledipasvir
• HCV is associated with lichen planus
• Choice A is incorrect because HCV is more
likely to be transmitted through IVDU
• Choice C is incorrect because HCV is
Photo Credit: (left) by emmanuelsegmen via Flickr (alterations made); (right)
Nephron [CC BY-SA 3.0 (https://creativecommons.org/licenses/by-sa/3.0)] enveloped and vaccines are unavailable
because of the high rate of antigenic
variation on the envelope
• Choice D is incorrect because elevated AST
and ALT indicate current liver inflammation
• Choice E is incorrect because asymptomatic
individuals can carry and transmit HCV

Photo Credit: (left) James Heilman, MD [CC BY-SA 3.0 (https://


creativecommons.org/licenses/by-sa/3.0)]; (right) Masryyy [CC BY-SA 4.0
(https://creativecommons.org/licenses/by-sa/4.0)]
Figure 5.4.84 - Lichen planus
296

Section 17 - Yellow fever

1. Rainbow: positive sense 9. “Arbol” sign on tree: arbovirus


2. Man wearing all yellow: yellow fever 10. Human running towards monkey being bitten by
3. Yellow clothes: jaundice mosquitoes: humans and monkeys are reservoirs
4. Sweat: sweating and malaise 11. Heat lamp: cause high fevers
5. Bars trapping chimp: linear virus 12. Cow with liver spot: causes hepatitis
6. Cane with icosahedral gem: icosahedral capsid 13. Cow running into person’s back: can cause back pain
7. Needle and “Live monkey” sign: prevent with live 14. Drinking water from lake with terrible flavor:
vaccine flavivirus
8. Mosquitos in lake: transmitted through Aedes 15. Vomiting: can cause black vomiting
mosquito 16. Red color scheme: RNA virus
297

REVIEW QUESTIONS ?
1. A 12-year-old girl is brought to the physician
due to “orange skin.” Her mother states that the
family recently returned from a trip to Brazil.
The patient denies receiving any vaccinations
prior to the trip. Her temperature is 39.4°C
(102.9°F). Physical examination confirms
jaundiced skin and tenderness of the right
upper quadrant of her abdomen. Blood results
indicate elevated AST and ALT. PCR is performed
and reveals the presence of an RNA arbovirus
with a linear genome. The physician reports
that the patient likely acquired a flavivirus
that is endemic to South America and Africa.
Which of the following is not true regarding the
presumed illness?
A. The Infection may have been prevented
with a live vaccine prior to travel
B. The virus does not carry its own RNA
polymerase
C. Mosquito repellant and long clothing may
have prevented infection
D. The virus has ribonucleotides in an
icosahedral configuration

• Correct answer: D
• The patient likely has yellow fever based
on the following:
• Jaundice and RUQ pain
• Elevated transaminases
• Viral PCR indicating an RNA
arbovirus with a linear genome
• Suspicion of a flavivirus endemic to
South America and Africa
• The capsid has an icosahedral
configuration, not the RNA genome
• Choices A, B and C are all true statements,
making them incorrect choices
298

Section 18 - Dengue Fever

1. Dinghy: Dengue fever 10. Water cooler: supportive care as treatment


2. Rainbow: positive sense 11. Scuba diver with back pain: severe back pain
3. Warm colors: RNA virus 12. Line of bubbles: linear virus
4. “Arbol” tree: arbovirus 13. Explosion above water: re-infection with different
5. Colony of mosquito larvae: obtained through serotypes is devastating
mosquito bites 14. Electric explosion: shock
6. Scuba diver with arm pain: myalgia 15. Blood in water: dengue hemorrhagic fever
7. Heat lamp: fever 16. Broken plates: low platelets
8. Rust on boat: rash
9. Icosahedral-shaped self propulsion device:
icosahedral capsid
299

REVIEW QUESTIONS ?
1. A 24-year-old man comes to his family physician
with severe back pain, a rash, and muscle
aches. Upon further history, the patient says
he recently returned from a humanitarian trip
to the Bahamas. He states that he has never
been there before and he received numerous
mosquito bites throughout the trip. His wife
said he likely has “breakbone fever”. If this is
correct, which of the following is true regarding
his condition?
A. He is infected with an arthropod-borne
parasite
B. If re-infected with a different serotype, his
presentation could be worse
C. The infectious organism is naked and
icosahedral
D. He may have diffuse hemorrhage prior to
illness resolution

• Correct answer: B
• The patient likely has Dengue fever
based on the following:
• Severe back pain, rash, and muscle
aches
• Mosquito bites
• If a patient acquires Dengue fever
a second time, and the serotype is
different, they can present with diffuse
hemorrhage
• Choice A is incorrect because Dengue fever
virus is a virus, not a parasite
• Choice C is incorrect because Dengue fever
virus is enveloped
• Choice D is incorrect because the first
Dengue fever infection is unlikely to cause
diffuse hemorrhage
300

Section 19 - West Nile Virus and St. Louis Encephalitis

1. “Breaking News: West Nile and St. Louis Encephalitis 7. Fainting news anchor with water on head: can cause
Ravage North America” sign: St. Louis encephalitis encephalitis
and West Nile Virus 8. Line of mosquitos on news anchor’s back: linear
2. Warm color scheme: RNA virus virus
3. St. Louis arch and west nile with swarms of 9. Worker spraying DEET mosquito repellent: prevent
mosquitoes: both viruses are very similar with DEET (diethyltoluamide)
4. Rainbow: positive sense viruses 10. Long sleeve shirts: prevent with long sleeved
5. Microphone with icosahedral shape: icosahedral clothing
capsid 11. Gagging on bad flavored chemicals: flavivirus
6. Mosquitos in news studio: transmitted via mosquito
bites (arbovirus)
301

REVIEW QUESTIONS ?
1. A 52-year-old man is brought to the emergency
department by his wife due to confusion,
headaches and worsening weakness for 2
days. The wife explains that they have spent a
lot of time outdoors this summer to get some
sunshine. When asked about the situation,
the patient fails to respond appropriately.
On physical exam his temperature is 38.8°C
(102°F) and he demonstrates 2/5 strength of
the left upper extremity and 5/5 strength in all
other extremities. There are various bumps on
both arms, consistent with recent mosquito
bites. The physician is concerned about a viral
CNS infection and orders a lumbar puncture.
Cerebrospinal fluid is collected and results
are pending. Which of the following is most
consistent with the patient’s presentation?
A. The infection may have been prevented
with insect repellent
B. The pathogen is a linear virus with a helical
capsid
C. The virus can be transmitted via saliva from
an infected human
D. The virus is a member of the Picornaviridae
family

• Correct answer: A
• The patient likely has viral encephalitis
based on the following:
• Confusion, headaches, weakness
• Asymmetric muscle weakness
• Evidence of mosquito bites
• Concern for a “viral CNS infection”
• West Nile and St. Louis Encephalitis
are transmitted by mosquitoes which
means insect repellent can prevent
infection
• Choice B is incorrect because West Nile and
St. Louis Encephalitis are icosahedral
• Choice C is incorrect because human saliva
does not transmit West Nile and St. Louis
Encephalitis
• Choice D is incorrect because West Nile and
St. Louis Encephalitis are in the Flavivirus
family
302

Section 20 - Zika Virus

1. Giant mosquitos: mosquito-borne (arbovirus) 9. Icosahedral-shaped lamps: icosahedral capsid


2. “Meet TanZIKA” sign: Zika virus 10. Little boy with rash: rash
3. Line of tourists: linear virus 11. Bloody bucket full of mosquitos and chains hanging
4. Warm colors: RNA virus from bucket: diagnose via serology or PCR
5. Rainbow: positive sense 12. Red eyes: viral conjunctivitis
6. Torches: transplacental infection (TORCHZ) 13. Water bottles: treat with supportive care
7. Penguin toy: congenital microcephaly 14. Gagging on water with bad flavor: Flavivirus
8. Heat lamp: fever
303

REVIEW QUESTIONS ?
1. A 23-year-old female at 9 weeks gestation
presents with a low-grade fever and red eyes
after returning from a vacation in Tahiti. She
learned of a recent outbreak of Zika virus at her
vacation destination and is now worried about
what a Zika infection could do to her baby. What
congenital malformation is associated with
transplacental Zika infection?

• Correct answer: Fetal microcephaly is a


known complication of Zika infections in
pregnant women
304

Section 21 - Rubella

1. Red bell: Rubella 11. Injured swordsman holding leg: causes arthralgias
2. German Shepherd dog: previously known as 12. Torch: TORCHES infection
“German Measles” 13. Mother wearing eye patch: congenital rubella may
3. Toga: a member of the togavirus family cause cataracts
4. Warm colors: RNA virus 14. Mother wearing ear muffs: congenital rubella may
5. Rainbow: positive sense virus cause deafness
6. Line of people waiting for fight: linear 15. Mother eating blueberry muffin: congenital rubella
7. Icosahedral shaped bell: icosahedral may cause a “blueberry muffin” rash
8. Mist: transmitted via aerosolized respiratory 16. Cart with “PDA” near couple showing PDA:
droplets congenital rubella may cause a patent ductus
9. Injured gladiator covered in blood: causes a arteriosus
maculopapular rash that starts on the face and 17. Yellow paint near mother: congenital rubella may
spreads to the trunk and extremities cause jaundice
10. Curly hair behind ears: causes posterior 18. “Live show” sign shaped like a syringe: live vaccine
lymphadenopathy available
305

REVIEW QUESTIONS ?
1. A newborn male is born at 38 weeks’ gestation
to a 27-year-old unvaccinated woman. He has
purple purpuric skin lesions and a continuous
murmur best heard over the left infraclavicular
area. During the first trimester the mother had
an illness caused by a single-stranded, positive-
sense, RNA virus with an envelope. Which of the
following clinical features did the mother most
likely experience during this time?
A. A mononucleosis-like illness
B. Vesicular genital lesions
C. A maculopapular rash
D. A painless genital ulcer

• Correct answer: C
• The infant is presenting with a “blueberry
muffin” rash caused by congenital rubella
• Purple purpuric skin lesions →
“blueberry muffin” rash
• Continuous murmur best heard over the
left infraclavicular area → patent ductus
arteriosus
• The mother developed an illness during
the first trimester which was caused by
a single-stranded, positive-sense, RNA
Photo Credit: CDC [Public domain] virus with an envelope
Figure 5.4.85 - Maculopapular rash of Rubella • Rubella causes a maculopapular rash
that starts on the face and spreads to
the trunk and extremities
• So the mother most likely developed a
rash during her first trimester but may
have also developed posterior auricular
lymphadenopathy or arthralgias
• A is a reference to CMV which can also
cause a “blueberry muffin” rash but is not
an RNA virus
• B is a reference to HSV which can cause
recurrent infections and chronic diarrhea in
a neonate - not a “blueberry muffin” rash
and a PDA
• D is a reference to syphilis but this is a
bacterium and is not associated with a PDA
or a “blueberry muffin” rash
306

Section 22 - Eastern, Western, and Venezuelan equine encephalitis viruses; Chikungunya


virus

1. Toga: Togaviridae 9. Red knee on cat in tree: Chikungunya virus can


2. East, West, and South sign: Eastern, Western, and cause joint pain
Venezuelan equine encephalitis 10. Black spots on fur of cat: Chikungunya virus can
3. Cat in tree eating chicken: Chikungunya virus cause a rash
4. Warm colors: RNA virus 11. Sweating cat: Chikungunya virus can cause flu-like
5. Rainbow: positive sense symptoms
6. Icosahedral-shaped souvenir: icosahedral 12. Characters throwing water on top of each other’s
7. Mosquito swamp: transmitted by mosquitoes heads: Eastern, Western, and Venezuelan equine
8. “Arbol” tree: arboviruses encephalitis viruses can cause encephalitis
307

Photo Credit: Nsaa [CC BY-SA (https://creativecommons.org/licenses/by-sa/3.0)]


Figure 5.4.86 - Chikungunya-related erythematous maculopapular rash

REVIEW QUESTIONS ?
1. A 25-year-old male presents to his doctor due • Correct answer: A
to a rash on his hands that he noticed yesterday. • This patient has an infection caused by
He states that 3 days ago he returned home Chikungunya virus
from a trip to West Africa and hasn’t been • A rash
feeling well since. His temperature is 39.9°C • Fever
(103.8°F). Physical examination reveals a • Due to a togavirus
maculopapular rash on the patient’s hands and • Chikungunya virus is transmitted by the
arms. Additional evaluation reveals that the Aedes mosquito
causal organism is a togavirus. This patient’s • B is a reference to several organisms such
condition is most likely transmitted by which of as Borrelia burgdorferi, Babesia, and
the following? Anplasma but none of these are togaviruses
• C and D are true of many pathogens but not
A. The Aedes mosquito
of togaviruses (togaviruses are transmitted
B. The Ixodes deer tick
C. Sexual contact by the Aedes mosquito)
D. Aerosolized water droplets
308

Section 23 - California Encephalitis

1. “Welcome to California” sign on Golden Gate 8. “Arbol” tree: arbovirus


Bridge: California encephalitis 9. Mosquito in car: transmitted through mosquitoes
2. Red colors: RNA virus 10. Map of the United States with central and east coast
3. Hot dog buns: a member of the Bunyavirus family highlighted: common in the central and eastern part
4. Rain clouds: negative sense of the U.S.
5. Helical shaped wires on bridge: helical capsid 11. Boy sticking head out of window into rain: causes
6. Little girl drawing circles: circular encephalitis
7. Little boy with chocolate bar: 3 segments
309

REVIEW QUESTIONS ?
1. A 9-year-old girl is brought to the emergency • Correct answer: B
department due to altered mental status. Her • This girl has encephalitis caused by
father states that yesterday she complained of California encephalitis
a headache and then vomited several times. • Altered mental status
However, this morning she was disoriented to • Headache
time and place. During the interview the patient • Vomiting
begins to seize. After appropriate management, • A virus that was found in the CSF
analysis of the CSF reveals an RNA virus with • The pathogen is an RNA virus with a
a single stranded circular genome that has 3 single stranded circular genome that has
segments. What is the most likely diagnosis? 3 segments → Bunyavirus
• A, C, and D can all cause neurological
A. Lymphocytic choriomeningitis virus
deficits but only California encephalitis has
B. California encephalitis
C. St. Louis encephalitis a circular genome with 3 segments
D. West Nile virus
310

Section 24 - Rift Valley Fever and Crimean-Congo Hemorrhagic Fever Virus

1. “WELCOME TO THE CONGO” sign: Crimean-Congo 11. Man in boat eating tic tacs: Crimean-Congo
hemorrhagic fever hemorrhagic fever is transmitted by ticks
2. “WELCOME TO THE VALLEY” sign: Rift Valley fever 12. Red river: Crimean-Congo hemorrhagic fever causes
virus hemorrhagic fever
3. Hotdog bun: Bunyavirus 13. Lamp: Rift Valley fever virus and Crimean-Congo
4. Warm colors: RNA virus hemorrhagic fever can cause fever
5. Dark rain cloud: negative sense virus 14. Headbands: Rift Valley Fever virus and Crimean-
6. Circular-shaped greenery: circular Congo hemorrhagic fever can cause neurological
7. 3 segmented chocolate bar: 3 segments disturbances
8. Helical shaped chain: helical capsid 15. Map of Africa: prevalent in Africa
9. “Arbol” tree: arbovirus
10. Traveler in the valley getting attacked by mosquitos:
Rift Valley fever virus is transmitted by mosquitoes
311

REVIEW QUESTIONS ?
1. A 24-year-old male presents to the emergency • Correct answer: C
department due to vision changes and a rash • This patient has Crimean-Congo
that he noticed yesterday. He recently returned hemorrhagic fever
home from a humanitarian trip to Nigeria. • Recent travel history to Nigeria, fever,
His temperature is 39.7°C (103.4°F). Physical petechial rash, and retinal hemorrhages
examination reveals a petechial rash on his • Single-stranded, negative-sense, 3
arms and legs. Fundoscopy reveals retinal segments → Bunyavirus
hemorrhage. Further evaluation reveals that the • Crimean-Congo hemorrhagic fever and
pathogen is a single-stranded, negative-sense Hantavirus are bunyaviruses and can
virus that is comprised of 3 RNA segments. This cause the patient’s symptoms but the
patient’s condition was most likely transmitted travel history makes Crimean-Congo
through which of the following? more likely
• Crimean-Congo hemorrhagic fever →
A. Rodents
Africa
B. Mosquitoes
C. Ticks • Hantavirus → China
D. Aerosolized respiratory droplets • Crimean-Congo hemorrhagic fever is
E. Dogs   transmitted by ticks
• A is incorrect because it refers to the route
of transmission of Hantavirus
• B is incorrect because while it is true that
many viruses (including some Bunyaviruses)
are transmitted by mosquitoes, they do not
present with hemorrhagic fever
• D is incorrect because Bunyaviruses are not
transmitted through aerosolized respiratory
droplets
• E is incorrect because it is a reference to
bacteria (Campylobacter jejuni and Yersinia
enterocolitica) but these do not cause
hemorrhagic fever
312

Section 25 - Hantavirus

1. Haunted house: hantavirus 8. Shirt with China graphic: prevalent in China


2. Red colors: RNA virus 9. Man in rodent costume: transmitted through
3. Rain clouds: negative sense rodents
4. Spiral staircase: helical capsid 10. Bloody teeth, holding lamp: causes hemorrhagic
5. Circular shaped entrance: circular fever
6. Girl eating chocolate bar: 3 segments 11. Axe shaped like lungs: causes pulmonary disease
7. Girl with hot dog bun: Bunyavirus
313

REVIEW QUESTIONS ?
1. A 36-year-old male presents to the physician
due to shortness of breath. He states that his
shortness of breath began earlier today, but
approximately one week ago he developed a
fever, chills, and myalgias. He also mentions
that he recently returned home from a trip to
China where he lived in a house infested with
rodents. This patient’s condition is most likely
due to a virus that has which of the following
characteristics?
A. RNA, 3 segments, circular
B. RNA, 2 segments, circular
C. RNA, 10-12 segments, linear
D. RNA, 8 segments, linear

• Correct answer: A
• This patient has a Hantavirus infection
• Shortness of breath
• Recent trip to China
• Exposure to rodent-infested house
• Hantavirus is a circular RNA virus with 3
segments
• B is false because it is referring to Lassa
fever encephalitis which is also spread by
rodents but presents with encephalitis and
not pulmonary symptoms
• C is false because it is referring to
Reoviruses which cause GI symptoms and
are not associated with rodent exposure
• D is false because it is referring to the
Influenza virus which also has pulmonary
symptoms but the travel history and rodent
exposure make Hantavirus more likely
314

Section 26 - Influenza virus

1. Flute: influenza 14. Gear shift handle, assorted chocolate box, and pet
2. Orthodontist sign: orthomyxovirus animal: exhibits genetic shift (two distinct strains
3. Warm colors: RNA virus undergo genetic reassortment resulting in a novel
4. Rain cloud: negative sense strain)
5. Line of people at concert: linear virus 15. Skull shirt: genetic shift is more deadly than genetic
6. Car with v8 engine: 8 segments drift
7. Spiral staircase: helical capsid 16. Heat lamps: may cause fever
8. Fish tank with nuclear sign: replication in the 17. Fan holding arm in pain: may cause myalgias
nucleus of the host cell 18. Fan coughing up soda: may cause a cough
9. “He needs in” near entrance: hemagglutinin is a 19. Smoke from broken down car: may cause primary
surface glycoprotein that facilitates viral entry into pneumonia
the host cell 20. “The office” bumper sticker, Merlin tattoo, and
10. Antibody-shaped door knocker: antibodies against lawn mower tattoo: May cause secondary bacterial
hemagglutinin provide future immunity to similar pneumonia (most often due to Staphylococcus
strains of influenza aureus, Streptococcus pneumoniae, and
11. Pack of new erasers near exit: neuraminidase is an Haemophilus influenzae)
enzyme associated with the release of viral progeny 21. Car: may cause cardiac complications
from the host cell 22. Hats: may cause neurological complications (aseptic
12. Hot tamales: zanamivir and oseltamivir (Tamiflu) meningitis)
can be used to treat influenza by disrupting 23. Ray of light: associated with Reye syndrome
neuraminidase 24. Orthodontist with syringe: killed vaccine
13. Letters drifting in the air: exhibits genetic drift 25. “Live show” syringe with guy smelling scented
(random mutations that occur in the hemagglutinin flowers: live vaccine (replicates in nose but not
and neuraminidase gene) lungs)
315

Figure 5.4.87 - Genetic shift and genetic drift

REVIEW QUESTIONS ?
1. A 64-year-old male presents to the physician • Correct answer: D
due to a fever, cough, and shortness of breath • This patient has secondary bacterial
that began yesterday. He states that he also had pneumonia following an infection by
similar symptoms a week ago. His symptoms Influenza virus
began to improve up until yesterday when • Fever, cough, shortness of breath,
they suddenly became worse. His temperature crackles in lower lobes of the lungs →
is 38.9°C (102.0°F). Pulmonary auscultation pneumonia
reveals crackles in the lower lobes bilaterally. • Secondary bacterial pneumonia is
A sputum culture will most likely grow an commonly caused by Staphylococcus
organism with which of the following features? aureus
• Staphylococcus aureus colonizes the
A. Pyocyanin production
nares
B. Growth on charcoal yeast extract
C. Decreased muramic acid in the cell wall • A-C are references to bacterial species that
D. Colonization of the nares cause pneumonia but are not as commonly
associated with secondary bacterial
pneumonia
• A is a reference to Pseudomonas aeruginosa
• B is a reference to Legionella pneumophila
• C is a reference to Chlamydia trachomatis
316

Section 27 - Lassa fever encephalitis and Lymphocytic choriomeningitis virus

1. Baseball arena: Arenavirus 7. Water on top of umpire helmet: causes


2. Warm colors: RNA virus meningoencephalitis
3. Two circular mounds: two segments and a circular 8. Lassie dog with rain over head: Lassa fever
structure encephalitis
4. Rainbow and clouds: positive sense and negative 9. Rodents: spreads by rodents
sense 10. Ghanaian person: prevalent in West Africa
5. Spiral staircase: helical capsid
6. Limping umpire with unique helmet: lymphocytic
choriomeningitis virus

REVIEW QUESTIONS ?
1. A 24-year-old female is brought to the • Correct answer: C
emergency department due to altered mental • This patient has encephalitis caused by HSV-
status over the past 24 hours. Her boyfriend 1
accompanies her and states that many of their • Altered mental status and fever →
recent conversations have been nonsensical. encephalitis
Her temperature is 38.6°C. PCR analysis of the • PCR reveals a linear double stranded
CSF reveals a virus with a linear double stranded DNA virus → HSV-1
DNA structure. Based on these findings, what is • A and B are incorrect because they are
the most likely diagnosis? arenaviruses which are RNA viruses
• D is incorrect because JC virus has a circular,
A. Lassa fever encephalitis
double-stranded DNA structure
B. Lymphocytic choriomeningitis virus
C. Herpes simplex virus-1
D. JC virus
317

Section 28 - Hepatitis D Virus

1. Dogs herding cows with liver spots: hepatitis D virus 7. Beehive: HDV is dependent on HBV to cause
2. Delta airplane: belongs to deltavirus family infection
3. Warm colors: RNA virus 8. People running away from bees and taking off their
4. Dark clouds: negative sense coats near dogs: must be coated with the surface
5. Circular lasso: circular antigen of HBV to cause infection
6. Pregnant woman with “S” on shirt: sexually and 9. Cow stuck in dead weeds: increases risk of cirrhosis
perinatally transmitted 10. Shattered glass on ground: a liver biopsy will show
“ground glass” hepatocytes
318

 
Figure 5.4.88 - Acute HDV-HBV coinfection
 

Figure 5.4.89 - HDV superinfection


319

REVIEW QUESTIONS ?
1. A 33-year-old male presents to the physician • Correct answer: B
due to 3 days of right upper quadrant • This patient has an HDV superinfection
abdominal pain, a subjective fever, and • 3 days of right upper quadrant
jaundice. He has been with multiple male abdominal pain, subjective fever,
partners over the past several years and is jaundice → acute hepatitis
worried he has HIV. Serological markers are • The patient’s sexual history makes HBV,
drawn and reveal the following: HDV, and HCV possible  
• IgG hepatitis B core antibody test →
Hepatitis B surface antigen positive chronic HBV infection present
IgG hepatitis B core antibody positive • Hepatitis D RNA →  acute HDV
Hepatitis D RNA positive superinfection
HIV-1 antibody negative • HDV is a single-stranded, negative-
 
sense, circular RNA virus
This patient’s acute condition is most likely caused
• A is true of HBV but the patient’s acute
by a virus with which of the following features?
condition is due to an HDV superinfection
A. A partially double-stranded, circular DNA • C is true of HAV which is not sexually
virus transmitted and would not include the lab
B. A single-stranded, negative-sense, circular findings present in this patient
RNA virus • D is true of HIV but the HIV-1 antibody test
C. A single-stranded, positive-sense, linear was negative
RNA virus
D. 2 copies of a single-stranded, positive-
sense, linear RNA virus
320

Section 29 - Parainfluenza Virus

1. Warm colors: RNA virus 10. Crayon drawing of church: steeple sign
2. Dark clouds on newspaper: negative sense 11. Stripes on church: stridor
3. Pair of crayons mixing and melting: paramyxovirus 12. Crayon poop: croup
4. Pair of flutes: parainfluenza virus 13. Dog sneezing: upper respiratory symptoms
5. Helical-shaped slinky: helical capsid 14. “He needs in”: hemagglutinin permits viral entry by
6. Dog sniffing line of footprints: linear virus binding to sialic acid
7. Crayon trying to catch his breath: respiratory 15. Crayons with new erasers: neuraminidase
distress 16. Dog pulling pair of boxes: can cause pulsus
8. Barking dog: barking cough paradoxus
9. Collar choking trachea of dog: tracheal narrowing
321

REVIEW QUESTIONS ?
1. A 2-year-old boy is brought to the pediatrician
due to strange noises each time he coughs.
The boy’s mother says that he sounds like
a coughing seal. On physical examination
he appears mildly uncomfortable and
demonstrates rib retractions. The physician
informs the mother that her son has croup.
Which of the following is true regarding this
patient’s condition?
A. He has been infected with an influenza virus
B. A radiograph may reveal subglottic
narrowing
C. The virus is ready to be translated upon
entry to the host cell
D. The virus has double stranded genetic
material
Photo Credit: Frank Gaillard [CC BY-SA 3.0 (https://creativecommons.
org/licenses/by-sa/3.0)]
• Correct answer: B
Figure 5.4.90 - Steeple sign on radiograph in croup • The patient likely has croup based on
the following:
• Seal-like cough
• Rib retractions
• Croup can cause subglottic narrowing
when radiographed
• Choice A is incorrect because parainfluenza
virus is a type of paramyxovirus
• Choice C is incorrect because parainfluenza
virus is a negative-sense RNA virus and must
first be converted to a positive sense strand
using its RNA-dependent RNA polymerase
• Choice D is incorrect because parainfluenza
virus is single-stranded
322

Section 30 - RSV and hMPV

1. Red Searing Vent: RSV 10. Dark clouds carry moisture: negative sense RNA
2. Pair of melted crayons mixing: paramyxovirus viruses carry their own RNA polymerase
3. Human metal toy: human metapneumovirus 11. Boy wiping mucus on seat: mucus, drainage, and
(HMPV) congestion
4. Line of cars: linear virus 12. Balloon letting out air: wheezing
5. Warm colors: RNA virus 13. Cracks in sidewalk shaped like lungs: bronchiolitis
6. Helical slinky toy: helical capsid 14. Pallet of visine: palivizumab
7. Retracting ribs on baby: respiratory distress in 15. Water jug: supportive care
infants 16. Oxygen mask: treat with respiratory support and
8. Sweating mom: fever oxygen supplementation
9. Dark clouds: negative sense
323

REVIEW QUESTIONS ?
1. A 2-week-old female is brought to the
pediatrician by her mother due to persistent
nasal drainage and a worsening cough. The
mother states the infant has “felt warm” but has
not taken the patient’s temperature. On physical
exam, the physician notices nasal flaring,
subcostal retractions, and respiratory crackles
on chest auscultation. The physician suspects
bronchial inflammation due to a negative sense
RNA virus. Assuming the physician is correct,
what should be the recommended treatment?

• Correct answer: provide supportive care,


including fluid and respiratory support
• The patient likely has an RSV or hMPV
Photo Credit: Matteo Di Nardo, Daniela Perrotta, Francesca Stoppa, infection based on the following:
Corrado Cecchetti, Marco Marano and Nicola Pirozzi [CC BY 2.0
(https://creativecommons.org/licenses/by/2.0)], via Wikimedia • Negative sense RNA virus
Commons • Bronchial inflammation
Figure 5.4.91 - Bronchiolitis on chest radiograph (bronchiolitis)
• Subcostal retractions and nasal
flaring (respiratory distress)
324

Section 31 - Measles

1. Weasels: measles 11. Weasel licking gum spot: can cause Koplik spots
2. Warm colors: RNA virus 12. Pair of melting and mixing crayons: paramyxovirus
3. Dark clouds: negative sense 13. Red skull: subacute sclerosing panencephalitis
4. Retail store: retinol 14. Red-eyed weasel: coryza and conjunctivitis
5. “Live weasels inside” sign shaped like a syringe: live 15. Helical-shaped slinky: helical capsid
vaccine prevents measles 16. Netted cage with white toy balls melting together:
6. Line of fire: linear virus fused lymphocytes (Warthin-Finkeldey) and
7. Coughing weasel: causes cough hyperplasia in lymph nodes
8. Oil lamp: fever 17. Giant explosion near man on stretcher: giant cell
9. Red ash landing on face and traveling down: pneumonia in immunocompromised patients
maculopapular rash spreads downwards 18. Weasels with cough, conjunctivitis, coryza, koplik
10. Water pouring over weasel’s head: can cause spots, and rash: uncomplicated measles
encephalitis
325

REVIEW QUESTIONS ?
1. An unvaccinated 2-year-old girl presents with
a cough and watery eyes. A serum lab test
confirms high levels of measles IgM. After
further discussion with the girl’s mother, the
clinician suspects the child is malnourished and
likely has a deficiency of retinol. Which of the
following symptoms is most likely to present
next?
A. Encephalitis
B. Giant cell pneumonia
C. Maculopapular rash
D. Watery diarrhea
Photo Credit: Wikimedia Commons [Public domain]
Figure 5.4.92 - Maculopapular rash in measles • Correct answer: C
• The patient likely has uncomplicated
measles based on the following:
• Cough
• Watery eyes (coryza)
• Measles IgM
• Choice A is incorrect because complicated
pneumonia with encephalitis is less likely
than a maculopapular rash, even though the
patient is deficient in vitamin A (retinol)
• Choice B is incorrect because the patient is
not immunocompromised
• Choice D is incorrect because diarrhea is not
associated with measles
Photo Credit: CDC [Public domain], via Wikimedia Commons
Figure 5.4.93 - Koplik’s spots

Photo Credit: Content Provider(s): CDC/Dr. Edwin P. Ewing, Jr. [Public


domain], via Wikimedia Commons
Figure 5.4.94 - Giant cells
326

Section 32 - Mumps

1. Warm color scheme: RNA virus 8. Pot dropping on head of kid with hat: meningitis
2. Bumps of crayons: mumps 9. Ball of crayons: Orchitis
3. Dark and negative rain clouds: negative sense 10. Stepping on testicle-shaped crayon mounds: sterility
4. Flaming pan: pancreatitis 11. “Vaccine free!!” hoodie: unvaccinated children at
5. Pair of mixing crayons: paramyxovirus risk
6. Crayon line: linear virus 12. “Live: Mumps outbreak”: live vaccine (MMR)
7. Big cat cheeks: parotitis
327

REVIEW QUESTIONS ?
1. A 35-year-old male presents to his family
physician complaining that he and his wife have
been unable to conceive despite trying for 5
years. He believes he may be infertile. Upon
further questioning, he describes having swollen
testicles when he was a kid during a “weird”
illness. The physician suggests the patient’s
described illness is likely the source of his
infertility. Which of the following is most likely
true regarding this patient’s childhood illness?
A. It was caused by an orthomyxovirus
B. He was fully immunized at the time
C. He may have also had parotid gland
inflammation
Afrodriguezg [CC BY-SA 3.0 (https://creativecommons.org/ D. The infectious agent did not carry its own
licenses/by-sa/3.0)], from Wikimedia Commons RNA polymerase
Figure 5.4.95 - Mumps-related parotitis
• Correct answer: C
• The patient likely had mumps as a child
based on the following:
• Swollen testicles
• Fertility issues
• Mumps is known to cause parotid gland
inflammation (parotitis)
• Choice A is incorrect because Mumps is a
paramyxovirus, not an orthomyxovirus
• Choice B is incorrect because Mumps is
prevented through vaccination
• Choice D is incorrect because Mumps is a
negative sense virus and must carry its own
RNA-dependent RNA polymerase into host
cells
328

Section 33 - Rabies Virus

1. Infected beast: Rabies 16. Water splashing on hat: causes encephalitis


2. Warm color: RNA virus 17. The bell and game of memory: Rabies virus can be
3. Negative rain clouds: negative sense found in the cerebellum and hippocampus
4. Foaming at the mouth: patients experience 18. Beast devouring seagull: Rabies virus attacks
hypersalivation acetylcholine receptors
5. Holding head: can cause headaches 19. Dynamite: dynein motors carry Rabies virus up the
6. Were-raccoon: raccoons are reservoirs axon in a retrograde fashion
7. Bats: bats are reservoirs 20. Lowest light burning out: Rabies travels up axon in a
8. Skunks: skunks are reservoirs retrograde fashion
9. Game of memory: rabies infects the hippocampus 21. Paralyzed leg: Rabies can cause sensation loss and
10. Dead beast: rabies ultimately causes death paralysis
11. Bullet shells: bullet-shaped envelope 22. Line the ground: Rabies is a linear virus
12. Wounds: Negri bodies are cytoplasmic inclusions 23. Curly tails: Rabies has a helical capsid
found in neurons 24. Sick-looking human: flu-like symptoms early in the
13. Man losing balance on bell tower: Rabies infects the disease
cerebellum 25. Broken vaccine needles: treat patients with killed
14. Bungee cord: rabies virus infects the Purkinje vaccine
neurons within the cerebellum 26. Clothes pins: treat acute infections with rabies
15. Rabid beast recoiling at water: rabies causes immunoglobulin
hydrophobia
329

Photo Credit: [Public domain]


Figure 5.4.96 - Rabies virus structure

Photo Credit: CDC, [Public domain] via Wikimedia Commons


Figure 5.4.97 - Rabies virus bullet-shaped envelopes
on electron microscopy

Photo Credit: CDC/Dr. Daniel P. Perl [Public domain]


Figure 5.4.98 - Rabies virus Negri bodies
330

Figure 5.4.99 - Retrograde movement of Rabies virus


331

REVIEW QUESTIONS ?
1. A 43-year-old male was bitten on the right • Correct answer: B
hand by a bat while cleaning out his cabin in • The patient likely has rabies based on
a forested area. Several months later he died the following:
and an autopsy was performed. An electron • Electron microscopy revealing
micrograph of the cerebellum was taken bullet-shaped envelopes
following dissection of the patient’s brain which • Bat bite
is shown below. Which of the following is most • Biopsy taken from the cerebellum
likely true regarding the deceased patient’s • Rabies causes loss of sensation or motor
condition? function in a retrograde fashion
• Choice A is incorrect because rabies causes
fever and malaise early in the infection
• Choice C is incorrect because infected
patients should receive a killed vaccine and
immunoglobulins
• Choice D is incorrect because painful
pharyngeal spasms cause hydrophobia

A. The patient did not experience fever and


malaise early in the disease
B. He experienced numbness in his right hand
during his illness
C. Rabies immunoglobulin and live rabies
vaccine should have been administered
soon after the bite
D. The patient avoided water due to perceiving
a bitter taste
332

Section 34 - Ebola Virus and Marburg Virus

1. Marburg’s bowling alley: Marburg virus and Ebola 11. Sick-looking man: flu-like symptoms
virus 12. Apes eating from dead body: spread via dead
2. Zombie files: filoviridae bodies, fomites, and apes
3. Negative rain clouds: negative sense 13. Walking through bodily fluids: spreads via direct
4. Warm colors: RNA virus contact with bodily fluids
5. Bleeding zombie: diffuse hemorrhage 14. Diarrhea and vomiting: diarrhea and vomiting
6. Smeared line of blood: linear virus 15. Healthcare worker: wear personal protective
7. Packs of coagulated Jello: diffuse intravascular equipment (PPE)
coagulation 16. Water bottle: treat with supportive care
8. Pulled muscle: myalgias 17. 3 chains: diagnose with polymerase chain reaction
9. Shocked to death: shock (PCR)
10. Heat lamps: fever
333

REVIEW QUESTIONS ?
1. A 35-year-old African-American female presents
to the emergency department due to concerns
that she is infected with the Ebola virus.
She recently traveled to Sierra Leone to visit
extended family and returned 1 week ago. Since
returning to the United States, she learned that
there was an outbreak of Ebola in the town she
visited. She now reports a fever, muscle aches
and several episodes of emesis over the past
24 hours. If she is infected with the Ebola virus,
which of the following is most likely true?
A. Antiviral medication may reduce the
duration of her illness
B. PCR will confirm the diagnosis
C. She had sexual contact with an infected
person
D. She should go home immediately and
minimize her contact with other people

• Correct answer: B
• Choice A is incorrect because antivirals are
unavailable to treat Ebola virus
• Choice C is incorrect because Ebola is not
sexually transmitted
• Choice D is incorrect because the patient
should be quarantined at the hospital, not
sent home
334

Section 35 - Norovirus

1. Chalice: Calicivirus 7. Poop on shoe: fecal-oral transmission


2. Nordic viking: Norovirus 8. Icosahedral water jug: icosahedral
3. Warm colors: RNA virus 9. Seasick Vikings: vomiting and watery diarrhea
4. Naked viking: naked 10. United States sail: common in developed countries
5. Rainbow: positive sense 11. Water: treat with supportive care
6. Linear formation: linear
335

REVIEW QUESTIONS ?
1. A resident physician is in Nepal on a
humanitarian trip when a diarrheal outbreak is
reported at a nearby elementary school. Nine
children are brought to the clinic after several
days of intense vomiting and watery diarrhea.
No blood is detected in the stool. None of the
children are immunized. Which of the following
pathogens is most likely responsible for the
outbreak?
A. Norovirus
B. Adenovirus
C. Rotavirus
D. Shigella
 
• Correct answer: C
• This patient has a viral gastroenteritis
caused by Rotavirus
• Unvaccinated children
• Developing country
• Absence of blood in stool
• A is incorrect because Norovirus is more
common in developing countries among
unimmunized individuals
• B is incorrect because Adenovirus can cause
gastroenteritis but is a much less common
cause of diarrheal outbreaks (more
commonly associated with pharyngitis,
conjunctivitis, and coryza)
• D is incorrect because Shigella is a bacterial
cause of gastroenteritis and is much more
likely to result in a bloody diarrhea
336

Section 36 - Hepatitis E virus

1. Cow being eaten: Hepatitis E virus 6. First egg to hatch: short incubation period
2. Warm colors: RNA virus 7. Man stepped in poo: fecal-oral transmission
3. Rainbow: positive sense 8. Pregnant woman next to skull: fulminant hepatitis in
4. Naked man: naked virus pregnant women (high mortality rate)
5. Heap of eggs: Hepevirus 9. Black spots: liver biopsy shows patchy necrosis
337

REVIEW QUESTIONS ?
1. A 29-year-old pregnant female presents to
the emergency department due to malaise,
nausea, vomiting, and abdominal pain. Physical
examination reveals diffuse jaundice and
hepatomegaly. Despite aggressive intervention,
she passes away several hours later. During an
autopsy, a pathological specimen of her liver
reveals ballooned hepatocytes and focal areas
of necrosis. The organism responsible for this
patient’s condition:
A. Is a DNA virus
B. Is transmitted parenterally
C. Is a positive sense virus
D. Is an enveloped virus
E. Causes hepatocellular carcinoma
 
• Correct answer: C
• The patient had a hepatitis E infection
resulting in death
• Pregnant female, jaundice, and
hepatomegaly
• Hepatitis E is a positive sense virus
• Causes fulminant hepatitis in pregnant
women
• A is incorrect because hepatitis E is an RNA
virus  
• B is incorrect because hepatitis E is
associated with fecal-oral transmission
• C is incorrect because parenteral
transmission refers to IV drug users and this
is more commonly associated with hepatitis
B, C, and D
• D is incorrect because hepatitis E is a naked
virus
• E is incorrect because hepatitis B and C are
associated with hepatocellular carcinoma
but hepatitis E is not
338

Section 37 - Rotavirus and Colorado Tick Fever

1. Rio Grande river: reoviruses 13. Wheel: wheel-like shape seen on electron
2. Colt gun: Coltivirus microscopy
3. “Welcome to Colorado” and heat lamp: Colorado 14. Vomiting child: Rotavirus causes watery diarrhea
tick fever virus is a Coltivirus and vomiting (can be fatal in children)
4. Arbol: Colorado tick fever virus is an arbovirus 15. Shaved ice and snow: Rotavirus is more common
5. Tic tacs: transmitted through ticks during the winter time in the U.S.
6. Rotary motion: Rotavirus 16. Cacti: Rotavirus causes intestinal villi destruction
7. Warm colors: RNA virus 17. “Live animals” sign: live vaccine for Rotavirus
8. Two waterfalls: double stranded 18. Skid marks and syringe: do not give vaccine to
9. Line of people: linear patients with SCID
10. “10-12 ways to Tube”: approximately 11 segments 19. Telescopic lens: live vaccine increases the risk of
11. Naked guy: naked intussusception
12. Icosahedral-shaped ice cream cone: Icosahedral
339

Figure 5.4.100 - Positive-sense and negative-sense viruses

Photo Credit: Dr. Graham Deards [CC BY 3.0 (https://


creativecommons.org/licenses/by 3.0)]
Figure 5.4.101 - Transmission electron micrograph
of Rotavirus

REVIEW QUESTIONS ?
1. A 3-year-old unvaccinated male is brought • Correct answer: C
to the physician due to a temperature of • This patient has viral gastroenteritis caused
38.6°C (101.5°F), vomiting, and diarrhea. After by Rotavirus
thorough evaluation, the patient’s condition • Unvaccinated child with vomiting and
is thought to be due to a viral infection. diarrhea
Laboratory analysis reveals that the virus is • The virus is double-stranded, RNA, and
double-stranded, RNA, and linear. Which of the linear
following is true regarding the most likely causal • Electron microscopy of a stool sample
organism? of individuals with a rotavirus infection
will reveal viral particles with a wheel-
A. It is enveloped
like shape
B. The capsid symmetry is helical
C. Electron microscopy of a stool sample will • A is false because reovirus is a naked virus
reveal viral particles with a wheel-like shape • B is false because the capsid symmetry is
D. The nucleic acid is non-segmented icosahedral
• D is false because reoviruses have 10-12
segments
340

Section 38 - Poliovirus

1. Warm color: RNA virus 12. Hat knocked off henchman: causes meningitis
2. Rainbow: positive sense 13. Grabbing ox horns: infects anterior horns of spinal
3. Pole-vaulting guy: poliovirus cord
4. Asteroid breaking into pieces: one big protein is 14. Shaking ox: causes fasciculations
cleaved into several smaller, functional proteins 15. Limp ox: flaccid paralysis
5. Panda eating socks with needle in head: Salk vaccine 16. Peacock emperor: picornavirus
is killed 17. Icosahedral gems on staff: icosahedral capsid
6. “Live saber tooth tiger” poker: Sabin vaccine is live 18. Wimpy and defeathered wings: causes atrophy
7. IRS henchman: contains internal ribosomal entry 19. Panda breaking lung-shaped axe: can cause
site (IRES) respiratory failure
8. Naked henchman: naked virus 20. Pandas roasting intestines: enterovirus
9. Naked henchman in mud river: naked viruses enter 21. Sweating pandas: causes signs of infection such as
host ell via endocytosis sweating, malaise, fever and vomiting
10. Henchman at junction of brown and red river: 22. Line of henchman: linear virus
poliovirus enters the intestines and then the 23. Ox that stepped in poo: fecal-oral transmission
bloodstream
11. Poles blocking entrance of central facility: poliovirus
enters the CNS
341

REVIEW QUESTIONS ?
1. A 14-year-old previously healthy girl presents
to the emergency department due to vomiting,
sweating, and muscle weakness. She recently
emigrated from Southeast Asia and has not
received any vaccinations in her life. Her
temperature is 38.6°C (101.5°F). On physical
exam she appears diaphoretic, fatigued, and
drags her left leg as she walks. Her strength
is 1/5 in her left lower extremity, 3/5 in her
right lower extremity, 4/5 in her left upper
extremity, and 4/5 in her right upper extremity.
The physician suspects a Picornavirus infection.
Which of the following is inconsistent with her
condition?
A. Gray matter of the anterior spinal cord has
been damaged
B. Upper motor neuron signs should be
expected
C. The illness may have been prevented with a
live Sabin vaccine
D. Her condition may cause low inspiratory
capacity
E. Sensation to all extremities is likely to
remain intact

• Correct answer: B
• The patient likely has poliovirus based
on the following:
• Vomiting and diaphoresis
• Asymmetric weakness
• Upper motor neuron signs should not
be expected
• Choices A, C, and D are consistent with
poliovirus infection, making them wrong
answer choices
342

Section 39 - Echovirus

1. Echo: Echovirus 9. Henchman stepping on poop: fecal-oral


2. Rainbow: positive sense transmission
3. Warm colors: RNA virus 10. Henchman foaming at the mouth: salivary
4. Icosahedral gems on cane: icosahedral capsid transmission
5. Line of henchman: linear virus 11. Hat knocked off: causes meningitis
6. IRS henchman: contains internal ribosomal entry 12. Asteroid breaking into pieces: one big protein is
site (IRES) cleaved into several smaller functional proteins
7. Naked pigs: naked virus 13. Naked pigs in mud pit: naked viruses enter host cell
8. Pig choking henchman with intestines: considered via endocytosis
an enterovirus
343

REVIEW QUESTIONS ?
1. A laboratory researcher is studying traits
associated with picornaviruses. She identifies
a member of the picornavirus family that
replicates within Peyer’s patches and can cause
meningitis. Which of the following statements is
true regarding this virus?
A. Proteins surrounding the genome curve
continuously around a center point
B. The virus is considered one of the
enteroviruses
C. There is an absence of uracil in the viral
genome
D. The virion contains an enzyme for
synthesizing nucleic acids

• Correct answer: B
• Enteroviruses replicate within the small
intestine and cause meningitis
• Choice A is incorrect because this describes
a helical capsid
• Choice C is incorrect because this suggests
picornaviruses lack RNA
• Choice D is incorrect because this describes
RNA-dependent RNA polymerase,
something carried only by negative sense
RNA viruses
344

Section 40 - Rhinovirus

1. Warm colors: RNA virus 10. Henchman in mud pit: naked viruses enter host cell
2. Rainbow: positive sense via endocytosis
3. Rhinos: rhinovirus 11. Asteroid breaking into pieces: one big piece is
4. Peacock emperor: picornavirus cleaved into several smaller functional proteins
5. Icosahedral gem on cane: Icosahedral capsid 12. Coughing IRS henchman: causes cough
6. Line of rhinos: linear virus 13. Congested rhino: causes mucus drainage
7. Hundreds of rhinos: over 100 serotypes 14. IRS henchman: contains internal ribosomal entry
8. Rhinos burning in acid pit: acid labile site (IRES)
9. Naked henchman: naked virus 15. Henchman pulling collar: can cause pharyngitis
345

REVIEW QUESTIONS ?
1. An 8-year-old boy presents with his father
due to a sore throat and a mild cough. The
boy states it has been difficult to swallow due
to pain. The boy’s father complains that the
boy has constantly been wiping his snot on
his clothes for the past week. A rapid antigen
detection test and a culture for group A
Streptococcus (GAS) are both negative. Which
of the following statements is true regarding the
most likely pathogen?
A. Functional virions can be excreted in the
feces
B. Ribosomes translate viral nucleotides
without prior viral transcription
C. Functional virions replicate in the nucleus of
the host cell
D. Host cell ribosomes attach to the 5’ cap to
initiate translation

• Correct answer: B
• The patient most likely has a rhinovirus
infection based on the following:
• Mucus drainage
• Pharyngitis (post-nasal drip)
• Rhinovirus is the most common
cause of a cold
• Rhinovirus is a positive sense RNA virus
which enables it to be translated upon
entry into the cytoplasm
• Choice A is incorrect because rhinovirus
is acid-labile and would not survive long
enough to be excreted in the feces
• Choice C is incorrect because all RNA viruses
replicate in the cytoplasm
• Choice D is incorrect because picornaviruses
lack a 5’ cap, which is why they require an
internal ribosomal entry site
346

Section 41 - Coxsackieviruses (A & B)

1. Cockfight: Coxsackie A and B 11. Falling and accidentally knocking hat off: can cause
2. Peacock emperor: picornavirus meningitis
3. Icosahedral gem on cane: icosahedral capsid 12. Muscular and enlarged rooster in cart: can cause
4. Warm color: RNA virus myocarditis and heart failure
5. Rainbow: positive sense virus 13. Fire surrounding cart: can cause pericarditis
6. Asteroid breaking into pieces: one big protein is 14. Wolf with fire on chest: chest pain from myocarditis
cleaved into smaller functional proteins or pericarditis
7. Line of henchmen: linear virus 15. Naked pig: naked virus
8. Blood splatters: causes vesicles on the hands, feet 16. Wolf wrapping arms around naked pig: naked
and oral mucosa (hand-foot-mouth disease) viruses enter cell via endocytosis
9. Blood splatters on tail: vesicles can appear on the 17. IRS henchman: contains internal ribosomal entry
buttocks site (IRES)
10. Pig intestine shop: considered an enterovirus
347

Photo Creadit James Heilman, MD [CC BY-SA 4.0 (https://


creativecommons.org/licenses/by-sa/4.0)]
Figure 5.4.104 - Coxsackie-related oropharyngeal
vesicular lesions (herpangina)

REVIEW QUESTIONS ?
Photo Credit: Mrradiostar82 [CC BY-SA 3.0 (https://creativecommons.
org/licenses/by-sa/3.0)] 1. An 8-year-old boy presents to his pediatrician
Figure 5.4.102 - Coxsackie-related palmar vesicular due to high fevers and a rash on his hands and
lesions feet. His temperature is 40°C (104.0°F). Physical
examination reveals multiple vesicles on the
boy’s palms and soles. The boy complains
of chest pain and shortness of breath. The
physician suspects that coxsackie is the culprit
of the boy’s presentation. Which of the
following statements is not true regarding this
patient’s condition?
A. The chest pain may be due to fluid in the
lungs
B. Vesicles may be present on the buttocks
C. Functional virions will not be found in the
patient’s stool
D. Nuchal rigidity may occur in some infected
patients

• Correct answer: C
• Coxsackieviruses are enteroviruses and
can produce functional virions in the
Photo Credit: Ngufra at English Wikipedia [CC BY-SA 3.0 (https://
stool
creativecommons.org/licenses/by-sa/3.0)] • Choices A, B, and D are all true statements
Figure 5.4.103 - Coxsackie-related plantar vesicular regarding Coxsackiviruses
lesions
348

Section 42 - Hepatitis A Virus

1. “A” shaped palanquin: Hepatitis A 16. Panda stepping in poo: fecal-oral transmission
2. Warm colors: RNA virus 17. Panda vomiting in terror: can cause nausea and
3. Rainbow: positive sense vomiting
4. Peacock emperor: picornavirus 18. Turtle slapping cigarette down: active Hepatitis A
5. Icosahedral gem on cane: Icosahedral capsid infections are often associated with aversion to
6. Asteroid breaking into smaller asteroids: one big cigarettes
protein is cleaved into several smaller functional 19. Starving pig: Hepatitis A causes anorexia
proteins 20. Wolf with charred fur on only half his body: often
7. Naked pig: naked virus causes jaundice
8. Net wrapped around pig: naked viruses enter the 21. Cow holding abs: causes RUQ pain
cell via endocytosis 22. Councilman attacking cow: liver biopsy shows
9. IRS henchman: contains internal ribosomal entry Councilman bodies
site (IRES) 23. Needles: prevent infection with vaccination
10. Line of wolves: linear virus 24. Needle poking wolf: vaccinations are warranted as
11. Snowflakes: IgM antibodies against HAV are present post-exposure prophylaxis
during an acute infection 25. Tiles: post-exposure prophylaxis may include
12. Eggs hatching: short incubation period immunoglobulin administration
13. Traveling daycare: Hepatitis A often infects travelers 26. Water: treat active infection with supportive care
14. Daycare children: Hepatitis A often infects daycare 27. Pig in little house: anti-HAV IgG antibodies indicate
children immunity
15. Crab booth: Hepatitis is often spread through
contaminated shellfish
349

REVIEW QUESTIONS ?
1. An 50-year-old female with a 20-pack-year
history presents to the physician complaining
of abdominal pain, nausea and vomiting.
She endorses recent travel to a developing
country. Just prior to returning home to the
United States, she recalls hearing about a viral
outbreak in that area that caused vomiting.
Her temperature is 38.4°C (101.2°F). Physical
examination is significant for right upper
quadrant abdominal pain upon palpation.
Photo Credit: Nephron [CC BY-SA 3.0 (https://creativecommons.org/licenses/ Scleral icterus is absent. She denies a history
by-sa/3.0)]
of IV drug use or unprotected sex. Laboratory
Figure 5.4.105 - Councilman bodies in HAV
results are shown below.
AST: 163 U/L
ALT: 182 U/L

2. Which of the following is true regarding the


pathogen most likely responsible for her
symptoms?
A. The period between exposure and symptom
onset is 3-4 months
B. IgM antibodies will develop against
envelope antigens during the acute phase
C. Hepatitis serology would reveal the
presence of anti-HAV IgG antibodies
D. A liver biopsy may reveal eosinophilic
bodies surrounded by normal cells

• Correct answer: D
• The patient likely has a hepatitis A
infection based on the following:
• RUQ pain
• Elevated AST and ALT
• Nausea and vomiting
• Liver biopsy in patients with hepatitis A
infections will likely reveal ballooning
hepatocytes as well as healthy
hepatocytes surrounding eosinophilic
apoptotic bodies (Councilman bodies)
• Choice A is incorrect because has a short
incubation period (< 1 month)
• Choice B is incorrect because hepatitis A is a
naked virus, not enveloped
• Choice C is incorrect because anti-HAV IgG
antibodies indicate immunity
350

TABLES & FIGURES


Table 5.1.1 - Bacterial structures....................................................................................................................... 6
Figure 5.1.1 - Gram positive bacterial envelope................................................................................................ 7
Figure 5.1.2 - Gram negative bacterial envelope............................................................................................... 7
Figure 5.1.3 - Envelope of Mycoplasma spp...................................................................................................... 8
Figure 5.1.4 - Envelope of Mycobacteria spp.................................................................................................... 8
Figure 5.1.5 - Acid-fast stain of Mycobacterium Tuberculosis .......................................................................... 8
Figure 5.1.6 - Bacterial endospore.................................................................................................................... 9
Table 5.1.2 - Overview of virulence factors..................................................................................................... 10
Figure 5.1.7 - Protein A mechanism................................................................................................................ 11
Figure 5.1.8 - Visual demonstration of Type III secretion systems.................................................................. 12
Table 5.1.3 - Bacterial exotoxins...................................................................................................................... 13
Figure 5.1.9 - Exotoxins that inhibit protein synthesis..................................................................................... 14
Figure 5.1.10 - Exotoxins that increase fluid secretion using adenylate cyclase.............................................14
Figure 5.1.11 - Exotoxin (heat-stable toxin) that increases fluid secretion through guanylate cyclase...........15
Figure 5.1.12 - Pertussis toxin mechanism...................................................................................................... 15
Figure 5.1.13 - Mechanism of tetanospasmin................................................................................................. 16
Figure 5.1.14 - Mechanism of botulinum toxin............................................................................................... 16
Figure 5.1.15 - Mechanism of exotoxins that disrupt cell membranes........................................................... 17
Figure 5.1.16 - Mechanism of superantigens.................................................................................................. 17
Table 5.1.4 - Mechanisms of genetic change in bacteria................................................................................. 20
Figure 5.1.17 - Random mutation in bacteria.................................................................................................. 21
Figure 5.1.18 - Phase variation in bacteria...................................................................................................... 21
Figure 5.1.19 - Transformation in bacteria...................................................................................................... 21
Figure 5.1.20 - Conjugation in bacteria........................................................................................................... 22
Figure 5.1.21 - Generalized transduction (lytic cycle)..................................................................................... 23
Figure 5.1.22 - Specialized transduction (lysogenic cycle)............................................................................... 24
Figure 5.1.23 - Restriction modification.......................................................................................................... 25
Figure 5.1.24 - Brain biopsy with vacuolations from spongiform encephalopathy.........................................28
Figure 5.1.25 - Fungi structure........................................................................................................................ 29
Figure 5.1.26 - Filamentous fungus................................................................................................................. 29
Figure 5.1.27 - Mold (left) and yeast (right).................................................................................................... 30
Figure 5.1.28 - Spores of Aspergillus............................................................................................................... 30
Figure 5.1.29 - Enveloped virus structures...................................................................................................... 32
Figure 5.1.30 - Naked virus structures............................................................................................................. 32
Figure 5.1.31 - Adsorption, penetration, and uncoating of naked viruses......................................................33
Figure 5.1.32 - Adsorption, penetration, and uncoating of enveloped viruses...............................................33
Figure 5.1.33 - Bacteriophage structure.......................................................................................................... 33
Table 5.1.5 - Genetic adjustments and activities in viruses............................................................................. 35
Figure 5.1.34 - Genetic shift and genetic drift................................................................................................. 35
Figure 5.1.35 - Phenotypic mixing................................................................................................................... 36
Figure 5.1.36 - HDV superinfection................................................................................................................. 36
Figure 5.1.37 - Positive-sense and negative-sense viruses.............................................................................. 38
Figure 5.1.38 - Poxvirus cytoplasm replication................................................................................................ 39
Figure 5.1.39 - HBV structure.......................................................................................................................... 39
Figure 5.1.40 - RNA virus replication............................................................................................................... 39
351

Figure 5.1.41 - DNA virus replication............................................................................................................... 40


Figure 5.2.1 - Bacteria overview figure............................................................................................................ 41
Figure 5.2.2 - Gram stain................................................................................................................................. 43
Figure 5.2.3 - Gram-positive bacilli.................................................................................................................. 43
Figure 5.2.4 - Tetanospasmin.......................................................................................................................... 43
Figure 5.2.5 - Gram stain of Clostridium botulinum........................................................................................ 45
Figure 5.2.6 - Gram stain of Clostridium difficile............................................................................................. 47
Figure 5.2.7 - Pseudomembranous colitis ...................................................................................................... 47
Figure 5.2.8 - Gram stain of Clostridium perfringens...................................................................................... 49
Figure 5.2.9 - Gas gangrene ............................................................................................................................ 49
Figure 5.2.10 - Gram stain of Bacillus anthracis.............................................................................................. 51
Figure 5.2.11 - Cutaneous anthrax ................................................................................................................. 51
Figure 5.2.12 - Gram stain of Bacillus cereus................................................................................................... 53
Figure 5.2.13 - Gram stain of gram-positive rods............................................................................................ 55
Figure 5.2.14 - Hemolysis on 5% sheep blood agar......................................................................................... 55
Figure 5.2.15 - Catalase test............................................................................................................................ 55
Figure 5.2.16 - Gram stain of Corynebacterium diphtheriae........................................................................... 57
Figure 5.2.17 - Cystine-tellurite agar............................................................................................................... 57
Figure 5.2.18 - Pseudomembranous pharyngitis............................................................................................. 57
Figure 5.2.19 - Gram stain of Streptococci...................................................................................................... 59
Figure 5.2.20 - Microbial resistance and sensitivity to various compounds....................................................59
Figure 5.2.21 - Splinter hemorrhages.............................................................................................................. 59
Figure 5.2.22 - Quellung reaction.................................................................................................................... 61
Figure 5.2.23 - Otitis media............................................................................................................................. 61
Figure 5.2.24 - Cellulitis................................................................................................................................... 63
Figure 5.2.25 - Impetigo.................................................................................................................................. 63
Figure 5.2.26 - Erysipelas................................................................................................................................. 64
Figure 5.2.27 - Sandpaper-like rash................................................................................................................. 64
Figure 5.2.28 - Strawberry tongue................................................................................................................... 64
Figure 5.2.29 - GAS pharyngitis....................................................................................................................... 64
Figure 5.2.30 - CAMP test................................................................................................................................ 67
Figure 5.2.31 - Gram stain of Enterococci....................................................................................................... 71
Figure 5.2.32 - Gram stain of Staphylococcus.................................................................................................. 73
Figure 5.2.33 - Mannitol fermentation test..................................................................................................... 73
Figure 5.2.34 - Coagulase test tube................................................................................................................. 73
Figure 5.2.35 - Urease test.............................................................................................................................. 75
Figure 5.2.36 - Gram stain of Nocardia........................................................................................................... 79
Figure 5.2.37 - Acid-fast stain of Mycobacterium Tuberculosis (similar to Nocardia).....................................79
Figure 5.2.38 - Gram stain of Actinomyces...................................................................................................... 81
Figure 5.2.39 - Sulfur granule.......................................................................................................................... 81
Figure 5.2.40 - Gram stain of Neisseria........................................................................................................... 82
Figure 5.2.41 - Chocolate agar......................................................................................................................... 82
Figure 5.2.42 - Gram negative bacterial cell wall . .......................................................................................... 83
Figure 5.2.43 - Oxidase test............................................................................................................................. 83
352

Figure 5.2.44 - Chocolate agar and Thayer-Martin agar.................................................................................. 83


Figure 5.2.45 - Gram stain of Moraxella.......................................................................................................... 89
Figure 5.2.46 - Otoscopic view of otitis media................................................................................................ 89
Figure 5.2.47 - Gram Stain of Haemophilus influenzae................................................................................... 91
Figure 5.2.48 - Satellite colonies of Haemophilus influenzae growing near Staphylococcus aureus on blood
agar.................................................................................................................................................................. 91
Figure 5.2.49 - Epiglottis.................................................................................................................................. 91
Figure 5.2.50 - Thumb sign.............................................................................................................................. 92
Figure 5.2.51 - Cherry red epiglottis................................................................................................................ 92
Figure 5.2.52 - Gram stain of Bordetella.......................................................................................................... 95
Figure 5.2.53 - Regan-Lowe agar..................................................................................................................... 95
Figure 5.2.54 - Gram stain of gram-negative coccobacilli................................................................................ 97
Figure 5.2.55 - Granuloma............................................................................................................................... 97
Figure 5.2.56 - Gram stain of Salmonella........................................................................................................ 99
Figure 5.2.57 - TSI agar.................................................................................................................................... 99
Figure 5.2.58 - Peyer’s patches........................................................................................................................ 99
Figure 5.2.59 - Rose spots............................................................................................................................. 101
Figure 5.2.60 - Gram stain of Shigella............................................................................................................ 103
Figure 5.2.61 - Actin...................................................................................................................................... 103
Figure 5.2.62 - Neutrophils............................................................................................................................ 103
Figure 5.2.63 - Schistocytes........................................................................................................................... 103
Figure 5.2.64 - Gram stain of E. coli............................................................................................................... 105
Figure 5.2.65 - Eosin methylene blue aga...................................................................................................... 105
Figure 5.2.66 - MacConkey agar.................................................................................................................... 105
Figure 5.2.67 - Indole test............................................................................................................................. 106
Figure 5.2.68 - Lipopolysaccharide................................................................................................................ 106
Figure 5.2.69 - Mucoid colonies.................................................................................................................... 109
Figure 5.2.70 - Struvite crystals..................................................................................................................... 109
Figure 5.2.71 - Staghorn calculus.................................................................................................................. 111
Figure 5.2.72 - Gram stain of Pseudomonas.................................................................................................. 113
Figure 5.2.73 - Pyoverdin and pyocyanin (bluish-green color) of Pseudomonas (left)..................................113
Figure 5.2.74 - Corneal abrasion................................................................................................................... 113
Figure 5.2.75 - Otitis externa......................................................................................................................... 114
Figure 5.2.76 - Hot tub folliculitis.................................................................................................................. 114
Figure 5.2.77 - Gram stain gram of Yersinia enterocolitica............................................................................ 117
Figure 5.2.78 - Axillary Bubo......................................................................................................................... 117
Figure 5.2.79 - Gram stain of Enterobacter................................................................................................... 119
Figure 5.2.80 - Red pigment from Serratia on a piece of bread.................................................................... 119
Figure 5.2.81 - Warthin-Starry stain.............................................................................................................. 121
Figure 5.2.82 - Gram stain of Legionella........................................................................................................ 123
Figure 5.2.83 - Silver stain of Legionella........................................................................................................ 123
Figure 5.2.84 - Buffered charcoal yeast extract agar..................................................................................... 123
Figure 5.2.85 - Gram stain Campylobacter.................................................................................................... 125
Figure 5.2.86 - Gram stain of Vibrio cholerae................................................................................................ 127
Figure 5.2.87 - “Rice-water” diarrhea........................................................................................................... 127
Figure 5.2.88 - Hematoxylin and eosin (H&E) stain of the gastric mucosa....................................................128
Figure 5.2.89 - Silver based stain................................................................................................................... 128
Figure 5.2.90 - Stomach anatomy and physiology......................................................................................... 129
Figure 5.2.91 - Acid-fast stain of Mycobacterium tuberculosis...................................................................... 131
353

Figure 5.2.92 - Lowenstein-Jensen medium.................................................................................................. 131


Figure 5.2.93 - Pathophysiology of Mycobacterium tuberculosis.................................................................. 132
Figure 5.2.94 - Miliary tuberculosis............................................................................................................... 133
Figure 5.2.95 - “Serpentine cords”................................................................................................................ 133
Figure 5.2.96 - PPD test................................................................................................................................. 133
Figure 5.2.97 - Ghon complex....................................................................................................................... 133
Figure 5.2.98 - Acid-fast stain of Mycobacterium leprae............................................................................... 135
Figure 5.2.99 - Pathophysiology of Mycobacterium leprae........................................................................... 135
Figure 5.2.100 - Leonine Facies..................................................................................................................... 136
Figure 5.2.101 - Acid-fast stain of Mycobacterium avium-intracellulare.......................................................139
Figure 5.2.102 - Spirochetes via dark-field microscopy................................................................................. 140
Figure 5.2.103 -  Ixodes deer tick................................................................................................................... 140
Figure 5.2.104 - Giemsa stain of Borrelia...................................................................................................... 141
Figure 5.2.105 - Erythema migrans............................................................................................................... 141
Figure 5.2.106 - Bell’s palsy........................................................................................................................... 141
Figure 5.2.107 - Electron microscopic image of Leptospira interrogans.......................................................143
Figure 5.2.108 - Dark-field photomicrograph of Treponema pallidum..........................................................145
Figure 5.2.109 - Chancre............................................................................................................................... 145
Figure 5.2.110 - Maculopapular rash............................................................................................................ 145
Figure 5.2.111 - Maculopapular rash on the palms....................................................................................... 145
Figure 5.2.112 - Condyloma lata.................................................................................................................... 146
Figure 5.2.113 - Gumma................................................................................................................................ 146
Figure 5.2.114 - Cross section of the spinal cord........................................................................................... 146
Figure 5.2.115 - Notched teeth..................................................................................................................... 146
Figure 5.2.116 - Saddle nose......................................................................................................................... 147
Figure 5.2.117 - Rhagades............................................................................................................................. 147
Figure 5.2.118 - Life cycle of Chlamydia trachomatis.................................................................................... 149
Figure 5.2.119 - Giemsa stain of Chlamydia.................................................................................................. 149
Figure 5.2.120 -  Gram stain of Coxiella burnetii........................................................................................... 151
Figure 5.2.121 -  Osler’s nodes...................................................................................................................... 151
Figure 5.2.122 - Clue cells.............................................................................................................................. 153
Figure 5.2.123 - IgM antibodies..................................................................................................................... 154
Figure 5.2.124 - Rocky Mountain Spotted Fever........................................................................................... 156
Figure 5.2.125 - Weil Felix Test...................................................................................................................... 157
Figure 5.2.126 - Ehrlichia within a monocyte................................................................................................ 159
Figure 5.2.127 - Anaplasma within a granulocyte......................................................................................... 159
Table 5.2.1 - Miscellaneous organisms.......................................................................................................... 162
Figure 5.2.128 - Intestinal biopsy in Whipple disease................................................................................... 163
Figure 5.2.129 - Granuloma inguinale caused by Klebsiella granulomatis (male).........................................163
Figure 5.2.130 - Granuloma inguinale caused by Klebsiella granulomatis (female)......................................163
Figure 5.2.131 - Bipolar Donovan bodies from granuloma inguinale ulcer biopsy........................................163
Figure 5.3.1 - Fungi overview figure.............................................................................................................. 165
Figure 5.3.2 - Histoplasma capsulatum yeast in macrophages..................................................................... 166
Figure 5.3.3 - Mold (left) and yeast (right).................................................................................................... 167
Figure 5.3.4 - Verrucous skin lesion............................................................................................................... 169
Figure 5.3.5 - Squamous cell carcinoma........................................................................................................ 169
Figure 5.3.6 - Granulomatous nodule............................................................................................................ 170
Figure 5.3.7 - Broad-based budding by Blastomyces dermatitidis................................................................ 170
Figure 5.3.8 - Endospore of Coccidioides immitis.......................................................................................... 172
354

Figure 5.3.9 - Coccidioides using the methenamine silver stain.................................................................... 173


Figure 5.3.10 - Erythema multiforme............................................................................................................ 173
Figure 5.3.11 - Erythema nodosum............................................................................................................... 173
Figure 5.3.12 - Budding cells of Paracoccidioides brasiliensis....................................................................... 175
Figure 5.3.13 - Hyphae and spores of a dermatophyte................................................................................. 177
Figure 5.3.14 - Tinea capitis........................................................................................................................... 177
Figure 5.3.15 - Tinea corporis........................................................................................................................ 177
Figure 5.3.16 - Tinea pedis............................................................................................................................ 177
Figure 5.3.17 - Vesicular athlete’s foot ......................................................................................................... 177
Figure 5.3.18 - Interdigital tinea pedis........................................................................................................... 178
Figure 5.3.19 - Onychomycosis...................................................................................................................... 178
Figure 5.3.20 - Tinea cruris............................................................................................................................ 178
Figure 5.3.21 - Tinea versicolor skin lesions.................................................................................................. 181
Figure 5.3.22 - Microscopic image of Malassezia furfur “Spaghetti and meatballs” ....................................181
Figure 5.3.23 - Sporothrix schenckii (yeast form).......................................................................................... 183
Figure 5.3.24 - Sporothrix schenckii (mold form).......................................................................................... 183
Figure 5.3.25 - Sporothrix schenckii lesion along draining lymphatic channels ............................................183
Figure 5.3.26 - Pseudohyphae and budding yeast......................................................................................... 185
Figure 5.3.27 - Germ tube formation............................................................................................................ 185
Figure 5.3.28 - Oral thrush............................................................................................................................ 186
Figure 5.3.29 - Esophageal candidiasis ......................................................................................................... 186
Figure 5.3.30 - Vulvovaginitis caused by Candida albicans............................................................................ 186
Figure 5.3.31 - Microscopic morphology of Aspergillus fumigatus............................................................... 189
Figure 5.3.32 - Aspergilloma (gross image)................................................................................................... 189
Figure 5.3.33 - Aspergilloma (imaging).......................................................................................................... 189
Figure 5.3.34 - India ink stain........................................................................................................................ 191
Figure 5.3.35 - Sabouraud dextrose agar....................................................................................................... 191
Figure 5.3.36 - Mucicarmine stain................................................................................................................. 191
Figure 5.3.37 - Microscopy of mucormycosis................................................................................................ 193
Figure 5.3.38 - Silver stain of Pneumocystis jirovecii cysts............................................................................ 195
Figure 5.3.39 - Chest x-rays of Pneumocystis pneumonia with “ground glass” opacities .............................195
Figure 5.3.40 - Pneumatoceles...................................................................................................................... 195
Figure 5.3.41 - Fluorescent antibody stain.................................................................................................... 195
Figure 5.4.1 - Parasites overview figure........................................................................................................ 197
Figure 5.4.2 - Giardia lamblia cyst................................................................................................................. 199
Figure 5.4.3 - Giardia lamblia trophozoite.................................................................................................... 199
Figure 5.4.4 - Red blood cells within Entamoeba cytoplasm (erythrophagocytosis).....................................201
Figure 5.4.5 - Entamoeba egg in stool sample.............................................................................................. 201
Figure 5.4.6 - Cryptosporidium oocysts with acid-fast staining..................................................................... 203
Figure 5.4.7 - Trypomastigotes on peripheral blood smear........................................................................... 209
Figure 5.4.8 - Malaria Life Cycle.................................................................................................................... 211
Figure 5.4.9 - Plasmodium trophozoite ring form within red blood cells .....................................................212
Figure 5.4.10 - Babesia trophozoite ring form (“Maltese cross”) within red blood cells...............................215
Figure 5.4.11 - Barium esophagram showing achalasia................................................................................ 216
Figure 5.4.12 - Cutaneous leishmaniasis....................................................................................................... 219
Figure 5.4.13 - Flagellated Gardnerella trophozoites.................................................................................... 221
Figure 5.4.14 - Gardnerella vaginalis on saline microscopy.......................................................................... 221
Figure 5.4.15 - Pinworm egg.......................................................................................................................... 223
Figure 5.4.16 - Life cycle of parasites that cause Löffler syndrome............................................................... 224
355

Figure 5.4.17 - Ascaris lumbricoides egg....................................................................................................... 225


Figure 5.4.18 - Strongyloides stercoralis larva............................................................................................... 227
Figure 5.4.19 - Hookworm egg...................................................................................................................... 229
Figure 5.4.20 - Cutaneous hookworm lesion................................................................................................. 229
Figure 5.4.21 - Striated muscle tissue infected with Trichinella spiralis........................................................231
Figure 5.4.22 - Periorbital edema.................................................................................................................. 231
Figure 5.4.23 - Whipworm egg...................................................................................................................... 233
Figure 5.4.24 - Blackfly.................................................................................................................................. 237
Figure 5.4.25 - Elephantiasis.......................................................................................................................... 239
Figure 5.4.26 - Taenia solium life cycle.......................................................................................................... 241
Figure 5.4.27 - Taenia solium on microscopy................................................................................................ 241
Figure 5.4.28 - Taenia solium egg in stool sample......................................................................................... 241
Figure 5.4.29 - Neurocysticercosis on imaging.............................................................................................. 242
Figure 5.4.30 - Echinococcus granulosus life cycle........................................................................................ 244
Figure 5.4.31 - Echinococcus granulosus tapeworm on microscopy............................................................. 245
Figure 5.4.32 - Liver hydatid cyst on MRI...................................................................................................... 245
Figure 5.4.33 - Liver hydatid cyst (gross)....................................................................................................... 245
Figure 5.4.34 - Schistosoma life cycle............................................................................................................ 247
Figure 5.4.35 - Lateral spine on Schistosoma egg (mansoni or japanicom)...................................................247
Figure 5.4.36 - Lateral spine on Schistosoma haematobium egg.................................................................. 247
Figure 5.4.37 - Scabies-related erythematous papules................................................................................. 251
Figure 5.4.38 - Pediculus capitis louse........................................................................................................... 253
Figure 5.4.39 - Pediculus nit on hair shaft..................................................................................................... 253
Figure 5.4.40 - Pediculus pubis louse............................................................................................................. 253
Figure 5.4.41 - Pediculus corporis louse........................................................................................................ 253
Figure 5.4.42 - Virus overview figure............................................................................................................. 254
Figure 5.4.43 - Trigeminal ganglion............................................................................................................... 256
Figure 5.4.44 - Gingivostomatitis................................................................................................................... 256
Figure 5.4.45 - Viral keratoconjunctivitis....................................................................................................... 256
Figure 5.4.46 - Herpetic whitlow................................................................................................................... 256
Figure 5.4.47 - Herpes labialis....................................................................................................................... 257
Figure 5.4.48 - Erythema multiforme............................................................................................................ 257
Figure 5.4.49 - HSV-related esophageal ulcers.............................................................................................. 257
Figure 5.4.50 - MRI of temporal lobe encephalitis........................................................................................ 258
Figure 5.4.51 - Neonatal herpes.................................................................................................................... 258
Figure 5.4.52 - Vesicular and ulcerative herpes lesions................................................................................. 258
Figure 5.4.53 - Multinucleated giant cells on Tzanck smear.......................................................................... 258
Figure 5.4.54 - Cowdry bodies in hepatocytes.............................................................................................. 258
Figure 5.4.55 - Chickenpox............................................................................................................................ 261
Figure 5.4.56 - Shingles................................................................................................................................. 261
Figure 5.4.57 - Herpes zoster ophthalmicus.................................................................................................. 261
Figure 5.4.58 - Cytotoxic T-cells on EBV-related peripheral blood smear......................................................263
Figure 5.4.59 - Starry sky appearance on biopsy of Burkitt’s lymphoma......................................................263
Figure 5.4.60 - MRI showing ring-enhancing lesion...................................................................................... 263
Figure 5.4.61 - CMV retinitis.......................................................................................................................... 265
Figure 5.4.62 - “Blueberry muffin” rash........................................................................................................ 265
Figure 5.4.63 - Periventricular calcifications on CT imaging.......................................................................... 265
Figure 5.4.64 - Owl’s eye appearance on lung biopsy................................................................................... 265
Figure 5.4.65 - Skin lesions in Kaposi’s sarcoma............................................................................................ 269
356

Figure 5.4.66 - HBV structure........................................................................................................................ 271


Table 5.4.1 - Descriptions of serologic markers in Hepatitis B....................................................................... 271
Figure 5.4.67 - Serological markers in acute hepatitis B infection................................................................. 272
Figure 5.4.68 - Serological markers in chronic hepatitis B infection.............................................................. 272
Figure 5.4.69 - H&E stain showing ground glass appearance........................................................................ 273
Figure 5.4.70 - Reverse transcriptase in HBV................................................................................................. 273
Figure 5.4.71 - Smallpox................................................................................................................................ 275
Figure 5.4.72 - Umbilicated lesions in Molluscum contagiosum................................................................... 275
Figure 5.4.73 - Plantar warts......................................................................................................................... 277
Figure 5.4.74 - Anal warts.............................................................................................................................. 277
Figure 5.4.75 - HPV carcinogenesis mechanism............................................................................................ 278
Figure 5.4.75 - Koilocytes on Pap smear........................................................................................................ 278
Figure 5.4.77 - Progressive multifocal leukoencephalopathy (PML) on MRI.................................................283
Figure 5.4.78 - Fifth disease........................................................................................................................... 285
Figure 5.4.79 - HIV structure......................................................................................................................... 289
Figure 5.4.80 - HIV replication cycle.............................................................................................................. 290
Figure 5.4.81 - Time course of HIV infection................................................................................................. 290
Figure 5.4.82 - HIV diagnosis......................................................................................................................... 291
Figure 5.4.83 - Fat and lymphoid aggregates seen on HCV liver biopsy........................................................295
Figure 5.4.84 - Lichen planus......................................................................................................................... 295
Figure 5.4.85 - Maculopapular rash of Rubella............................................................................................. 305
Figure 5.4.86 - Chikungunya-related erythematous maculopapular rash.....................................................307
Figure 5.4.87 - Genetic shift and genetic drift............................................................................................... 315
Figure 5.4.88 - Acute HDV-HBV coinfection................................................................................................... 318
Figure 5.4.89 - HDV superinfection............................................................................................................... 318
Figure 5.4.90 - Steeple sign on radiograph in croup...................................................................................... 321
Figure 5.4.91 - Bronchiolitis on chest radiograph.......................................................................................... 323
Figure 5.4.92 - Maculopapular rash in measles............................................................................................. 325
Figure 5.4.93 - Koplik’s spots......................................................................................................................... 325
Figure 5.4.94 - Giant cells.............................................................................................................................. 325
Figure 5.4.95 - Mumps-related parotitis....................................................................................................... 327
Figure 5.4.96 - Rabies virus structure............................................................................................................ 329
Figure 5.4.97 - Rabies virus bullet-shaped envelopes on electron microscopy.............................................329
Figure 5.4.98 - Rabies virus Negri bodies...................................................................................................... 329
Figure 5.4.99 - Retrograde movement of Rabies virus.................................................................................. 330
Figure 5.4.100 - Positive-sense and negative-sense viruses.......................................................................... 339
Figure 5.4.101 - Transmission electron micrograph of Rotavirus.................................................................. 339
Figure 5.4.102 - Coxsackie-related palmar vesicular lesions......................................................................... 347
Figure 5.4.103 - Coxsackie-related plantar vesicular lesions......................................................................... 347
Figure 5.4.104 - Coxsackie-related oropharyngeal vesicular lesions (herpangina).......................................347
Figure 5.4.105 - Councilman bodies in HAV.................................................................................................. 349
357

You might also like